Вы находитесь на странице: 1из 52

.

By-Laws are indispensable to corporations, since they are required by law


LOYOLA GRAND VILLAS HOMEOWNERS (SOUTH) for an orderly management of corporations. However, failure to file them
ASSOCIATION, INC., petitioner, vs.HON. COURT OF APPEALS, within the period prescribed does not equate to the automatic dissolution
HOME INSURANCE AND GUARANTY CORPORATION, EMDEN of a corporation
ENCARNACION and HORATIO AYCARDO, respondents.
SAWADJAAN
G.R. No. 117188 August 7, 1997 Vs.
COURT OF APPEALS,
ROMERO, J.: G.R. No. 141735

Loyola Grand Villas Homeowners Association, Inc. (LGVHAI) was FACTS OF THE CASE
organized on 8 February 1983 as the homeoenwers' association for Loyola
Grand Villas. It was also registered as the sole homeowners' association in Petitioner Sappari K. Sawadjaan was among the first employees of the
the said village with the Home Financing Corporation (which eventually Philippine Amanah Bank (PAB) when it was created by virtue of
became Home Insurance Guarantee Corporation ["HIGC"]). However, the Presidential Decree No. 264. Sometime in 1988, while still designated as
association was not able file its corporate by-laws. appraiser/investigator, he was assigned to inspect the properties offered as
collaterals by Compressed Air Machineries and Equipment Corporation
The LGVHAI officers then tried to registered its By-Laws in 1988, but (CAMEC) for a credit line of Five Million Pesos (P5,000,000.00). The
they failed to do so. They then discovered that there were two other properties consisted of two parcels of land covered by Transfer
homeowners' organizations within the subdivision - the Loyola Grand Certificates of Title (TCTs). On the basis of his Inspection and Appraisal
Villas Homeowners (North) Association, Inc. [North Association] and Report, the PAB granted the loan application. When the loan matured,
herein Petitioner Loyola Grand Villas Homeowners (South) Association, CAMEC requested an extension to repay the loan.
Inc.["South Association]. In January 1990, Congress passed Republic Act 6848 creating the AIIBP
and repealing P.D. No. 264 (which created the PAB). All assets, liabilities
Upon inquiry by the LGVHAI to HIGC, it was discovered that LGVHAI and capital accounts of the PAB were transferred to the AIIBP, and the
was dissolved for its failure to submit its by-laws within the period existing personnel of the PAB were to continue to discharge their
required by the Corporation Code and for its non-user of corporate charter functions unless discharged. In the ensuing reorganization, Sawadjaan was
because HIGC had not received any report on the association's activities. among the personnel retained by the AIIBP.
These paved the way for the formation of the North and South When CAMEC failed to pay despite the given extension, the bank, now
Associations. referred to as the AIIBP, discovered that one of the TCT was spurious, the
property described therein non-existent, and that the property covered by
LGVHAI then lodged a complaint with HIGC Hearing Officer Danilo the other TCT had a prior existing mortgage.
Javier, and questioned the revocation of its registration. Hearing Officer The Board of Directors of the AIIBP created an Investigating Committee
Javier ruled in favor of LGVHAI, revoking the registration of the North to look into the CAMEC transaction, which had cost the bank Six Million
and South Associations. Pesos (P6, 000,000.00) in losses. The Board of Directors of AIIBP
adopted Resolution No. 2309 finding petitioner guilty of Dishonesty in the
Petitioner South Association appealed the ruling, contending that Performance of Official Duties and/or Conduct Prejudicial to the Best
LGVHAI's failure to file its by-laws within the period prescribed by Interest of the Service and imposing the penalty of Dismissal from the
Section 46 of the Corporation Code effectively automatically dissolved the Service. On reconsideration, they adopted the Resolution No. 2332
corporation. The Appeals Board of the HIGC and the Court of Appeals reducing the penalty imposed on petitioner from dismissal to suspension
both rejected the contention of the Petitioner affirmed the decision of for a period of six (6) months and one (1) day.
Hearing Officer Javier. Petitioner filed a notice of appeal to the Merit System Protection Board
(MSPB). The CSC adopted Resolution No. 94-4483 dismissing the appeal
Issue: for lack of merit and affirming Resolution No. 2309 of the Board of
Directors of AIIBP. The CSC adopted Resolution No. 95-2574 denying
W/N LGVHAI's failure to file its by-laws within the period prescribed by petitioner’s Motion for Reconsideration. Hence this petition for certiorari
Section 46 of the Corporation Code had the effect of automatically under Rule 65 of the Rules of Court.
dissolving the said corporation.
ISSUE
Ruling: No. Whether or not the failure of AIIBP to file its by-laws within the period
prescribed results to a nullity of all actions and proceedings it has initiated.
The pertinent provision of the Corporation Code that is the focal point of
controversy in this case states: RULING
Sec. 46. Adoption of by-laws. - Every corporation formed under this Petitioner’s efforts are unavailing, and we deny his petition for its
Code, must within one (1) month after receipt of official notice of the procedural and substantive flaws. Petitioner’s recurrent argument, tenuous
issuance of its certificate of incorporation by the Securities and Exchange at its 128
Commission, adopt a code of by-laws for its government not inconsistent
with this Code. very best, is premised on the fact that since respondent AIIBP failed to file
Ordinarily, the word "must" connotes an imposition of duty which must be its by-laws within the designated 60 days from the effectively of Rep. Act
enforced. However, the word "must" in a statute, like "shall," is not always No. 6848, all proceedings initiated by AIIBP and all actions resulting
imperative. It may be consistent with an ecercise of discretion. If the therefrom are a patent nullity. Petitioner already raised the question of
language of a statute, considered as a whole with due regard to its nature AIIBP’s corporate existence and lack of jurisdiction in his Motion for
and object, reveals that the legislature intended to use the words "shall" New Trial/Motion for Reconsideration and was denied by the Court of
and "must" to be directory, they should be given that meaning. Appeals. Despite the volume of pleadings he has submitted thus far, he has
added nothing substantial to his arguments.
The legislative deliberations of the Corporation Code reveals that it was The AIIBP was created by Rep. Act No. 6848. It has a main office where
not the intention of Congress to automatically dissolve a corporation for it conducts business, has shareholders, corporate officers, a board of
failure to file the By-Laws on time. directors, assets, and personnel. It is, in fact, here represented by the
Office of the Government Corporate Counsel, "the principal law office of
Moreover, By-Laws may be necessary to govern the corporation, but By- government-owned corporations, one of which is respondent bank." At the
Laws are still subordinate to the Articles of Incorporation and the very least, by its failure to submit its by-laws on time, the AIIBP may be
Corporation Code. In fact, there are cases where By-Laws are unnecessary considered a de facto corporation who’s right to exercise corporate powers
to the corporate existence and to the valid exercise of corporate powers. may not be inquired into collaterally in any private suit to which such
corporations may be a party.
The Corporation Code does not expressly provide for the effects of non- Moreover, a corporation which has failed to file its by-laws within the
filing of By-Laws. However, these have been rectified by Section 6 of PD prescribed period does not ipso facto lose its powers as such. The SEC
902-A which provides that SEC shall possess the power to suspend or Rules on Suspension/Revocation of the Certificate of Registration of
revoke, after proper notice and hearing, the franchise or certificate of Corporations, details the procedures and remedies that may be availed of
registration of corporations upon failure to file By-Laws within the before an order of revocation can be issued. There is no showing that such
required period. a procedure has been initiated in this case.
Wherefore, the petition is dismissed
This shows that there must be notice and hearing before a corporation is
dissolved for failure to file its By-Laws. Even assuming that the existence PMI COLLEGES, petitioner, vs. THE NATIONAL LABOR
of a ground, the penalty is not necessarily revocation, but may only be RELATIONS COMMISSION and ALEJANDRO
suspension. GALVAN, respondents.
277 SCRA 462 – Business Organization – Corporation Law – By-laws and Whether the acts of respondent corporation merit its dissolution or
Innocent Third Persons deprivation of its corporate franchise and to exclude it from all corporate
In 1991, PMI Colleges hired the services of Alejandro Galvan for the rights and privileges
latter to teach in said institution. However, for unknown reasons, PMI
defaulted from paying the remunerations due to Galvan. Galvan made HELD:
demands but were ignored by PMI. Eventually, Galvan filed a labor case
against PMI. Galvan got a favorable judgment from the Labor Arbiter; this SUSTAINED only as to administering of real property not owned by it
was affirmed by the National Labor Relations Commission. On appeal, and when permitted by contract.
PMI reiterated, among others, that the employment of Galvan is void
because it did not comply with its by-laws. Apparently, the by-laws Causes of action:
require that an employment contract must be signed by the Chairman of 1) Alleged illegal holding of real property for a period exceeding
the Board of PMI. PMI asserts that Galvan’s employment contract was not five years from receipt of title-Cause of delay is not
signed by the Chairman of the Board. respondent’s fault
ISSUE: Whether or not Galvan’s employment contract is void.
HELD: No. PMI Colleges never even presented a copy of the by-laws to 2) That respondent is owning and holding a business lot with the
prove the existence of such provision. But even if it did, the employment structure thereon in excess of its reasonable requirements and in
contract cannot be rendered invalid just because it does not bear the contravention of Sec. 13(5) of Corpo. Law- WITHOUT MERIT
signature of the Chairman of the Board of PMI. By-Laws operate merely Every corporation has the power to purchase, hold and lease such real
as internal rules among the stockholders, they cannot affect or prejudice property as the transaction would of the lawful business may reasonably
third persons who deal with the corporation, unless they have knowledge and necessarily require.
of the same. In this case, PMI was not able to prove that Galvan knew of
said provision in the by-laws when he was employed by PMI. 3) That respondent is engaged in activities foreign to the purposes
for which the corporation was created and not reasonably
The Government of the Philippine Islands vs. El Hogar Filipino necessary to its legitimate ends-VALID
G.R. No. L-26649 July 13, 1917 The administration of property, payment of real estate taxes, causing
necessary repairs, managing real properties of non-borrowing shareholders
FACTS: is more befitting to the business of a real estate agent or a trust company
than a building and loan association.
The Philippine Commission enacted Act No. 1459, also known as the
Corporation Law, on March 1, 1906. El Hogar Filipino, organized in 1911 4) That the by-laws of the association stating that, “the board of
under the laws of the Philippine Islands, was the first corporation directors by the vote of an absolute majority of its members is
organized under Sec. 171-190 Act No. 1459, devoted to the subject of empowered to cancel shares and to return the balance to the
building and loan associations, their organization and administration. In owner by reason of their conduct or any other motive or
the said law, the capital of the corporation was not permitted to exceed liquidation” is in direct conflict with Sec. 187 of the Corporation
P3M, but Act No. 2092 amended the statute, permitting capitalization to Law which provides that the board of directors shall not have
the amount of ten millions. the power to force the surrender and withdrawal of unmatured
El Hogar took advantage of the amendment of Act No. 1459 and stock except in case of liquidation or forfeiture of stock for
amended its AOI as a result thereof, stating that the amount of capital must delinquency-WITHOUT MERIT
not exceed what has been stated in Act No. 2092. This resulted to El There is no provision of law making it a misdemeanor to incorporate an
Hogar having 5,826 shareholders, 125,750 shares with paid-up value of invalid provision in the by-laws of a corporation; and if there were such,
P8.7M. The corporation paid P7.16M to its withdrawing stockholders. the hazards incident to corporate effort would be largely increased.
The Government of the Philippine Islands filed an action against
El Hogar due to the alleged illegal holding title to real property for a 5) Art. 61 of El Hogar’s by-laws which states that “ attendance in
period exceeding five (5) years after the same was bought in a foreclosure person or by proxy by shareholders owning one-half plus one of
sale. Sec. 13(5) of the Corporation Law states that corporations must the shareholders shall be necessary to constitute a quorum for
dispose of real estate obtained within 5 years from receiving the title. The the election of directors” is contrary to Sec. 31 of the Corpo Law
Philippine Government also prays that El Hogar be excluded from all which provides that owners of the majority of the subscribed
corporate rights and privileges and effecting a final dissolution of said capital stock entitled to vote must be present either in person or
corporation. by proxy at all elections of directors- WITHOUT MERIT
It appears from the records that El Hogar was the holder of a No fault can be imputed to the corporation on account of the failure of the
recorded mortgage on the San Clemente land as security for a P24K loan shareholders to attend the annual meetings and their non-attendance in
to El Hogar. However, shareholders and borrowers defaulted in payment meetings is doubtless to be interpreted in part as expressing their
so El Hogar foreclosed the mortgage and purchased the land during the satisfaction of the way in which things have been conducted. Mere failure
auction sale. A deed of conveyance in favor of El Hogar was executed and of a corporation to elect officers does not terminate the terms of existing
sent to the Register of Deeds of Tralac with a request that the certificate of officers nor dissolve the corporation. The general rule is to allow the
title be cancelled and a new one be issued in favor of El Hogar from the officer to holdover until his successor is duly qualified.
Register of Deeds of Tarlac. However, no reply was received. El Hogar
filed a complaint with the Chief of the General Land Registration Office. 6) That the directors of El Hogar, instead of receiving nominal pay
The certificate of title to the San Clemente land was received by El Hogar or serving without pay, have been receiving large compensation,
and a board resolution authorizing Benzon to find a buyer was issued. varying in amount from time to time, out of respondents’
Alcantara, the buyer of the land, was given extension of time to make profits- WITHOUT MERIT
payment but defaulted so the contract treated rescinded. Efforts were made With the growth of the corporation, the amount paid as compensation to
to find another buyer. Respondent acquired title in December 1920 until the directors has increased beyond what would probably be necessary is a
the property was finally sold to Felipa Alberto in July 1926. The interval matter that cannot be corrected in this action. Nor can it properly be made
exceeded 5 years but the period did not commence to run until May 7, a basis for depriving respondent of its franchise or enjoining it from
1921 when the register of deeds delivered the new certificate of title. It has compliance with the provisions of its own by-laws. If a mistake has been
been held that a purchaser of land registered under the Torrens system made, the remedy is to lie rather in publicity and competition.
cannot acquire the status of an innocent purchaser for value unless the
vendor is able to place the owner’s duplicate in his hands showing the title 7) That the promoter and organizer of El Hogar was Mr. Antonio
to be in the vendor. During the period before May 1921, El Hogar was not Melian and that in the early stages of the organization of the
in a position to pass an indefeasible title to any purchaser. Therefore, El association, the board of directors authorized the association to
Hogar cannot be held accountable for this delay which was not due to its make a contract with him and that the royalty given to him as
fault. Likewise, the period from March 25, 1926 to April 20, 1926 must founder is “unconscionable, excessive and out of proportion to
not be part of the five-year period because this was the period where the services rendered”-NOT SUSTAINED
respondent was under the obligation to sell the property to Alcantara prior The mere fact that compensation is in excess of what may be considered
to the contract’s rescission due to Alcantara’s non-payment. appropriate is not a proper consideration for the court to resolve. That El
Another circumstance causing the delay is the fact that El Hogar Hogar is in contact with its promoter did not affect the association’s legal
purchased the property in the full amount of the loan made by the former character. The court is of the opinion that the traditional respect for the
owner which is nearly P24K when it was subsequently found that the sanctity of the contract obligation should prevail over the radical and
property was not salable and later sold for P6K notwithstanding El innovating tendencies.
Hogar’s efforts to find a purchaser upon better terms.
8) That Art. 70 of El Hogar’s by-laws, requiring persons elected as
ISSUE: board of directors to be holders of shares of the paid up value of
P5,000 which shall be held as security, is objectionable since a
poor member or wage earner cannot serve as a director
irrespective of other qualifications- NOT SUSTAINED
Corpo. Law expressly gives the power to the corporation to provide in its no matter for judicial interference and much less could the resumption of
by-laws for the qualification of its directors and the requirement of the franchise be justified on this ground.
security from them for the proper discharge of the duties of their pffice in
the manner prescribed in Art. 70 is highly prudent and in conformity with 16) That various outstanding loans have been made by the respondent to
good practice. corporations and partnerships and such entities subscribed to respondents’
shares for the sole purpose of obtaining such loans-NO MERIT
9) That respondent abused its franchise in issuing “special” shares Sec. 173 of Corpo Law declares that “any person” may become a
alleged to be illegal and inconsistent with the plan and purposes stockholder in building and loan associations. The phrase ANY PERSON
of building and loan associations- WITHOUT MERIT does not prevent a finding that the phrase may not be taken in its proper
The said special shares are generally known as advance payment shares and broad sense of either a natural or artificial person.
which were evidently created for the purpose of meeting the condition
caused by the prepayment of dues that is permitted. Sec. 178 of Corpo 17) That in disposing real estate purchased by it, some of the properties
Law allows payment of dues or interest to be paid in advance but the were sold on credit and the persons and entities to which it was sold are
corporation shall not allow interest on advance payment grater than 6% not members nor shareholders nor were they made members or
per annum nor for a period longer than one year. The amount is satisfied shareholders, contrary to the provision of Corpo Law requiring requiring
by applying a portion of the shareholder’s participation in the annual loans to be stockholders only- NOT SUSTAINED
earnings.The mission of special shares does not involve any violation of The law does not prescribe that the property must be sold for cash or that
the principle that the shares must be sold at par. the purchaser shall be a shareholder in the corporation. Such sales can be
made upon the terms and conditions approved by the parties.
10) That in making purchases at foreclosure sales constituting as
security for 54 of the loans, El Hogar bids the full amount after Respondent is enjoined in the future from administering real property
deducting the withdrawal value, alleged to be pusuing a policy not owned by itself, except as may be permitted to it by contract when a
of depreciating at the rate of 10 percent per annum, the value of borrowing shareholder defaults in his obligation. In all other respects, the
the real properties it acquired and that this rate is excessive- complaint is DISMISSED.
UNSUSTAINABLE
The board of directors possess discretion in this matter. There is no Thomson vs. Court of Appeals, 298 SCRA 280 (1998)
provision of law prohibiting the association from writing off a reasonable Ponente: Justice Quisumbing
amount for depreciation on its assets for the purpose of determining its
real profits. Art. 74 of its by-laws expressly authorizes the board of Facts:
directors to determine each year the amount to be written down upon the Petitioner Marsh Thomson (Thomson) was the Executive Vice-President
expenses for the installation and the property of the corporation. The court and, later on, the Management Consultant of private respondent, the
cannot control the discretion of the board of directors about an American Chamber of Commerce of the Philippines, Inc. (AmCham) for
administrative matter as to which they have no legitimate power of action. over ten years. While petitioner was still working with private respondent,
his superior, A. Lewis Burridge, retired as AmCham's President. Before
11) That respondent maintains excessive reserve funds- Burridge decided to return to his home country, he wanted to transfer his
UNFOUNDED proprietary share in the Manila Polo Club (MPC) to petitioner. However,
The function of this fund is to insure stockholders against losses. When through the intercession of Burridge, private respondent paid for the share
the reserves become excessive, the remedy is in the hands of the but had it listed in petitioner's name. This was made clear in an
Legislature. employment advice dated January 13, 1986, wherein petitioner was
No prudent person would be inclined to take a policy in a informed by private respondent. On April 25, 1986, Burridge transferred
company which had so improvidently conducted its affairs that it only said proprietary share to petitioner, as confirmed in a letter of notification
retained a fund barely sufficient to pay its present liabilities and therefore to the Manila Polo Club. Upon his admission as a new member of the
was in a condition where any change by the reduction of interest upon or MPC, petitioner paid the transfer fee of P40,000.00 from his own funds;
depreciation in the value of securities or increase of mortality would but private respondent subsequently reimbursed this amount. On
render it insolvent and subject to be placed in the hands of a receiver. November 19, 1986, MPC issued Proprietary Membership Certificate
Number 3398 in favor of petitioner. But petitioner, however, failed to
12) That the board of directors has settled upon the unlawful policy of execute a document recognizing private respondent's beneficial ownership
paying a straight annual dividend of 10 percent per centum regardless of over said share. Following AmCham's policy and practice, there was a
losses suffered and profits made by the corporation, in contravention with yearly renewal of employment contract between the petitioner and private
the requirements of Sec. 188 of the Corpo law- UNFOUNDED respondent. But petitioner never acknowledged that private respondent is
As provided in the previous cause of action, the profits and losses shall be the beneficial owner of the share. When petitioner's contract of
determined by the board of directors and this means that they shall employment was up for renewal in 1989, he notified private respondent
exercise the usual discretion of good businessmen in allocating a portion that he would no longer be available. The private respondent asked the
of the annual profits to purposes needful of the welfare of the association. petitioner to stay on for another six (6) months. Pending the negotiation
The law contemplates distribution of earnings and losses after legitimate for the consultancy arrangement, private respondent executed a Release
obligations have been met. and Quitclaim, stating that AMCHAM, its directors, officers and assigns,
employees and/or representatives do hereby release, waive, abandon and
13) That El Hogar has made loans to the knowledge of its officers which discharge J. MARSH THOMSON from any and all existing claims. The
were intended to be used by the borrowers for other purposes than the quitclaim, expressed in general terms, did not mention specifically the
building of homes and no attempt has been made to control the borrowers MPC share. On April 5, 1990, private respondent, through counsel sent a
with respect to the use made of the borrowed funds- UNFOUNDED letter to the petitioner demanding the return and delivery of the MPC
There is no statute expressly declaring that loans may be made by these share. Failing to get a favorable response, private respondent filed on May
associations SOLELY for the purpose of building homes. The building of 15, 1990, a complaint against petitioner. The trial court awarded the MPC
himes in Sec. 171 of Corpo Law is only one among several ends which share to defendant (petitioner now) on the ground that the Articles of
building and loan associations are designed to promote and Sec. 181 Incorporation and By-laws of Manila Polo Club prohibit artificial persons,
authorizes the board of directors of the association to fix the premium to such as 383
be charged.
corporations, to be club members. Private respondent appealed to the
14) That the loans made by defendant for purposes other than building or Court of Appeals which in turn reversed the RTC decision.
acquiring homes have been extended in extremely large amounts and to
wealthy persons and large companies- WITHOUT MERIT Issue/s:
The question of whether the making of large loans constitutes a misuser of Whether the Court of Appeals in declaring AmCham to be the real owner
the franchise as would justify the court in depriving the association of its of the shares
corporate life is a matter confided to the discretion of the board of
directors. The law states no limit as to the size of the loans to be made by Ruling:
the association. Resort should be had to the legislature because it is not a The Supreme court ruled that the Court of Appeals did not err and
matter amenable to judicial control affirmed the appellate court’s decision. It held that there is a trust
relationship that existed between the parties. Moreover, petitioner failed to
15) That when the franchise expires, supposing the corporation is not present evidence to support his allegation of being merely a debtor when
reorganized, upon final liquidation of the corporation, a reserve fund may the private respondent paid the purchase price of the MPC share.
exist which is out of all proportion to the requirements that may fall upon Applicable here is the rule that a trust arises in favor of one who pays the
it in the liquidation of the company-NO MERIT purchase money of property in the name of another, because of the
This matter may be left to the discretion of the board of directors or to presumption that he who pays for a thing intends a beneficial interest
legislative action if it should be deemed expedient to require the gradual therein for himself. Although petitioner initiated the acquisition of the
suppression of reserve funds as the time for dissolution approaches. It is share, evidence on record shows that private respondent acquired said
share with its funds. Petitioner did not pay for said share, although he later
wanted to, but according to his own terms, particularly the price. The remove any employee from his employment by the simple expediency of
quitclaim executed by private respondent does not clearly show the intent amending its by-laws and providing that his/her position shall cease to
to include therein the ownership over the MPC share. Private respondent exist upon the occurrence of a specified event.
even asserts that at the time the Release and Quitclaim was executed on If private respondent wanted to make the petitioner's position co-
September 29, 1989, the ownership of the MPC share was not terminus with that of the Board of Directors, then the amendment
controversial nor contested. Settled is the rule that a waiver to be valid and must be effective after petitioner's stay with the private respondent,
effective must, in the first place, be couched in clear and unequivocal not during his term.
terms which leave no doubt as to the intention of a party to give up a right Obviously, the measure taken by the private respondent in amending
or benefit which legally pertains to him. A waiver may not be attributed to its by-laws is nothing but a devious, but crude, attempt to circumvent
a person when the terms thereof do not explicitly and clearly evidence an petitioner's right to security of tenure as a regular employee
intent to abandon a right vested in such person. If we apply the standard guaranteed under the Labor Code.
rule that waiver must be cast in clear and unequivocal terms, then clearly
the general terms of the cited release and quitclaim indicates merely a WHEREFORE, in view of the foregoing, the instant petition is
clearance from general accountability, not specifically a waiver of GRANTED. The NLRC decision dated June 15, 1995 is hereby
AmCham's beneficial ownership of the disputed shares. REVERSED and SET ASIDE.
Salafranca v. Philamlife (Pamplona Village Homeowners Assoc)
ENRIQUE SALAFRANCA, petitioner,
vs.
PHILAMLIFE (PAMPLONA) VILLAGE HOMEOWNERS GRACE CHRISTIAN HIGH SCHOOL, petitioner,vs. THE COURT
ASSOCIATION, INC., BONIFACIO DAZO and THE SECOND OF APPEALS, GRACE VILLAGE ASSOCIATION, INC.,
DIVISION, NATIONAL LABOR RELATIONS COMMISSION ALEJANDRO G. BELTRAN, and ERNESTO L. GO, respondents.
(NLRC), respondents.
G.R. No. 108905 October 23, 1997
ROMERO, J.:
MENDOZA, J.:
DOCTRINE: The right to amend the by-laws lies solely in the discretion
of the employer, this being in the exercise of management prerogative or Petitioner Grace Christian High School is an educational institution
business judgment. However this right, extensive as it may be, cannot located at the Grace Village in Quezon City, while Private respondent
impair the obligation of existing contracts or rights. Grace Village Association, Inc. ["Association'] is an organization of lot
and/or building owners, lessees and residents at Grace Village.
FACTS:
Petitioner Enrique Salafranca started working with the private The original 1968 by-laws provide that the Board of Directors, composed
respondent Philamlife Village Homeowners Association on May 1, of eleven (11) members, shall serve for one (1) year until their successors
1981 as administrative officer for a period of six months. are duly elected and have qualified.
From this date until December 31, 1983, petitioner was reappointed
to his position three more times. Petitioner was generally responsible for On 20 December 1975, a committee of the board of directors prepared a
the management of the village's day to day activities. After petitioner's draft of an amendment to the
term of employment expired on December 31, 1983, he still continued to by-laws which provides that "GRACE CHRISTIAN HIGH SCHOOL
work in the same capacity, albeit, without the benefit of a renewed representative is a permanent
contract. Director of the ASSOCIATION."
Sometime in 1987, private respondent decided to amend its by-laws.
Included therein was a provision regarding officers, specifically, the However, this draft was never presented to the general membership for
position of administrative officer under which said officer shall hold approval. Nevertheless, from 1975 to 1990, petitioner was given a
office at the pleasure of the Board of Directors. permanent seat in the board of directors of the association.
In view of this development, private respondent, informed the
petitioner that his term of office shall be co-terminus with the Board On 13 February 1990, the association's committee on election sought to
of Directors which appointed him to his position. Furthermore, until change the by-laws and informed the Petitioner's school principal "the
he submits a medical certificate showing his state of health, his proposal to make the Grace Christian High School representative as a
employment shall be on a month-to-month basis. Oddly, permanent director of the association, although previously tolerated in the
notwithstanding the failure of herein petitioner to submit his medical past elections should be reexamined."
certificate, he continued working until his termination in December 1992.
Claiming that his services had been unlawfully and unceremoniously Following this advice, notices were sent to the members of the association
dispensed with, petitioner filed a complaint for illegal dismissal with that the provision on election of directors of the 1968 by-laws of the
money claims and for damages. association would be observed. Petitioner requested the chairman of the
election committee to change the notice to honor the 1975 by-laws
Labor Arbiter: Ordering private respondent to pay the petitioner the provision, but was denied.
amount of P257,833.33 representing his backwages, separation pay and
13th month pay. The school then brought suit for mandamus in the Home Insurance and
Said Amendment would not be applicable to the case of complainant who Guaranty Corporation (HIGC) to compel the board of directors to
had become a regular employee long time before the Amendment took recognize its right to a permanent seat in the board.
place. Should be applied prospectively and not retroactively.
Meanwhile, the opinion of the SEC was sought by the association, and
NLRC: Reversed the decision of the L.A. Reducing petitioner's SEC rendered an opinion to the effect that the practice of allowing
monetary award to only one-half (1/2) month pay for every year of service unelected members in the board was contrary to the existing by-laws of
representing his retirement pay. the association and to §92 of the Corporation Code (B.P. Blg. 68). This
That the Association's Rules and Regulations were amended to put to rest was adopted by the association in its Answer in the mandamus filed with
the tenural (sic) limit of the office of the Administrative Officer in the HIGC.
accordance with its earlier intention, that it is co-terminus with that of the
members of the Board of Directors. The HIGC hearing officer ruled in favor of the association, which decision
was affirmed by the HIGC Appeals Board and the Court of Appeals.
ISSUE: WON petitioner’s dismissal is valid, on the ground that the latter's
position is coterminus with that of the Village's Board of Directors, as Issue: W/N the 1975 provision giving the petitioner a permanent board
provided for in its amended by-laws. seat was valid.

HELD: NO Ruling: No.


Admittedly, the right to amend the by-laws lies solely in the discretion
of the employer, this being in the exercise of management prerogative Section 23 of the Corporation Code (and its predecessor Section 28 and 29
or business judgment. However this right, extensive as it may be, of the Corporation Law) leaves no room for doubt that the Board of
cannot impair the obligation of existing contracts or rights. Directors of a Corporation must be elected from among the stockholders
Prescinding from these premises, private respondent's insistence that it can or members.
legally dismiss petitioner on the ground that his tenure has expired is
untenable. To reiterate, petitioner, being a regular employee, is entitled There may be corporations in which there are unelected members in the
to security of tenure, hence, his services may only be terminated for board but it is clear that in these instances, the unelected members sit as ex
causes provided by law. officio members, i.e., by virtue of and for as long as they hold a particular
A contrary interpretation would not find justification in the laws or the office (e.g. whoever is the Archbishop of Manila is considered a member
Constitution. If we were to rule otherwise, it would enable an employer to of the board of Cardinal Santos Memorial Hospital, Inc.)
But in the case of petitioner, there is no reason at all for its representative controversies. An intracorporate controversy has been defined as one
to be given a seat in the board. Nor does petitioner claim a right to such which arises between a stockholder and the corporation. There is no
seat by virtue of an office held. In fact it was not given such seat in the distinction, qualification, nor any exception whatsoever (Rivera vs.
beginning. It was only in 1975 that a proposed amendment to the by-laws Florendo, 144 SCRA 643 [1986]). The case at bar involves shares of
sought to give it one. stock, their registration, cancellation and issuances thereof by petitioner
Rural Bank of Salinas. It is therefore within the power of respondent SEC
Since the provision in question is contrary to law, the fact that it has gone to adjudicate.
unchallenged for fifteen years cannot forestall a later challenge to its Respondent SEC correctly ruled in favor of the registering of the shares of
validity. Neither can it attain validity through acquiescence because, if it is stock in question in private respondent's names. Such ruling finds support
contrary to law, it is beyond the power of the members of the association under Section 63 of the Corporation Code, to wit:
to waive its invalidity. Sec. 63. . . . Shares of stock so issued are personal property and may be
transferred by delivery of the certificate or certificates indorsed by the
It is more accurate to say that the members merely tolerated petitioner's owner or his attorney-in-fact or other person legally authorized to make
representative and tolerance cannot be considered ratification. the transfer. No transfer, however, shall be valid, except as between the
parties, until the transfer is recorded in the books of the corporation . . .
Nor can petitioner claim a vested right to sit in the board on the basis of In the case of Fleisher vs. Botica Nolasco, 47 Phil. 583, the Court
"practice." Practice, no matter how long continued, cannot give rise to any interpreted Sec. 63 in his wise:
vested right if it is contrary to law. Said Section (Sec. 35 of Act 1459 [now Sec. 63 of the Corporation Code])
contemplates no restriction as to whom the stocks may be transferred. It
SALINAS INC versus CA does not suggest that any discrimination may be created by the corporation
in favor of, or against a certain purchaser. The owner of shares, as owner
FACTS: On June 10, 1979, Clemente G. Guerrero, President of the Rural of personal property, is at liberty, under said section to dispose them in
Bank of Salinas, Inc., executed a Special Power of Attorney in favor of his favor of whomever he pleases, without limitation in this respect, than the
wife, private respondent Melania Guerrero, giving and granting the latter general provisions of law. . . .
full power and authority to sell or otherwise dispose of and/or mortgage The only limitation imposed by Section 63 of the Corporation Code is
473 shares of stock of the Bank registered in his name (represented by the when the corporation holds any unpaid claim against the shares intended
Bank's stock certificates nos. 26, 49 and 65), to execute the proper to be transferred, which is absent here.
documents therefor, and to receive and sign receipts for the dispositions. A corporation, either by its board, its by-laws, or the act of its officers,
Pursuant to said SPA, private respondent Melania Guerrero, as Attorney- cannot create restrictions in stock transfers, because:
in-Fact, executed a Deed of Assignment for 472 shares out of the 473 . . . Restrictions in the traffic of stock must have their source in legislative
shares, in favor of private respondents Luz Andico (457 shares), enactment, as the corporation itself cannot create such impediment. By-
Wilhelmina Rosales (10 shares) and Francisco Guerrero, Jr. (5 shares). laws are intended merely for the protection of the corporation, and
Almost four months later, or two (2) days before the death of Clemente prescribe regulation, not restriction; they are always subject to the charter
Guerrero on June 24, 1980, private respondent Melania Guerrero, pursuant of the corporation. The corporation, in the absence of such power, cannot
to the same SPA, executed a Deed of Assignment for the remaining one (1) ordinarily inquire into or pass upon the legality of the transactions by
share of stock in favor of private respondent Francisco Guerrero, Sr. which its stock passes from one person to another, nor can it question the
Subsequently, private respondent Melania Guerrero presented to petitioner consideration upon which a sale is based. . . . (Tomson on Corporation
Rural Bank of Salinas the two (2) Deeds of Assignment for registration Sec. 4137, citedin Fleisher vs. Nolasco, Supra).
with a request for the transfer in the Bank's stock and transfer book of the The right of a transferee/assignee to have stocks transferred to his name is
473 shares of stock so assigned, the cancellation of stock certificates in the an inherent right flowing from his ownership of the stocks. Thus:
name of Clemente G. Guerrero, and the issuance of new stock certificates Whenever a corporation refuses to transfer and register stock in cases like
covering the transferred shares of stocks in the name of the new owners the present, mandamuswill lie to compel the officers of the corporation to
thereof. However, petitioner Bank denied the request of respondent. transfer said stock in the books of the corporation" (26, Cyc. 347, Hyer vs.
Private respondent filed with the Securities and Exchange Commission" Bryan, 19 Phil. 138; Fleisher vs. Botica Nolasco, 47 Phil. 583, 594).
(SEC) an action for mandamus against petitioners Rural Bank of Salinas, The corporation's obligation to register is ministerial.
its President and Corporate Secretary. In transferring stock, the secretary of a corporation acts in purely
Petitioners filed their Answer with counterclaim on December 19, 1980 ministerial capacity, and does not try to decide the question of ownership.
alleging the upon the death of Clemente G. Guerrero, his 473 shares of (Fletcher, Sec. 5528, page 434).
stock became the property of his estate, and his property and that of his The duty of the corporation to transfer is a ministerial one and if it refuses
widow should first be settled and liquidated in accordance with law before to make such transaction without good cause, it may be compelled to do so
any distribution can be effected so that petitioners may not be a party to by mandamus. (See. 5518, 12 Fletcher 394)
any scheme to evade payment of estate or inheritance tax and in order to For the petitioner Rural Bank of Salinas to refuse registration of the
avoid liability to any third persons or creditors of the late Clemente G. transferred shares in its stock and transfer book, which duty is ministerial
Guerrero. on its part, is to render nugatory and ineffectual the spirit and intent of
On January 29, 1981, a motion for intervention was filed by Maripol Section 63 of the Corporation Code. Thus, respondent Court of Appeals
Guerrero, a legally adopted daughter of the late Clemente G. Guerrero and did not err in upholding the Decision of respondent SEC affirming the
private respondent Melania Guerrero, who stated therein that on Decision of its Hearing Officer directing the registration of the 473 shares
November 26, 1980 (almost two weeks before the filing of the petition in the stock and transfer book in the names of private respondents. At all
for Mandamus) a Petition for the administration of the estate of the late events, the registration is without prejudice to the proceedings in court to
Clemente G. Guerrero had been filed with the Regional Trial Court, Pasig. determine the validity of the Deeds of Assignment of the shares of stock in
Maripol further claimed that the Deeds of Assignment for the subject question.
shares of stock are fictitious and antedated; that said conveyances are WHEREFORE, the petition is DISMISSED for lack of merit.
donations since the considerations therefor are below the book value of the
shares, the assignees/private respondents being close relatives of private SAN MIGUEL CORPORATION (MANDAUE PACKAGING
respondent Melania Guerrero; and that the transfer of the shares in PRODUCTS PLANTS), petitioner, vs. MANDAUE PACKING
question to assignees/private respondents, other than private respondent PRODUCTS PLANTS-SAN PACKAGING PRODUCTS –SAN
Melania Guerrero, would deprive her (Maripol Guerrero) of her rightful MIGUEL CORPORATION MONTHLIES RANK-AND-FILE UNION –
share in the inheritance. The SEC hearing officer denied the Motion for FFW (MPPP-SMPP-SMAMRFU-FFW), respondent.
Intervention for lack of merit. On appeal, the SEC En Banc affirmed the Facts:
decision of the hearing officer. On 15 June 1998, respondent, identifying itself as an affiliate of
Federation of Free Workers (FFW), filed a petition for certification
On December 10, 1984, the SEC Hearing Officer rendered a Decision election with the DOLE Regional Office No. VII. In the petition,
granting the writ of Mandamus prayed for by the private respondents and respondent stated that it sought to be certified and to represent the
directing petitioners to cancel stock certificates nos. 26, 49 and 65 of the permanent rank-and-file monthly paid employees of the petitioner. The
Bank, all in the name of Clemente G. Guerrero, and to issue new following documents were attached to the petition: (1) a Charter
certificates in the names of private respondents, except Melania Guerrero. Certificate issued by FFW on 5 June 1998 certifying that respondent as of
On appeal, the SEC En Banc affirmed the decision of the Hearing Officer. that date was duly certified as a local or chapter of FFW; (2) a copy of the
Petitioner filed a petition for review with the Court of Appeals but said constitution of respondent prepared by its Secretary and attested by its
Court likewise affirmed the decision of the SEC. President; (3) a list of respondent’s officers and their respective addresses;
ISSUE: Whether or not the respondent court erred in sustaining the (4) a certification signifying that respondent had just been organized and
Securities and Exchange Commission when it compelled no amount had yet been collected from its members, signed by
by Mandamus the Rural Bank of Salinas to register in its stock and respondent’s treasurer and (5) a list of all the rank-and-file monthly paid
transfer book the transfer of 473 shares of stock to private respondents employees of the Mandaue Packaging Products Plants and Mandaue Glass
HELD: We rule in favor of the respondents. Plant.
Section 5 (b) of P.D. No. 902-A grants to the SEC the original and On 27 July 1998, petitioner filed a motion to dismiss the petition for
exclusive jurisdiction to hear and decide cases involving intracorporate certification election on the sole ground that herein respondent is not listed
or included in the roster of legitimate labor organizations based on the corporation, which was avowed because the questioned amendment gave
certification issued by the Officer-In-Charge, Regional Director of the the Board itself the prerogative of determining whether they or other
DOLE Regional Office No. VII, Atty. Jesus B. Gabor, on 24 July 1998. persons are engaged in competitive or antagonistic business; that the
On 20 August 1998, petitioner filed a petition to cancel the union portion of the amended by-laws which states that in determining whether
registration of respondent. However, this petition was denied, and such or not a person is engaged in competitive business, the Board may
denial was subsequently affirmed by the Court of Appeals in a decision consider such factors as business and family relationship, is unreasonable
that has since become final. and oppressive and, therefore, void; and that the portion of the amended
Respondent promptly appealed the 15 September 1998 Order to the by-laws which requires that "all nominations for election of directors shall
DOLE. On 22 February 1999, DOLE Undersecretary rendered a Decision be submitted in writing to the Board of Directors at least five (5) working
reversing the Order. days before the date of the Annual Meeting" is likewise unreasonable and
Issue: oppressive. It was, therefore, prayed that the amended by-laws be declared
WON the Union acquired legal personality. null and void and the certificate of filing thereof be cancelled, and that
Held: Soriano, et. al. be made to pay damages, in specified amounts, to
Yes. Gokongwei. On 28 October 1976, in connection with the same case,
Section 3, Rule VI of Department Order No. 9 provides when the Gokongwei filed with the Securities and Exchange Commission an
local/chapter acquires legal personality. "Urgent Motion for Production and Inspection of Documents", alleging
Section 3. Acquisition of legal personality by local chapter. – A that the Secretary of the corporation refused to allow him to inspect its
local/chapter constituted in accordance with Section 1 of this Rule shall records despite request made by Gokongwei for production of certain
acquire legal personality from the date of filing of the complete documents documents enumerated in the request, and that the corporation had been
enumerated therein. Upon compliance with all the documentary attempting to suppress information from its stockholders despite a
requirements, the Regional Office or Bureau shall issue in favor of the negative reply by the SEC to its query regarding their authority to do so.
local/chapter a certificate indicating that it is included in the roster of
legitimate labor organizations. The motion was opposed by Soriano, et. al. The Corporation, Soriano, et.
It is evident based on this rule that the local/chapter acquires legal al. filed their answer, and their opposition to the petition, respectively.
personality from the date of the filing of the complete documentary Meanwhile, on 10 December 1976, while the petition was yet to be heard,
requirements, and not from the issuance of a certification to such effect by the corporation issued a notice of special stockholders' meeting for the
the Regional Office or Bureau. On the other hand, a labor organization is purpose of "ratification and confirmation of the amendment to the By-
deemed to have acquired legal personality only on the date of issuance of laws", setting such meeting for 10 February 1977. This prompted
its certificate of registration, which takes place only after the Bureau of Gokongwei to ask the SEC for a summary judgment insofar as the first
Labor Relations or its Regional Offices has undertaken an evaluation cause of action is concerned, for the alleged reason that by calling a
process lasting up until thirty (30) days, within which period it approves or special stockholders' meeting for the aforesaid purpose, Soriano, et. al.
denies the application. In contrast, no such period of evaluation is admitted the invalidity of the amendments of 18 September 1976. The
provided in Department Order No. 9 for the application of a local/chapter, motion for summary judgment was opposed by Soriano, et. al. Pending
and more importantly, under it such local/chapter is deemed to acquire action on the motion, Gokongwei filed an "Urgent Motion for the Issuance
legal personality “from the date of filing” of the documents enumerated of a Temporary Restraining Order", praying that pending the
under Section 1, Rule VI, Book V. determination of Gokongwei's application for the issuance of a
preliminary injunction and or Gokongwei's motion for summary judgment,
Gokongwei vs. Securities and Exchange Commission a temporary restraining order be issued, restraining Soriano, et. al. from
[GR L-45911, 11 April 1979] holding the special stockholders' meeting as scheduled. This motion was
duly opposed by Soriano, et. al. On 10 February 1977, Cremation issued
Facts: an order denying the motion for issuance of temporary restraining order.
After receipt of the order of denial, Soriano, et. al. conducted the special
On 22 October 1976, John Gokongwei Jr., as stockholder of San Miguel stockholders' meeting wherein the amendments to the by-laws were
Corporation, filed with the Securities and Exchange Commission (SEC) a ratified. On 14 February 1977, Gokongwei filed a consolidated motion for
petition for "declaration of nullity of amended by-laws, cancellation of contempt and for nullification of the special stockholders' meeting. A
certificate of filing of amended by-laws, injunction and damages with motion for reconsideration of the order denying Gokongwei's motion for
prayer for a preliminary injunction" against the majority of the members summary judgment was filed by Gokongwei before the SEC on 10 March
of the Board of Directors and San Miguel Corporation as an unwilling 1977.
petitioner. As a first cause of action, Gokongwei alleged that on 18
September 1976, Andres Soriano, Jr., Jose M. Soriano, Enrique Zobel, Gokongwei alleged that, having discovered that the corporation has been
Antonio Roxas, Emeterio Buñao, Walthrode B. Conde, Miguel Ortigas, investing corporate funds in other corporations and businesses outside of
and Antonio Prieto amended by bylaws of the corporation, basing their the primary purpose clause of the corporation, in violation of section 17-
authority to do so on a resolution of the stockholders adopted on 13 March 1/2 of the Corporation Law, he filed with SEC, on 20 January 1977, a
1961, when the outstanding capital stock of the corporation was only petition seeking to have Andres M. Soriano, Jr. and Jose M. Soriano, as
P70,139.740.00, divided into 5,513,974 common shares at P10.00 per well as the corporation declared guilty of such violation, and ordered to
share and 150,000 preferred shares at P100.00 per share. At the time of the account for such investments and to answer for damages. On 4 February
amendment, the outstanding and paid up shares totalled 30,127,043, with a 1977, motions to dismiss were filed by Soriano, et. al., to which a
total par value of P301,270,430.00. consolidated motion to strike and to declare Soriano, et. al. in default and
an opposition ad abundantiorem cautelam were filed by Gokongwei.
It was contended that according to section 22 of the Corporation Law and Despite the fact that said motions were filed as early as 4 February 1977,
Article VIII of the by-laws of the corporation, the power to amend, the Commission acted thereon only on 25 April 1977, when it denied
modify, repeal or adopt new by-laws may be delegated to the Board of Soriano, et. al.'s motions to dismiss and gave them two (2) days within
Directors only by the affirmative vote of stockholders representing not less which to file their answer, and set the case for hearing on April 29 and
than 2/3 of the subscribed and paid up capital stock of the corporation, May 3, 1977. Soriano, et. al. issued notices of the annual stockholders'
which 2/3 should have been computed on the basis of the capitalization at meeting, including in the Agenda thereof, the "reaffirmation of the
the time of the amendment. authorization to the Board of Directors by the stockholders at the meeting
Since the amendment was based on the 1961 authorization, Gokongwei on 20 March 1972 to invest corporate funds in other companies or
contended that the Board acted without authority and in usurpation of the businesses or for purposes other than the main purpose for which the
power of the stockholders. As a second cause of action, it was alleged that Corporation has been organized, and ratification of the investments
the authority granted in 1961 had already been exercised in 1962 and thereafter made pursuant thereto."
1963, after which the authority of the Board ceased to exist. As a third
cause of action, Gokongwei averred that the membership of the Board of By reason of the foregoing, on 28 April 1977, Gokongwei filed with the
Directors had changed since the authority was given in 1961, there being 6 SEC an urgent motion for the issuance of a writ of preliminary injunction
new directors. As a fourth cause of action, it was claimed that prior to the to restrain Soriano, et. al. from taking up Item 6 of the Agenda at the
questioned amendment, Gokogwei had all the qualifications to be a annual stockholders' meeting, requesting that the same be set for hearing
director of the corporation, being a substantial stockholder thereof; that as on 3 May 1977, the date set for the second hearing of the case on the
a stockholder, Gokongwei had acquired rights inherent in stock merits. The SEC, however, cancelled the dates of hearing originally
ownership, such as the rights to vote and to be voted upon in the election scheduled and reset the same to May 16 and 17, 1977, or after the
of directors; and that in amending the by-laws, Soriano, et. al. purposely scheduled annual stockholders' meeting. For the purpose of urging the
provided for Gokongwei's disqualification and deprived him of his vested Commission to act, Gokongwei filed an urgent manifestation on 3 May
right as afore-mentioned, hence the amended by-laws are null and void. 1977, but this notwithstanding, no action has been taken up to the date of
As additional causes of action, it was alleged that corporations have no the filing of the instant petition.
inherent power to disqualify a stockholder from being elected as a director
and, therefore, the questioned act is ultra vires and void; that Andres M. Gokongwei filed a petition for petition for certiorari, mandamus and
Soriano, Jr. and/or Jose M. Soriano, while representing other corporations, injunction, with prayer for issuance of writ of preliminary injunction, with
entered into contracts (specifically a management contract) with the
the Supreme Court, alleging that there appears a deliberate and concerted meticulous he is to satisfy technical requirements. For that
inability on the part of the SEC to act. power is at all times subject to the equitable limitation that it
may not be exercised for the aggrandizement, preference, or
Issue: advantage of the fiduciary to the exclusion or detriment of the
1. Whether the corporation has the power to provide for the cestuis. The doctrine of "corporate opportunity" is precisely a
(additional) qualifications of its directors. recognition by the courts that the fiduciary standards could not
2. Whether the disqualification of a competitor from being elected be upheld where the fiduciary was acting for two entities with
to the Board of Directors is a reasonable exercise of corporate competing interests. This doctrine rests fundamentally on the
authority. unfairness, in particular circumstances, of an officer or director
3. Whether the SEC gravely abused its discretion in denying taking advantage of an opportunity for his own personal profit
Gokongwei's request for an examination of the records of San when the interest of the corporation justly calls for protection. It
Miguel International, Inc., a fully owned subsidiary of San is not denied that a member of the Board of Directors of the San
Miguel Corporation. Miguel Corporation has access to sensitive and highly
4. Whether the SEC gravely abused its discretion in allowing the confidential information, such as: (a) marketing strategies and
stockholders of San Miguel Corporation to ratify the investment pricing structure; (b) budget for expansion and diversification;
of corporate funds in a foreign corporation. (c) research and development; and (d) sources of funding,
Held: availability of personnel, proposals of mergers or tie-ups with
other firms. It is obviously to prevent the creation of an
1. It is recognized by all authorities that "every corporation has the opportunity for an officer or director of San Miguel Corporation,
inherent power to adopt by-laws 'for its internal government, who is also the officer or owner of a competing corporation,
and to regulate the conduct and prescribe the rights and duties of from taking advantage of the information which he acquires as
its members towards itself and among themselves in reference to director to promote his individual or corporate interests to the
the management of its affairs.'" In this jurisdiction under section prejudice of San Miguel Corporation and its stockholders, that
21 of the Corporation Law, a corporation may prescribe in its the questioned amendment of the by-laws was made. Certainly,
by-laws "the qualifications, duties and compensation of where two corporations are competitive in a substantial sense, it
directors, officers and employees." This must necessarily refer would seem improbable, if not impossible, for the director, if he
to a qualification in addition to that specified by section 30 of were to discharge effectively his duty, to satisfy his loyalty to
the Corporation Law, which provides that "every director must both corporations and place the performance of his corporation
own in his right at least one share of the capital stock of the duties above his personal concerns. The offer and assurance of
stock corporation of which he is a director." Any person "who Gokongwei that to avoid any possibility of his taking unfair
buys stock in a corporation does so with the knowledge that its advantage of his position as director of San Miguel Corporation,
affairs are dominated by a majority of the stockholders and that he would absent himself from meetings at which confidential
he impliedly contracts that the will of the majority shall govern matters would be discussed, would not detract from the validity
in all matters within the limits of the act of incorporation and and reasonableness of the by-laws involved. Apart from the
lawfully enacted by-laws and not forbidden by law." To this impractical results that would ensue from such arrangement, it
extent, therefore, the stockholder may be considered to have would be inconsistent with Gokongwei's primary motive in
"parted with his personal right or privilege to regulate the running for board membership — which is to protect his
disposition of his property which he has invested in the capital investments in San Miguel Corporation. More important, such a
stock of the corporation, and surrendered it to the will of the proposed norm of conduct would be against all accepted
majority of his fellow incorporators. It can not therefore be principles underlying a director's duty of fidelity to the
justly said that the contract, express or implied, between the corporation, for the policy of the law is to encourage and enforce
corporation and the stockholders is infringed by any act of the responsible corporate management.
former which is authorized by a majority." Pursuant to section
18 of the Corporation Law, any corporation may amend its 3. Pursuant to the second paragraph of section 51 of the
articles of incorporation by a vote or written assent of the Corporation Law, "(t)he record of all business transactions of
stockholders representing at least two-thirds of the subscribed the corporation and minutes of any meeting shall be open to the
capital stock of the corporation. If the amendment changes, inspection of any director, member or stockholder of the
diminishes or restricts the rights of the existing shareholders, corporation at reasonable hours." The stockholder's right of
then the dissenting minority has only one right, viz.: "to object inspection of the corporation's books and records is based upon
thereto in writing and demand payment for his share." Under their ownership of the assets and property of the corporation. It
section 22 of the same law, the owners of the majority of the is, therefore, an incident of ownership of the corporate property,
subscribed capital stock may amend or repeal any by-law or whether this ownership or interest be termed an equitable
adopt new by-laws. It cannot be said, therefore, that Gokongwei ownership, a beneficial ownership, or a quasi-ownership. This
has a vested right to be elected director, in the face of the fact right is predicated upon the necessity of self-protection. It is
that the law at the time such right as stockholder was acquired generally held by majority of the courts that where the right is
contained the prescription that the corporate charter and the by- granted by statute to the stockholder, it is given to him as such
law shall be subject to amendment, alteration and modification. and must be exercised by him with respect to his interest as a
stockholder and for some purpose germane thereto or in the
2. Although in the strict and technical sense, directors of a private interest of the corporation. In other words, the inspection has to
corporation are not regarded as trustees, there cannot be any be germane to the petitioner's interest as a stockholder, and has
doubt that their character is that of a fiduciary insofar as the to be proper and lawful in character and not inimical to the
corporation and the stockholders as a body are concerned. As interest of the corporation. The "general rule that stockholders
agents entrusted with the management of the corporation for the are entitled to full information as to the management of the
collective benefit of the stockholders, "they occupy a fiduciary corporation and the manner of expenditure of its funds, and to
relation, and in this sense the relation is one of trust." "The inspection to obtain such information, especially where it
ordinary trust relationship of directors of a corporation and appears that the company is being mismanaged or that it is being
stockholders is not a matter of statutory or technical law. It managed for the personal benefit of officers or directors or
springs from the fact that directors have the control and certain of the stockholders to the exclusion of others." While the
guidance of corporate affairs and property and hence of the right of a stockholder to examine the books and records of a
property interests of the stockholders. Equity recognizes that corporation for a lawful purpose is a matter of law, the right of
stockholders are the proprietors of the corporate interests and are such stockholder to examine the books and records of a wholly-
ultimately the only beneficiaries thereof." A director is a owned subsidiary of the corporation in which he is a stockholder
fiduciary. Their powers are powers in trust. He who is in such is a different thing. Stockholders are entitled to inspect the
fiduciary position cannot serve himself first and his cestuis books and records of a corporation in order to investigate the
second. He cannot manipulate the affairs of his corporation to conduct of the management, determine the financial condition of
their detriment and in disregard of the standards of common the corporation, and generally take an account of the
decency. He cannot by the intervention of a corporate entity stewardship of the officers and directors. herein, considering that
violate the ancient precept against serving two masters. He the foreign subsidiary is wholly owned by San Miguel
cannot utilize his inside information and strategic position for Corporation and, therefore, under Its control, it would be more
his own preferment. He cannot violate rules of fair play by doing in accord with equity, good faith and fair dealing to construe the
indirectly through the corporation what he could not do so statutory right of petitioner as stockholder to inspect the books
directly. He cannot violate rules of fair play by doing indirectly and records of the corporation as extending to books and records
through the corporation what he could not do so directly. He of such wholly owned subsidiary which are in the corporation's
cannot use his power for his personal advantage and to the possession and control.
detriment of the stockholders and creditors no matter how
absolute in terms that power may be and no matter how
4. Section 17-1/2 of the Corporation Law allows a corporation to delayed printing, failure to correct errors, neglect and refusal to
"invest its funds in any other corporation or business or for any print.
purpose other than the main purpose for which it was organized"  At the trial of the case the plaintiffs presented in evidence
provided that its Board of Directors has been so authorized by Exhibit A which purports to be a contract between Chen and
the affirmative vote of stockholders holding shares entitling the plaintiffs and which provides that in the event the plaintiffs
them to exercise at least two-thirds of the voting power. If the should be discharged without cause before the expirations of the
investment is made in pursuance of the corporate purpose, it term of three years from January 1, 1920, they would be given
does not need the approval of the stockholders. It is only when full pay for the unexpired portion of the term "even if the said
the purchase of shares is done solely for investment and not to paper has to fall into bankruptcy." The contract is signed by the
accomplish the purpose of its incorporation that the vote of plaintiffs and also bears the signature "C. C. Chen, manager
approval of the stockholders holding shares entitling them to of Kong Li Po." The authenticity of the latter signature is
exercise at least two-thirds of the voting power is necessary. As questioned by the defendant, but the court below found that the
stated by the corporation, the purchase of beer manufacturing evidence upon this point preponderate in favor of the plaintiffs
facilities by SMC was an investment in the same business stated and there appears to be no sufficient reason to disturb this
as its main purpose in its Articles of Incorporation, which is to finding.
manufacture and market beer. It appears that the original  Trial Court found in favour of petitioners saying contract had
investment was made in 1947-1948, when SMC, then San been impliedly ratified by the defendant. Kong Li Po appeals
Miguel Brewery, Inc., purchased a beer brewery in Hongkong saying that contract was not signed by C.C. Chen and in any
(Hongkong Brewery & Distillery, Ltd.) for the manufacture and event C. C. Chen had no power or authority to bind the
marketing of San Miguel beer thereat. Restructuring of the defendant corporation by such contract; and that there was no
investment was made in 1970-1971 thru the organization of SMI ratification of the contract by the corporation.
in Bermuda as a tax free reorganization. Assuming arguendo ISSUE: WON Chen [the general manger] had the power to bind the
that the Board of Directors of SMC had no authority to make the corporation by a contract of the character indicated – NO. Only valid by a
assailed investment, there is no question that a corporation, like reasonable and usual contract of employment
an individual, may ratify and thereby render binding upon it the RATIO:
originally unauthorized acts of its officers or other agents. This Procedure/Evidence
is true because the questioned investment is neither contrary to The contract supposedly attached with the complaint was a translation. As
law, morals, public order or public policy. It is a corporate this translation may be considered a copy and as the defendant failed to
transaction or contract which is within the corporate powers, but deny its authenticity under oath, it will perhaps be said that under
which is defective from a purported failure to observe in its section 103 of the Code of Civil Procedure the omission to so deny it
execution the requirement of the law that the investment must be constitutes an admission of the genuineness and due execution of the
authorized by the affirmative vote of the stockholders holding document as well as of the agent's authority to bind the defendant.
two-thirds of the voting power. This requirement is for the (Merchant vs. International Banking Corporation, 6 Phil., 314.) However
benefit of the stockholders. The stockholders for whose benefit the court ruled that this case was an exception since evidence was
the requirement was enacted may, therefore, ratify the presented by plaintiff on the execution of the document. [Plaintiff waived]
investment and its ratification by said stockholders obliterates Chen’s Authority
any defect which it may have had at the outset. Besides, the It is conceded that Chen had no express authority to do so, but the
investment was for the purchase of beer manufacturing and evidence is conclusive that he, at the time the contract was entered into,
marketing facilities which is apparently relevant to the corporate was in effect the general business manager of the newspaper Kong Li
purpose. The mere fact that the corporation submitted the Po and that he, as such, had charge of the printing of the paper, and the
assailed investment to the stockholders for ratification at the plaintiff maintain that he, as such general business manager, had implied
annual meeting of 10 May 1977 cannot be construed as an authority to employ them on the terms stated and that the defendant
admission that the corporation had committed an ultra vires act, corporation is bound by his action.
considering the common practice of corporations of periodically The general rule is that the power to bind a corporation by contract lies
submitting for the ratification of their stockholders the acts of with its board of directors or trustees, but this power may either expressly
their directors, officers and managers. or impliedly be delegated to other officers or agents of the corporation,
and it is well settled that except where the authority of employing servants
YU CHUCK, MACK YUENG, and DING MOON, plaintiffs- and agent is expressly vested in the board of directors or trustees, an
appellees, officer or agent who has general control and management of the
vs. corporation's business, or a specific part thereof, may bind the corporation
"KONG LI PO," defendant-appellant. by the employment of such agent and employees as are usual and
December 3, 1924, OSTRAND, J.: necessary in the conduct of such business. But the contracts of
FACTS: employment must be reasonable.
 The defendant is a domestic corporation organized in Chen, as general manager of the Kong Li Po, had implied authority to bind
accordance with the laws of the Philippine Islands and engaged the defendant corporation by a reasonable and usual contract of
in the publication of a Chinese newspaper styled Kong Li Po. Its employment with the plaintiffs, but we do not think that the contract here
articles of incorporation and by-laws are in the usual form and in question can be so considered. Not only is the term of employment
provide for a board of directors and for other officers among unusually long, but the conditions are otherwise so onerous to the
them a president whose duty it is to "sign all contracts and other defendant that the possibility of the corporation being thrown into
instruments of writing." No special provision is made for a insolvency thereby is expressly contemplated in the same contract.
business or general manager. Neither do we think that the contention that the corporation impliedly
 Sometime in 1919, C. C./T. C. Chen was appointed general ratified the contract is supported by the evidence. The contention is based
business manager of the newspaper. principally on the fact that Te Kim Hua, the president of the corporation
 Dec 1919, Chen entered into an agreement with the plaintiffs by for the year 1920, admitted on the witness stand that he saw the plaintiffs
which the latter bound themselves to do the necessary printing work as printers in the office of the newspaper. He denied, however, any
for the newspaper for the sum of P580 per month. knowledge of the existence of the contract and asserted that it was never
 Under this agreement the plaintiffs worked for the defendant presented neither to him nor to the board of directors. Before a contract
from January 1, 1920, until January 31, 1921, when they were can be ratified knowledge of its existence must, of course, be brought
discharged by the new manager, Tan Tian Hong, who had been home to the parties who have authority to ratify it or circumstances must
appointed in the meantime, C. C. Chen having left for China. be shown from which such knowledge may be presumed. No such
The letter of dismissal stated no special reasons for the discharge knowledge or circumstances have been shown here. That the president of
of the plaintiffs. the corporation saw the plaintiffs working in its office is of little
 The plaintiffs thereupon brought the present action alleging, significance; there were other printers working there at that time and as the
among other things, in the complaint that their contract of president had nothing to do with their employment, it was hardly to be
employment was for a term of three years from the first day of expected that be would inquire into the terms of their contracts. Moreover,
January, 1920; that in the case of their discharge by the a ratification by him would have been of no avail; in order to validate a
defendant without just cause before the expiration of the term of contract, a ratification by the board of directors was necessary. The
the contract, they were to receive full pay for the remaining fact that the president was required by the by-laws to sign the documents
portion of the term; that they had been so discharged without evidencing contracts of the corporation, does not mean that he had power
just cause and therefore asked judgment for damages in the sum to make the contracts.
of P20,880. In his decision his Honor, the learned judge of the court below appears to
 Included in its 5 special defenses , Kong Li Po states that C. C. have placed some weight on a notice inserted in the January 14th issue of
Chen, the person whose name appears to have been signed to the the Kong Li Po by T. C. Chen and which, in translation, reads as follows:
contract of employment was not authorized by the defendant to To Whom It May Concern: Announcement is hereby given that thereafter
execute such a contract in its behalf. Other defenses include all contracts, agreements and receipts are considered to be null and void
unless duly signed by T. C. Chen, General Manager of this paper. This is The payment of all interests, charges, penalties, reimbursements and other
signed by Chen. This notice led the plaintiffs to think that Chen had obligations owing by the MORTGAGOR and/or DEBTOR to the
authority to make the contract. It may further be observed that the notice MORTGAGEE whether direct or indirect, principal or secondary; absolute
confers no special powers, but is, in effect, only an assertion by Chen that or contingent as appearing in the accounts, books and records of the
he would recognize no contracts, agreements, and receipts not duty signed MORTGAGEE.
by him. It may be presumed that the contracts, agreements, and receipts The payment of all obligations of the MORTGAGOR and/or DEBTOR of
were such as were ordinarily made in the course of the business of whatever kind or nature whether such obligations have been contracted
managing the newspaper. There is no evidence to show that the notice was before, during, or after the constitution of [the] MORTGAGE.
ever brought to the attention of the officers of the defendant corporation. In case the MORTGAGOR and/or DEBTOR incurs subsequent
Held: The judgment appealed from is reversed and the defendant obligations of whatever kind or nature whether such obligations, as
corporation is absolved from the complaint. extension thereof, or as new loans or is given any other kind of
Concurring (Street) accommodations, the payment of said obligations, and/or accommodations
There must be a limit somewhat upon the authority of a manager with without the necessity of executing new agreements.
respect to the duration of contracts which he makes for the corporation, The faithful and strict performance and compliance by the MORTGAGOR
and my eye has fallen upon no decision in which contract for the period of and/or DEBTOR of all the terms and conditions of the MORTGAGE, the
three years, or longer, has been upheld on the bare fact that the contract credit agreements, promissory notes and other loan documents and
was made by a manager, though there are case in which contracts for the agreements evidencing the loan, overdrafts, credit lines and other credit
period of only one year have been sustained. accommodations granted to the MORTGAGOR and/or DEBTOR;
But no presumption of law can be indulged in that, because as person acts including all amendments thereon, such as but not limited to changes in
as such a manager, he has the power to bind his principal to contracts of an the interest rates, penalties, charges, or fees; acceleration of payments; and
extraordinary nature, and of such a character as would involved the the like.
corporation in enormous obligations and for long periods of time.
Dissenting (Malcolm) x x x x[4] (Emphasis, italics and underscoring supplied)
Defendant corporation held T. C. Chen out to the public as the business Prime Aggregates subsequently obtained several loans from IEB from
manager of the newspaper Kong Li Po and clothes him with apparent September 1997 until September 1998.[5]
authority to bind the corporation. The president of the corporation
admitted as much on the witness stand, while public announcement was Prime Aggregates failed to settle its outstanding obligation which stood at
made [Notice]. P90,267,854.96 and US$211,547.12[6] as of September 15, 2000, drawing
The action of the business manager was thus ratified by his superior IEB to file a petition for extra-judicial foreclosure of mortgage before the
officers and they are now in estoppel to deny such ratification. As held Regional Trial Court (RTC) of Cebu City.
in the case of Macke vs. Camps ([1907], 7 Phil., 553), one who clothes
another with apparent authority as his agent and holds him out to the Respondent Sheriff IV Arthur R. Cabigon (Cabigon) having issued on
public as such, cannot be permitted to deny the authority of such person to October 18, 2000 a Notice of Extra-Judicial Foreclosure and
act as his agent in good faith and in the honest belief that he is what he Sale[7]scheduled on November 28, 2000, petitioner filed a complaint[8] for
appears to be. Unless the contrary appears, the authority of an agent must Injunction with application for writ of preliminary injunction/temporary
be presumed to include all the necessary and usual means of carrying his restraining order before the Cebu City RTC, alleging that the real estate
agency into effect. mortgage was null and void because Amparo and Zosa were authorized to
Dealing with corporations the public at large is bound to rely to a large execute it to secure only one obligation of Prime Aggregates. Petitioner
extent upon outward appearances. If a man is found acting for a thus prayed
corporation with the external indicia of authority, any person, not having
notice of want of authority, may usually rely upon those appearances; and x x x that after due notice and hearing, judgment be rendered declaring the
if it be found that the directors had permitted the agent to exercise that real estate mortgage and its extrajudicial foreclosure sale as null and void
authority and thereby held him out as a person competent to bind the and that defendant bank be sentenced to pay plaintiff the sum of
corporation, or had acquiesced in a contract and retained the benefit P100,000.00 as attorney's fees and P100,000.00 as litigation expenses.
supposed to have been conferred by it, the corporation will be bound,
notwithstanding the actual authority may never have been granted. The In the meantime, it is most respectfully prayed that a writ of
public is not supposed nor required to know the transactions which happen preliminary injunction/TRO be issued enjoining the extrajudicial
around the table where the corporate board of directors or the stockholders foreclosure sale of plaintiff's properties scheduled on November 28, 2000
are from time to time convoked. Whether as particular officer actually or December 5, 2000.
possesses the authority which he assumes to exercise is frequently known
to very few, and the proof of it usually is not readily accessible to the . . . that after trial, the writ of preliminary injunction be made permanent. x
stranger who deals with the corporation on the faith of the ostensible x x[9] (Emphasis and underscoring supplied)
authority exercised by some of the corporate officers. It is therefore The complaint, docketed as Civil Case No. CEB-25762, was amended on
reasonable, in a case where an officer of a corporation has made a contract November 15, 2000.
in its name, that the corporation should be required, if it denies his
authority, to state such defense in its answer. [Merchant vs. International Branch 9 of the Cebu City RTC denied petitioner's prayer for a writ of
Banking Corporation, supra, and other cases ] preliminary injunction.[10] Petitioner filed a Motion for
SECOND DIVISION Reconsideration[11] and a Motion for Admission of a Second Amended
[ G.R. No. 150694, March 13, 2009 ] Complaint,[12] albeit it later filed a Motion to Withdraw Second Amended
Complaint and to admit Third Amended Complaint.[13] The trial court
ZOMER DEVELOPMENT COMPANY, INC. PETITIONER, VS. denied petitioner's Motion for Reconsideration.[14]
INTERNATIONAL EXCHANGE BANK AND SHERIFF IV
ARTHUR R. CABIGON, RESPONDENTS. Petitioner assailed the trial court's orders denying its prayer for the
issuance of a writ of preliminary injunction before the Court of Appeals
DECISION via certiorari,[15] docketed as CA-G.R. SP No. 64390 (certiorari case),
CARPIO MORALES, J.: alleging, in the main, that the real estate mortgage it executed was null and
On August 25, 1997, the Board of Directors of Zomer Development void for being ultra vires[16] as it was not empowered to mortgage its
Company, Inc. (petitioner) approved a resolution authorizing it to apply properties as security for the payment of obligations of third parties; and
for and obtain a credit line with respondent International Exchange Bank that Amparo and Zosa were authorized to mortgage its properties to secure
(IEB) in the amount of P60,000,000 as well as temporary excesses or only a P60,000,000 term loan and one credit facility of Prime
permanent increases thereon as may be approved by IEB from time to Aggregates.[17]
time.[1] The Board of Directors also authorized petitioner to assign, pledge,
or mortgage its properties as security for this credit line; and to secure and In the meantime, Branch 15 of the Cebu City RTC to which Civil Case
guarantee the term loan and other credit facility of IDHI Prime Aggregates No. CEB-25762 was re-raffled after the Presiding Judge of Branch 9
Corporation (Prime Aggregates) with IEB.[2] inhibited himself in the case, dismissed petitioner's Third Amended
Complaint[18] by Order of September 10, 2001. Petitioner appealed this
Prime Aggregates obtained on August 26, 1997 a term loan from IEB in Order to the Court of Appeals which docketed it as CA-G.R. CV No.
the amount of P60,000,000.[3] On September 2, 1997, petitioner, through 73063.
its Treasurer Amparo Zosa (Amparo) and its General Manager Manuel
Zosa, Jr. (Zosa), executed a real estate mortgage covering three parcels of By Decision[19] of October 30, 2001, the appellate court, acting on the
land (the real estate mortgage) in favor of IEB to secure certiorari case filed by petitioners, denied it due course as it found that the
trial court committed no grave abuse of discretion in denying petitioner's
The payment of all loans, overdrafts, credit lines and other credit facilities prayer for preliminary injunction.[20] It brushed aside petitioner's
or accommodations obtained or hereinafter obtained by the arguments that the real estate mortgage was ultra vires and that Amparo
MORTGAGOR and/or by IDHI Prime Aggregates and Zosa were only authorized to mortgage petitioner's properties to
Corporation (hereinafter referred to as DEBTOR) secure the P60,000,000 term loan and one credit facility of Prime
Aggregates. have already been committed, since they are continuing in nature and in
derogation of its rights at the outset, preliminary mandatory injunction
Hence, the present petition[21] for review faulting the Court of Appeals in may still be availed of to restore the status quo, citingManila Electric
Railroad and Light Company v. del Rosario and Jose.[26]
I - X X X NOT HOLDING THAT THE JUDGE WHO DENIED
PETITIONER'S APPLICATION FOR INJUNCTION WAS A BIASED Acting on petitioner's appeal from the dismissal by Branch 15 of
AND PARTIAL JUDGE AS RESPONDENTS WERE GIVEN A COPY its Third Amended Complaint, the appellate court, by Decision of April
OF THE ORDER ON MARCH 2, 2001 WHEN IT WAS SIGNED BY 14, 2005, set aside the trial court's order of dismissal and ordered the
THE JUDGE BUT BEFORE ITS OFFICIAL RELEASE ON MARCH 5, reinstatement of said complaint to the docket of Branch 15 of the Cebu
2001. City RTC.

II - X X X USING THE DECISION OF THIS HONORABLE COURT IN The records show that, indeed, petitioner's mortgaged properties were
THE CASE OF UNION BANK V. COURT OF APPEALS, ET. AL., 311 already foreclosed, as shown by the Certificate of Sale issued by Cabigon
SCRA 795 IN SAYING THAT PETITIONER IS NOT ENTITLED TO A on November 19, 2001.[27] And they also show that ownership of the
WRIT OF PRELIMINARY INJUNCTION INSTEAD OF USING THE lands-subject of the real estate mortgage had been consolidated and
CASE OF REPUBLIC V. COURT OF APPEALS, 324 SCRA 569 transfer certificates of title had been issued in IEB's name.[28] It is on this
WHEREIN THIS HONORABLE COURT HELD THAT EVEN P.D. 385 score that the Court finds petitioner's prayer for a writ of preliminary
CANNOT BE USED AS A SHIELD TO STOP BY INJUNCTION THE injunction moot and academic. This leaves it unnecessary for the Court to
FORECLOSURE OF A MORTGAGE WHERE THE VERY still dwell on petitioner's argument that it was not, under its By-Laws,
PROPRIETY OF SAID FORECLOSURE IS IN SERIOUS DOUBT empowered to mortgage its properties to secure the obligation of a third
WHICH IS THE SAME ISSUE RAISED IN THE CASE AT BAR. party. In any event, the Court finds well-taken the appellate court's
following disposition of such argument:
III - X X X HOLDING THAT [PRIME AGGREGATES] IS A
SUBSIDIARY OF PETITIONER IN THE ABSENCE OF A FINDING We do agree that the Petitioner, under its "By-Laws," is not empowered
THAT PETITIONER OWNS ANY SHARE IN [PRIME to mortgage its properties as a security for the payment of the obligations
AGGREGATES]. of third parties. This is on the general premise that the properties of a
corporation are regarded as held in trust for the payment of corporate
IV - X X X NOT HOLDING THAT THE SECRETARY'S creditors and not for the creditors of third parties. However, the Petitioner
CERTIFICATE OF PETITIONER WAS NULL AND VOID FOR NOT is not proscribed from mortgaging its properties as security for the
PUTTING ANY LIMITATION OF THE AMOUNT OF THE payment of obligations of third parties. In an opinion of the Securities and
OBLIGATION OF [PRIME AGGREGATES] TO BE SECURED BY A Exchange Commission, dated April 15, 1987, it declared that a private
THIRD PARTY MORTGAGE OF ITS PROPERTIES corporation, by way of exceptions, may give a third party mortgage:

V - X X X NOT HOLDING THAT THE THIRD PARTY REAL "1. When the mortgage of corporate assets/properties shall be done in the
ESTATE MORTGAGE EXECUTED BY THE AGENTS OF furtherance of the interest of the corporation and in the usual and regular
PETITIONER IN FAVOR OF PRIVATE RESPONDENT IS NULL AND course of its business; and
VOID BECAUSE THEY EXCEEDED THEIR AUTHORITY IN
SIGNING THE SAME. 2. To secure the debt of a subsidiary."
While admittedly, the "Opinion" of the Securities & Exchange
VI - X X X NOT CONSTRUING STRICTLY AGAINST PRIVATE Commission may not be conclusive on the Respondent Court, however,
RESPONDENT THE SECRETARY'S CERTIFICATE AND THIRD admittedly the same is of persuasive effect.
PARTY REAL ESTATE MORTGAGE WHICH WERE ALL
DOCUMENTS OF ADHESION AND ALL PREPARED BY IT AND TO In the present recourse, the Respondent Court found that not only is Prime
EFFECT THE LEAST TRANSMISSION OF RIGHTS PURSUANT TO Aggregates a subsidiary of the Petitioenr but that the Petitioner
ARTICLE 1378 OF THE NEW CIVIL CODE SINCE THE THIRD appeared to be a "family" corporation:
PARTY REAL ESTATE MORTGAGE IS A GRATUITOUS
CONTRACT WHICH WAS EXECUTED PURELY FOR "a. The plaintiff appears to be a family corporation. The incorporators and
ACCOMODATION OF [PRIME AGGREGATES]. stockholders and the membership of the board of directors are Zosa
family. x x x
VII - X X X NOT LAYING THE BLAME ON PRIVATE
RESPONDENT IN MAKING THE AGENTS OF PETITIONER SIGN b. Francis and Rolando Zosa are directors of [Prime Aggregates] and of
AN ILLEGAL CONTRACT SINCE IT WAS VERY WELL AWARE OF plaintiff corporation x x x
THEIR AUTHORITY AS ALL THE DOCUMENTS WERE ITS
FORMS, PRE-PRINTED AND PREPARED BY IT. c. The REM was executed by Amparo Zosa who was the treasurer of
plaintiff and Manuel Zosa, the General Manager, both are
VIII - X X X HOLDING THAT THE PETITIONER RATIFIED BY directors/stockholders of the plaintiff. Amparo Zosa is the biggest
INACTION THE ILLEGAL CONTRACT EXECUTED BY ITS stockholder and is the mother of practically all the other stockholders of
AGENTS SINCE THE PRIVATE RESPONDENT WAS VERY WELL plaintiff. Manuel Zosa, Jr. is the General Manager and a son of Amparo.
AWARE OF THE EXTENT OF THEIR AUTHORITY.
d. The Corporate Secretary of plaintiff and [Prime Aggregates] are
IX - MAKING CONFLICTING FINDINGS OF FACTS.[22] members of the Zosa family. The Corporate Secretary of [Prime
Respondents, in their Comment[23] dated February 27, 2002, move for the Aggregates] is also the daughter of Francis Zosa, president of plaintiff.
dismissal of the petition for being moot and academic, alleging that:
e. The President of plaintiff corporation, Francis Zosa and the president of
On October 8, 2001 [sic], [petitioner's] principal action for annulment [Prime Aggregates], Rolando Zosa, are brothers (aside from being
of real estate mortgage was dismissed by the trial court and that said common directors of both corporations.)
action is now on appeal with the Court of Appeals x x x [.] We agree with the Respondent Court.

On November 19, 2001, [petitioner's] mortgaged properties The Petitioner's shrill incantations that the "Resolution", approved by its
were foreclosed by [IEB]. In fact, as the highest bidder in the said Board of Directors, authorizing its Treasurer and General Manager to
foreclosure sale and in view of the passage of the new General Banking execute a "Real Estate Mortgage" as security for the payment of the
Law (which allows banks to consolidate its [sic] title within a shorter account of Prime Aggregates, a sister corporation, is not for its best
period if the mortgagor of a foreclosed property is a corporation), iBank interest, is a "puzzlement" xxx. Since when is a private corporation, going
had consolidated its title on the mortgaged properties. to the aid of a sister corporation, not for the best interest of both
corporation? For in doing so, the two (2) corporations are enhancing,
[Petitioner's] application for issuance of writ of preliminary injunction, the boosting and promoting a common interest, the interest of "family" having
subject of the instant appeal purportedly under Rule 45 of the Rules of ownership of both corporations. In the second place, Courts are loathe to
Court, cannot survive the dismissal of its principal action as well as the overturn decisions of the management of a corporation in the conduct of
foreclosure and consolidation in [IEB] name of its mortgaged its business via its Board of Directors x x x.
properties.[24] (Emphasis and underscoring supplied)
In its Reply,[25] petitioner argues that when Branch 15 of the Cebu City xxxx
RTC dismissed the Third Amended Complaint in Civil Case No. CEB-
25762 on September 10, 2001, it no longer had jurisdiction over it because There is no evidence on record that the "Real Estate Mortgage" was
said Branch had on August 14, 2001 been designated as a drug court. executed by the Petitioner and the Private Respondent to prejudice
corporate creditors of the Petitioner or will result in the infringement of
Petitioner goes on to argue that even if the acts sought to be restrained the trust fund doctrine or hamper the continuous business operation of the
Petitioner or that the Prime Aggregates was insolvent or incapable of property of such corporations controlled and held by the board of directors
paying the Private Respondent. Indeed, the latter approved Prime or trustees . . . x x x x x x x x x The corporation can also act through its
Aggregates' loan availments and credit facilities after its investigation of corporate officers who may be authorized either expressly by the by-laws
the financial capability of Prime Aggregates and its capacity to pay its or board resolutions or impliedly such as by general practice or policy or
account to the Private respondent.[29] as are implied from express powers.The general principles of agency
govern the relation between the corporation and its officers or agents.
xxxx When authorized, their acts can bind the corporation. Conversely, when
unauthorized, their acts cannot bind it.
[U]nder the "Resolution" of the Board of Directors, it authorized its
Treasurer and General Manager to execute a "Real Estate Mortgage" Same; Same; The corporation may ratify—expressly or impliedly—the
over its properties as security for the "term loan and credit facility" of unauthorized acts of its corporate officers.—The corporation may ratify
Prime Aggregates. The maximum amounts of such term loan and credit the unauthorized act of its corporate officer. Ratification means that the
facility were not fixed in the "Resolution". The term "credit facility" is a principal voluntarily adopts, confirms and gives sanction to some
broad term in credit business transactions to denote loans, pledges, unauthorized act of its agent on its behalf. It is this voluntary choice,
mortgages, trust receipt transactions and credit agreements. And then, knowingly made, which amounts to a ratification of what was theretofore
again, such term loan and/or credit facility may be granted, by the Private unauthorized and becomes the authorized act of the party so making the
Respondent, in favor of Prime Aggregates, in trenches or in staggered ratification. The substance of the doctrine is confirmation after conduct,
basis, each disbursement evidenced by separate agreements depending amounting to a substitute for a prior authority. Ratification can be made
upon the needs of Prime Aggregates for the establishment of its sand and either expressly or impliedly. Implied ratification may take various
gravel plant and port facilities and the purchase of equipments and forms—like silence or acquiescence, acts showing approval or adoption of
machinery for said project. Hence, the "Long Term Agreements" and the act, or acceptance and retention of benefits flowing therefrom.
"Credit Agreements" executed by Prime Aggregates and the Private
Respondent, with the Petitioner's properties, as collateral therefore, were Same; Same; Loans; Special Power of Attorney; The power to borrow
envisaged in the terms "term loan and credit facility" in the "Resolution" money is one of those cases where corporate officers as agents of the
of the Board of Directors of the Petitioner. corporation need a special power of attorney.—The power to borrow
money is one of those cases where corporate officers as agents of the
The intention of the Members of the Board of Directors of the Petitioner, corporation need a special power of attorney. In the case at bar, no special
in approving the "Resolution," may be ascertained xxx also from the power of attorney conferring authority on de Villa was ever presented. The
contemporaneous and subsequent acts of the Petitioner, the Private promissory notes evidencing the loans were signed by de Villa (who was
Respondent and Prime Aggregates. Given the factual milieu in the present the president of respondent corporation) as borrower without indicating in
recourse, as found and declared by the Respondent Court, there can be no what capacity he was signing them. In fact, there was no mention at all of
equivocation that, indeed the Petitioner conformed to and ratified, and respondent corporation. On their face, they appeared to be personal loans
hence, is bound by the execution, by its Treasurer and General Manager, of de Villa.
of the "Real Estate Mortgage" in favor of the Private respondent, with its
properties used as securities for the payment of the credit and loan Same; Same; Same; Ratification is a voluntary choice that is knowingly
availments of Prime Aggregates from the Private Respondent on the basis made.—Respondent corporation could not have ratified the act of de Villa
of the "Resolution" approved by its Board of Directors. As our Supreme because there was no proof that it knew that he took out a loan on its
Court declared, ratification and/or approval by the corporation of the acts behalf. As stated earlier, ratification is a voluntary choice that is
of its agents/officers may be ascertained through x x x the acquiescence in knowingly made. The corporation could not have ratified an act it had no
his acts of a particular nature, with actual or constructive thereof, whether knowledge of: x x x x x x x x x Ordinarily, the principal must have full
within or beyond the scope of his ordinary powers. knowledge at the time of ratification of all the material facts and
circumstances relating to the unauthorized act of the person who assumed
As it was, the Petitioner finally awoke from its slumber when the Private to act as agent. Thus, if material facts were suppressed or unknown, there
Respondent filed its "Petition" for the extra-judicial foreclosure of the can be no valid ratification . . . . The fact that the corporation admitted
"Real Estate Mortgage", with the Sheriff, and assailed the authority of its receiving the proceeds of the loan did not amount to ratification of the
Board of Directors to approve the said "Resolution" and of its Treasurer loan. It accepted the amount from de Villa, its president at that time, in
and General Manager to execute the deed and brand the said "Resolution" good faith. Good faith is always presumed. Petitioner did not show that the
and the said deed as "ultra vires" and hence, not binding on the Petitioner, corporation acted in bad faith. It follows that respondent corporation was
and hurried off to the Respondent Court and prayed for injunctive relief. not liable for the subsequent loss of the money which it accepted as an
Before then, the Petitioner maintained a stoic silence and adopted a "hands investment. It could not be faulted for not knowing that it was the
off" stance. We find the Petitioner's stance grossly inequitable. We must proceeds of a loan obtained by de Villa. It was under no obligation to
take heed and pay obeisance to the equity rule that if one maintains silence check the source of the investments which went into its coffers. As long as
when, in conscience he ought to speak, equity will debar him from the investment was used for legitimate corporate purposes, the investor
speaking when, in conscience, he ought to remain silent. He who remains bore the risk of loss.
silent when he ought to speak cannot be heard to speak when he ought to
be silent. More, the transactions between the Petitioner and the Private Same; Same; Same; Investments; As long as the investment is used for
Respondent over its properties are neither malum in se or malum legitimate corporate purposes, the investor bears the risk of loss.—The
prohibitum. Hence, the Petitioner cannot hide behind the cloak of "ultra fact that the corporation admitted receiving the proceeds of the loan did
vires" for a defense. not amount to ratification of the loan. It accepted the amount from de
Villa, its president at that time, in good faith. Good faith is always
xxxx presumed. Petitioner did not show that the corporation acted in bad faith.
It follows that respondent corporation was not liable for the subsequent
The plea of "ultra vires" will not be allowed to prevail, whether interposed loss of the money which it accepted as an investment. It could not be
for or against a corporation, when it will not advance justice but, on the faulted for not knowing that it was the proceeds of a loan obtained by de
contrary, will accomplish a legal wrong to the prejudice of another who Villa. It was under no obligation to check the source of the investments
acted in good faith.[30] (Underscoring and emphasis in the original) which went into its coffers. As long as the investment was used for
WHEREFORE, the petition is DISMISSED. legitimate corporate purposes, the investor bore the risk of loss.

Costs against petitioner. Same; Same; Real Estate Mortgage; Special Power of Attorney; In the
absence of a special power of attorney in favor of the corporation’s
SO ORDERED. president, no valid mortgage could be executed by him.—A special power
of attorney is necessary to create or convey real rights over immovable
Quisumbing, Tinga, Velasco, Jr., and Brion, JJ., concur. property. Furthermore, the special power of attorney must appear in a
public document. In the absence of a special power of attorney in favor of
Yasuma vs. Heirs of Cecilio S. De Villa, 499 SCRA 466 , August 22, de Villa as president of the corporation, no valid mortgage could have
2006 been executed by him. Since the mortgage was void, it could not be
ratified. Petitioner cannot blame anyone but himself. He did not check if
Corporation Law; Agency; The general principles of agency govern the the person he was dealing with had the authority to mortgage the property
relation between the corporation and its officers or agents—when being offered as collateral. [suma vs. Heirs of Cecilio S. De Villa, 499
authorized, their acts bind the corporation, otherwise, their acts cannot SCRA 466(2006)]
bind it.—A corporation is a juridical person, separate and distinct from its
stockholders. Being a juridical entity, a corporation may act through its WOODCHILD HOLDINGS, INC. vs ROXAS ELECTRIC AND
board of directors, as provided in Section 23 of the Corporation Code of CONSTRUCTION COMPANY, INC.
the Philippines: Sec. 23. The Board of Directors or Trustees.—Unless - Roxas Electric and Construction Company, Inc. (RECCI) authorized its
otherwise provided in this Code, the corporate powers of all corporations President Roberto B. Roxas through a resolution to sell a parcel of land
formed under this Code shall be exercised, all business conducted and all
owned by the corporation, and to execute, sign and deliver for and on DOCTRINE:
behalf of the company. If a corporation, however, consciously lets one of its officers, or any other
- Petitioner Woodchild Holdings, Inc. (WHI) through its President agent, to act within the scope of an apparent authority, it will be estopped
Jonathan Y. Dy, offered to buy the land from RECCI. from denying such officer’s authority.
- The offer to purchase stated that it is made on the representation and FACTS:
warranty of the OWNER/SELLER, that he holds a good and registrable Respondent Inland Construction and Development Corp. (Inland) obtained
title to the property, which shall be conveyed CLEAR and FREE of all various loans from petitioner Westmont Bank (Westmont). To secure the
liens and encumbrances, and that in the event that the right of way is payment of its obligations, Inland executed Real Estate Mortgages over
insufficient for the buyer’s purpose, the seller agrees to sell additional three real properties and issued promissory notes in favor of the bank.
square meter from his current adjacent property to allow the buyer full By a Deed of Assignment, Conveyance and Release, one Felix Aranda,
access and full use of the property. assigned and conveyed all his rights and interests at Hanil-Gonzales
- Roxas accepted the offer and indicated his acceptance on Page 2 of the Construction & Development Phils. Corporation (HGCDP) in favor of
Deed. Horacio Abrante. Under the same Deed, it appears that HGCDP assumed
- The sale was consummated. the obligations of Inland. Westmont’s Account Officer, Lionel Calo, Jr.
- WHI subsequently entered into a construction agreement with Wimbeco (Calo), signed for its conformity to the deed. Inland was subsequently
Builder’s Inc. (WBI) for the construction of a warehouse, and a lease served with a Notice of Sheriff’s Sale foreclosing the real estate mortgages
agreement with Poderosa Leather Goods Company, Inc. with a condition over its real properties prompting it to file a complaint for injunction
that the warehouse be ready by April 1, 1992. against the Westmont.
- The building was finished and Poderosa became the lessee. In its answer, Westmont underscored that it had no knowledge, much less
- WHI complained to Roberto Roxas that the vehicles of RECCI were did it give its conformity to the alleged assignment of the obligation. The
parked on a portion of the property over which WHI had been granted a trial court found that Westmont ratified the act of Calo. It accordingly
right of way. Roxas promised to look into the matter. Dy and Roxas rendered judgment in favor of Inland. On appeal, the appellate court
discussed the need of the WHI to buy a 500-square-meter portion the affirmed the trial court’s decision insofar as it finds Westmont to have
adjacent lot as provided for in the deed of absolute sale. However, Roxas ratified the Deed of Assignment.
died soon thereafter. ISSUE:
- WHI wrote the RECCI, reiterating its verbal requests to purchase a Whether or not Westmont Bank ratified the Deed of Assignment
portion of the said lot as provided for in the deed of absolute sale, and HELD:
complained about the latter’s failure to eject the squatters within the three- The general rule remains that, in the absence of authority from the board
month period agreed upon in the said deed. of directors, no person, not even its officers, can validly bind a
- RECCI rejected the demand of WHI, so WHI filed a case for Specific corporation. If a corporation, however, consciously lets one of its officers,
Performance and Damages in the RTC of Makati. or any other agent, to act within the scope of an apparent authority, it will
be estopped from denying such officer’s authority.
RTC - in favor of WHI. The records show that Calo was the one assigned to transact on
CA - reversed the RTC decision and dismissed the complaint. The CA petitioner’s behalf respecting the loan transactions and arrangements of
ruled that, under the resolution of the Board of Directors of the RECCI, Inland as well as those of Hanil-Gonzales and Abrantes. Since it
Roxas was merely authorized to sell the first lot, but not to grant right of conducted business through Calo, who is an Account Officer, it is
way in favor of the WHI over a portion of the second lot, or to grant an presumed that he had authority to sign for the bank in the Deed of
option to the petitioner to buy a portion thereof. Assignment.
Unmistakably, the Court’s directive in Yao Ka Sin Trading is that a
ISSUE - WON respondent is bound by the provisions of the deed of sale corporation should first prove by clear evidence that its corporate officer is
granting to the petitioner the beneficial use and right of way over the not in fact authorized to act on its behalf before the burden of evidence
adjacent lot of the lot they previously bought. WON such provision is shifts to the other party to prove, by previous specific acts, that an officer
enforceable. was clothed by the corporation with apparent authority. In the present
petitions, Westmont Bank failed to discharge its primary burden of
SC - We agree with respondent. Judgment of CA affirmed with proving that Calo was not authorized to bind it, as it did not present proof
modification. that Calo was unauthorized. It did not present, much less cite, any
- A corporation is a juridical person separate and distinct from its Resolution from its Board of Directors or its Charter or By-laws from
stockholders or members. Accordingly, the property of the corporation is which the Court could reasonably infer that he indeed had no authority to
not the property of its stockholders or members and may not be sold by the sign in its behalf or bind it in the Deed of Assignment.
stockholders or members without express authorization from the
corporation’s board of directors. Western Institute of Technology Inc. vs. Salas
- Indubitably, a corporation may act only through its board of directors or, [GR 113032, 21 August 1997]
when authorized either by its by-laws or by its board resolution, through
its officers or agents in the normal course of business. The general Facts: Salas family, are the majority and controlling members of the
principles of agency govern the relation between the corporation and its Board of Trustees of Western Institute of Technology, Inc. (WIT), a stock
officers or agents, subject to the articles of incorporation, by-laws, or corporation engaged in the operation, among others, of an educational
relevant provisions of law. institution. According to the Villasis the minority stockholders of WIT,
- Generally, the acts of the corporate officers within the scope of their sometime on 1 June 1986 in the principal office of WIT at La Paz, Iloilo
authority are binding on the corporation. However, under Article 1910 of City, a Special Board Meeting was held. In attendance were other
the New Civil Code, acts done by such officers beyond the scope of their members of the Board including Reginald Villasis. Prior to said Special
authority cannot bind the corporation unless it has ratified such acts Board Meeting, copies of notice thereof, dated 24 May 1986, were
expressly or tacitly, or is estopped from denying them. distributed to all Board Members. The notice allegedly indicated that the
- In this case, the respondent denied authorizing its then president Roberto meeting to be held on 1 June 1986 included Item 6 which states that
B. Roxas to sell a portion of Lot No. 491-A-3-B-1 covered by TCT No. "Possible implementation of Art. III, Sec. 6 of the Amended By-Laws of
78085, and to create a lien or burden thereon. The petitioner was thus Western Institute of Technology, Inc. on compensation of all officers of
burdened to prove that the respondent so authorized Roxas to sell the same the corporation." In said meeting, the Board of Trustees passed Resolution
and to create a lien thereon. 48, series 1986, granting monthly compensation to Salas, et. al. as
- Evidently, Roxas was not specifically authorized under the said corporate officers retroactive 1 June 1985, in the following amounts:
resolution to grant a right of way in favor of the petitioner on a portion of “Chairman 9,000.00/month, Vice Chairman P3,500.00/month, Corporate
the second lot or to agree to sell to the petitioner a portion thereof. Treasurer P3,500.00/month and Corporate Secretary P3,500.00/month,
- For the principle of apparent authority to apply, the petitioner was retroactive June 1, 1985 and the ten percentum of the net profits shall be
burdened to prove the following: (a) the acts of the respondent justifying distributed equally among the ten members of the Board of Trustees. This
belief in the agency by the petitioner; (b) knowledge thereof by the shall amend and supercede any previous resolution.”
respondent which is sought to be held; and, (c) reliance thereon by the
petitioner consistent with ordinary care and prudence.[34] In this case, A few years later, or on 13 March 1991 the Villasis and Dimas Enriquez
there is no evidence on record of specific acts made by the respondent[35] filed an affidavit-complaint against Salas, et. al. before the Office of the
showing or indicating that it had full knowledge of any representations City Prosecutor of Iloilo, as a result of which 2 separate criminal
made by Roxas to the petitioner that the respondent had authorized him to informations, one for falsification of a public document under Article 171
grant to the respondent an option to buy a portion of Lot No. 491-A-3-B-1 of the Revised Penal Code and the other for estafa under Article 315, par.
covered by TCT No. 78085, or to create a burden or lien thereon, or that 1(b) of the RPC, were filed before Branch 33 of the Regional Trial Court
the respondent allowed him to do so. of Iloilo City. The charge for falsification of public document was
anchored on Salas, et. al.'s submission of WIT's income statement for the
WESTMONT BANK v. INLAND CONSTRUCTION AND fiscal year 1985-1986 with the Securities and Exchange Commission
DEVELOPMENT CORP. (SEC) reflecting therein the disbursement of corporate funds for the
582 SCRA 230 (2009), SECOND DIVISION (Carpio Morales, J.) compensation of Salas, et. al. based on Resolution 4, series of 1986,
making it appear that the same was passed by the board on 30 March
1986, when in truth, the same was actually passed on 1 June 1986, a date that the Board of Directors do not agree with the valuation set by
not covered by the corporation's fiscal year 1985-1986 (beginning May 1, the court.
1995 and ending April 30, 1986). Thereafter, trial for the two criminal  The trial court rendered judgment ordering Hi Cement to pay the
cases (Criminal Cases 37097 and 37098), was consolidated. After a full- plaintiffs.
blown hearing, Judge Porfirio Parian handed down a verdict of acquittal  A new motion was filed by the plaintiff on the ground that
on both counts dated 6 September 1993 without imposing any civil decision was based on a CA which was null and void for want of
liability against the accused therein. Villasis, et. al. filed a Motion for a special authority by Hi Cement’s lawyers to enter into said
Reconsideration of the civil aspect of the RTC Decision which was, agreement.
however, denied in an Order dated 23 November 1993. Villasis, et. al.
filed the petition for review on certiorari. Significantly on 8 December Issue:
1994, a Motion for Intervention, dated 2 December 1994, was filed before W/N the compromise agreement entered into by corporration’s
this Court by Western Institute of Technology, Inc., disowning its lawyer is valid.
inclusion in the petition and submitting that Atty. Tranquilino R. Gale,
counsel for Villasis, et. al., had no authority whatsoever to represent the Held:
corporation in filing the petition. Intervenor likewise prayed for the No.
dismissal of the petition for being utterly without merit. The Motion for
Intervention was granted on 16 January 1995. Ruling:
Special powers of attorney are necessary in the ff: to compromise, and to
Issue: Whether the grant of compensation to Salas, et. al. is proscribed renounce the right to appeal from a judgment. Attorneys have authority to
under Section 30 of the Corporation Code. bind their clients in any case by any agreement in relation thereto made in
writing, and in taking appeals, and in all matters of ordinary judicial
Held: Directors or trustees, as the case may be, are not entitled to salary or procedure, but they cannot, without special authority/compromise their
other compensation when they perform nothing more than the usual and clients’ litigation, or receive anything in discharge of their clients’ claims
ordinary duties of their office. This rule is founded upon a presumption but the full amount in cash.
that directors/trustees render service gratuitously, and that the return upon The Compromise Agreement was signed only by the lawyers for
their shares adequately furnishes the motives for service, without petitioners and by the lawyers for private respondent corporation.
compensation. Under Section 30 of the Corporation Code, there are only The Court held that the Rules require for attorneys to compromise the
two (2) ways by which members of the board can be granted litigation of their clients, a special authority. And while the same does not
compensation apart from reasonable per diems: (1) when there is a state that the special authority be in writing, the same be duly established
provision in the by-laws fixing their compensation; and (2) when the by evidence other than the self-serving assertion of counsel himself that
stockholders representing a majority of the outstanding capital stock at a such authority was verbally given him.
regular or special stockholders' meeting agree to give it to them. Also, the Law specifically required that juridical persons may compromise only in
proscription, however, against granting compensation to director/trustees the form and with the requisites which may be necessary to alienate their
of a corporation is not a sweeping rule. Worthy of note is the clear property. Under the corporation law the power to compromise or settle
phraseology of Section 30 which state: "[T]he directors shall not receive claims in favor of or against the corporation is ordinarily and primarily
any compensation, as such directors." The phrase as such directors is not committed to the Board of Directors.
without significance for it delimits the scope of the prohibition to As a general rule, an officer or agent of the corporation has no power to
compensation given to them for services performed purely in their compromise or settle a claim by or against the corporation, except to the
capacity as directors or trustees. The unambiguous implication is that extent that such power is given to him either expressly or by reasonable
members of the board may receive compensation, in addition to implication from the circumstances.
reasonable per diems, when they render services to the corporation in a A corporation officer's power as an agent of the corporation must therefore
capacity other than as directors/trustees. Herein, resolution 48, s. 1986 be sought from the statute, the charter, the by-laws, or in a delegation of
granted monthly compensation to Salas, et. al. not in their capacity as authority to such officer, from the acts of board of directors, formally
members of the board, but rather as officers of the corporation, more
particularly as Chairman, Vice-Chairman, Treasurer and Secretary of expressed or implied from a habit or custom of doing business.
Western Institute of Technology. Clearly, therefore, the prohibition with In the case at bar no provision of the charter and by-laws of the
respect to granting compensation to corporate directors/trustees as such corporation or any resolution or any other act of the board of directors of
under Section 30 is not violated in this particular case. Consequently, the HI Cement Corporation has been cited, from which We could reasonably
last sentence of Section 30 which provides that "In no case shall the total infer that the administrative manager had been granted expressly or
yearly compensation of directors, as such directors, exceed ten (10%) impliedly the power to bind the corporation or the authority to
percent of the net income before income tax of the corporation during the compromise the case. Absent such authority to enter into the compromise,
preceding year" does not likewise find application in this case since the the signature of Atty. Cardenas on the agreement would be legally
compensation is being given to Salas, et. al. in their capacity as officers of
ineffectual.
WIT and not as board members.

Ignacio Vicente an Moises Angeles v Hon. Ambrosio Geraldez and Hi Valle Verde Country Club, Inc. v. Africa
Cement Corporation G.R. No. 151969, September 4, 2009, (Brion, J.)
GR. No. L-32473, 31 July 1973
Juan Bernabe v Hi Cement Corporation and Hon Ambrosio Geraldez FACTS: In 1996, during the Annual Stockholders’ Meeting of Valle
GR No. L-32483, 31 July 1973 Verde Country Club, Inc. (VVCC), Villaluna, Dinglasan, Makalintal,
Ortigas III, Salta, Santiago, Jr., Dee, Sunico, and Gamboa were elected as
Facts: members of the VVCC Board of Directors. From 1997 to 2001, the
requisite quorum for the holding of the stockholders’ meeting could not be
 Hi Cement filed a complaint for injuction and damages against
obtained. Consequently, the above-named directors continued to serve in
petitioners alleging that it had acquired Placer Lease Contract
the VVCC Board in a hold-over capacity.
under a deed of sale and transfer which was duly registered with
In 1998, Dinglasan resigned from his position. He was replaced by Roxas
the Office of the Mining Recorder of Bulacan and duly approved
who was elected by the board still constituting a quorum. A year later,
by the Sec. of Agriculture and Natural Resources for a period of
Makalintal also resigned and was replaced by Jose Ramirez in 2001.
25 years covering two mining claims.
Ramirez was elected by the remaining members of the Board.
 Within the limits of the Placer Mining Claims are three parcels
Africa, a member of VVCC, questioned the election of Roxas and
of land each owned by petitioners.
Ramirez with the SEC and the RTC, respectively. Before the RTC, Africa
 Hi Cement prayed that the petitioners allow it, its agents and alleged that a year after Makalintal’s election as member of the VVCC
workers to enter, develop and extract minerals from the areas Board in 1996, his term – as well as those of the other members of the
claimed by defendants. Upon suggestion of the court, a surveyor VVCC Board – should be considered to have already expired. According
surveyed the area to relocate the boundaries of the mining to him, for the members to exercise the authority to fill in vacancies in the
claims, which showed that the properties of the plaintiffs were board of directors, that there should be an unexpired term during which
covered the successor-member shall serve. Further, that the resulting vacancy
 The counsels of parties conferred on the possibility of should have been filled by the stockholders in a regular or special meeting
terminating the case by compromise, the plaintiffs having called for that purpose, and not by the remaining members of the VVCC
previously signified their willingness to sell their respective Board, as was done in this case.
properties. The RTC and the SEC ruled in favor of Africa. VVCC filed a petition for
 Counsels of parties executed and submitted to the court for its review on certiorari.
approval a Compromise Agreement, which was approved and ISSUE: Whether or not the remaining directors of the corporation’s
enjoined the parties to comply with the terms and conditions. Board, still constituting a quorum, can elect another director to fill in a
One of the three lawyers of Hi Cement sent a copy of the CA to vacancy caused by the resignation of a hold-over director.
the president of the corporation. However, the latter answered
RULING: Petition DENIED. Under Section 29 of the Corporation Code, meeting, Petitioners Ernesto Tanchi, Edwin Ngo, Virginia Khoo, and
a vacancy occurring in the board of directors caused by the expiration of a Judith Tan were voted to replace the four deceased member-trustees. The
member’s term shall be filled by the corporation’s stockholders. As the controversy reached SEC and the petitioners maintained that the deceased
vacancy in this case was caused by Makalintal’s resignation, not by the member-trustees should not be counted in the computation of the quorum
expiration of his term, VVCC insists that the board rightfully appointed because, upon their death, members automatically lost all their rights
Ramirez to fill in the vacancy. (including the right to vote) and interests in the corporation. SEC declared
The holdover period is not part of the term of office of a member of the the meeting null and void and ruled that the phrase “entitled to vote” under
board of directors. In several cases, we have defined "term" as the time Sec 24 should be read with Sec 89 of Corpo Code.
during which the officer may claim to hold the office as of right, and fixes ISSUE: In a non-stock corporation, should dead members still be counted
the interval after which the several incumbents shall succeed one in determination of quorum for purposed of conducting the Annual
another. The term of office is not affected by the holdover. Members’ Meeting?
Section 23 of the Corporation Code declares that the term of the members HELD: For stock corporations, the "quorum" referred to in Section 52 of
of the board of directors shall be only for one year; their term expires one the Corporation Code is based on the number of outstanding voting stocks.
year after election to the office. After the lapse of one year from his For nonstock corporations, only those who are actual, living members with
election, Makalintal’s term of office is deemed to have already expired. voting rights shall be counted in determining the existence of a quorum
With the expiration of Makalintal’s term of office, a vacancy resulted during members’ meetings. Dead members shall not be counted.
which, by the terms of Section 29, must be filled by the stockholders of One of the most important rights of a qualified shareholder or member is
VVCC in a regular or special meeting called for the purpose. His the right
resignation as a holdover director did not change the nature of the to vote -- either personally or by proxy -- for the directors or trustees who
vacancy; the vacancy due to the expiration of Makalintal’s term had been are to manage the corporate affairs. The right to vote is inherent in and
created long before his resignation. incidental to the ownership of corporate stocks. In nonstock corporations,
the voting rights attach to membership. The principle for determining the
Umale v. ASB Realty Corporation quorum for stock corporations is applied by analogy to nonstock
[G.R. 181126, June 15, 2011] corporations, only those who are actual members with voting rights should
TOPIC: Restrictions on capacity to act (Insolvency and Trusteeship be counted. Under Section 52, the majority of the members representing
PONENTE: Del Castillo, J. the actual number of voting rights, not the number or numerical constant
FACTS: (chronological order) that may originally be specified in the articles of incorporation, constitutes
1996: Amethyst Pearl executed a Deed of Assignment in Liquidation of a the quorum.
parcel of land in favor of ASB Realty in consideration of Amethyst Pearl’s Having thus determined that the quorum in a members’ meeting is to be
outstanding capital stock from ASB Realty making ASB Realty the owner reckoned as the actual number of members of the corporation, the next
of the parcel of land. question to resolve is what happens in the event of the death of one of
them. In stock corporations, the executor or administrator duly appointed
Sometime in 2003: ASB Realty commenced an action in the MTC for by the Court is vested with the legal title to the stock and entitled to vote
unlawful detainer against Umale. ASB Realty alleged that it entered into a it. Until a settlement and division of the estate is effected, the stocks of the
lease contract with Umale for the period June 1, 1999-May 31, 2000. decedent are held by the administrator or executor. On the other hand,
Their agreement was for Umale to conduct a pay-parking business on the membership in and all rights arising from a nonstock corporation are
property and pay a monthly rent of P60,720.00. Upon the contract's personal and non-transferable, unless the articles of incorporation or the
expiration on continued occupying the premises and paying rentals. bylaws of the corporation provide otherwise. In other words, the
determination of whether or not "dead members" are entitled to exercise
June 2003: ASB Realty served on Umale a Notice of Termination of Lease their voting rights (through their executor or administrator), depends on
and Demand to Vacate and Pay. ASB Realty stated that it was terminating those articles of incorporation or bylaws.
the lease effective midnight of June 30, 2003.Umale failed to comply with Under the By-Laws of GCHS, membership in the corporation shall,
ASB Realty's demands and continued in possession of the subject among others, be terminated by the death of the member. Applying
premises, even constructing commercial establishments thereon Section 91, dead members who are dropped from the membership roster in
the manner and for the cause provided for in the By-Laws of GCHS are
ISSUE(S): Can a corporate officer of ASB Realty (duly authorized by the not to be counted in determining the requisite vote in corporate matters or
Board of Directors) file suit to recover an unlawfully detained corporate the requisite quorum for the annual members’ meeting. With 11 remaining
property despite the fact that the corporation had already been placed members, the quorum in the present case should be 6. Therefore, there
under rehabilitation? being a quorum, the annual members’ meeting was valid.

HELD: Yes TUASON VS. BOLANOS


RATIO: GR. No. L-4935. May 28, 1954
- What petitioners argue is that the corporate officer of ASB 95 Phil. 106
Realty is incapacitated to file this suit to recover a corporate CASE DIGEST
property because ASB Realty has a duly-appointed Facts:
rehabilitation receiver. Allegedly, this rehabilitation receiver is Plaintiff’s complaint against defendant was to recover possession of a
the only one that can file the instant suit. registered land. In the complaint, the plaintiff is represented by its
- Corporations, such as ASB Realty, are juridical entities that Managing Partner, Gregorio Araneta, Inc., another corporation.
exist by operation of law. As a creature of law, the powers and Defendant, in his answer, sets up prescription and title in himself thru
attributes of a corporation are those set out, expressly or "open, continuous, exclusive and public and notorious possession under
impliedly, in the law. claim of ownership, adverse to the entire world by defendant and his
- Corporate Rehabilitation’s concept of preserving the predecessors in interest" from "time immemorial". After trial, the lower
corporation’s business as a going concern while it is undergoing court rendered judgment for plaintiff, declaring defendant to be without
rehabilitation is called debtor-in-possession or debtor-in-place. any right to the land in question and ordering him to restore possession
CASE LAW/ DOCTRINE: thereof to plaintiff and to pay the latter a monthly rent. Defendant
Corporate rehabilitation is defined as “the restoration of the debtor to as appealed directly to the Supreme Court and contended, among others, that
position of successful operation and solvency, if it is shown that its Gregorio Araneta, Inc. can not act as managing partner for plaintiff on the
continuance of operation is economically feasible and its creditors can theory that it is illegal for two corporations to enter into a partnership
recover by way of the present value of payments projected in the plan
more if the corporation continues as a going concern than if it is Issue:
immediately liquidated”
Whether or not a corporation may enter into a joint venture with another
G.R. No. 153468 August 17, 2006 corporation.
PAUL LEE TAN, ANDREW LIUSON, ESTHER WONG, STEPHEN
CO, JAMES TAN, JUDITH TAN, ERNESTO TANCHI JR., EDWIN Ruling:
NGO, VIRGINIA KHOO, SABINO PADILLA JR., EDUARDO P.
LIZARES and GRACE CHRISTIAN HIGH SCHOOL, It is true that the complaint states that the plaintiff is "represented herein
Petitioners,vs.PAUL SYCIP and MERRITTO LIM, Respondents by its Managing Partner Gregorio Araneta, Inc.", another corporation, but
. there is nothing against one corporation being represented by another
FACTS: Grace Christian High School (GCHS) is a nonstock, non-profit person, natural or juridical, in a suit in court. The contention that Gregorio
educational corporation with 15 regular members, who also constitute the Araneta, Inc. cannot act as managing partner for plaintiff on the theory
board of trustees. During the annual members’ meeting, there were only that it is illegal for two corporations to enter into a partnership is without
11 living member-trustees, as 4 have already died. Out of the 11, 7 merit, for the true rule is that "though a corporation has no power to enter
attended the meeting through their respective proxies. The meeting was into a partnership, it may nevertheless enter into a joint venture with
convened and chaired by Atty. Sabino Padilla Jr. over the objection of another where the nature of that venture is in line with the business
Atty. Antonio C. Pacis, who argued that there was no quorum. In the authorized by its charter." (Wyoming-Indiana Oil Gas Co. vs. Weston, 80
A. L. R., 1043, citing 2. Fletcher Cyc. of Corp., 1082.). There is nothing in  The fact that Filfinance owns majority shares in Filriters is not
the record to indicate that the venture in which plaintiff is represented by by itself a ground to disregard the independent corporate status
Gregorio Araneta, Inc. as "its managing partner" is not in line with the of Filriters.
corporate business of either of them.  Traders knew that Philfinance is not registered owner of the
CBCI.
Negotiable Instruments Case Digest: Traders Royal Bank v. CA o The fact that a non-owner was disposing of the
(1997) registered CBCI owned by another entity was a good
reason for petitioner to verify of inquire as to the title
FACTS: Philfinance to dispose to the CBCI.
Filriters (assigned) > Philfinance (still under the name of Filriters
assigned) > Traders Royal Bank = ? (valid or not) Strong and Strong vs. Repide
 November 27, 1979: Filriters Guaranty Assurance Corporation 41 Phil. 947 3 May 1909
(Filriters) executed a "Detached Assignment whereby Filriters,
as registered owner, sold, transferred, assigned and delivered PONENTE: Justice Peckham
unto Philippine Underwriters Finance Corporation (Philfinance) FACTS:
all its rights and title to Central Bank Certificates of Among the lands comprising the friar lands are the Dominican
Indebtedness (CBCI) of P500k and having an aggregate value of lands, the only valuable asset owned by the corporation Philippine Sugar
P3.5M Estates Development Company Limited (Philippine Sugar Estates).
o The Detached Assignment contains an express Francisco Gutierrez Repide (Repide), defendant, was the majority
authorization executed by the transferor intended to stockholder and one of the five directors of Philippine Sugar Estates. He
complete the assignment through the registration of was likewise elected by the board as the agent and administrator general of
the transfer in the name of PhilFinance such company.
 February 4, 1981: Traders Royal Bank (Traders) entered into a
Repurchase Agreement w/ PhilFinance whereby in consideration The factual backdrop being during US occupation, the US
of the sum of P500,000.00, PhilFinance sold, transferred and Government wanted to secure title over the friar lands. To accomplish this
delivered a CBCI w/ a face value of P500K which CBCI was objective, Governor for the Philippines entered into negotiations for the
among those previously acquired by PhilFinance from Filriters purchase of the Dominican lands, during which Repide represented
 PhilFinance failed to repurchase on the agreed date of maturity, Philippine Sugar Estates. The first offer of the Governor was to purchase
April 27, 1981, when the checks it issued in favor of petitioner the subject lands in the amount of $6,043,219.47. As the majority
were dishonored for insufficient funds stockholder of Philippine Sugar Estates and without prior consultation
 Philfinance transferred and assigned all, its rights and title in the with the other stockholders, Repide rejected the offer. For the second
CBCI to Traders offer, the purchase price was increased to $7,535,000.
 Respondent failed and refused to register the transfer as
While negotiations for the second offer were ongoing and while
requested, and continues to do so notwithstanding petitioner's
still holding out for a higher price of the Dominican lands, Repide took
valid and just title over the same and despite repeated demands
steps to purchase the 800 shares of stock of Philippine Sugar Estates.
in writing
These shares were owned by Mrs. Eleanor Strong (Strong) which were
 Traders prayed for the registration by the Central Bank of the then in the possession of her agent, F. Stuart Jones (Jones). Repide, instead
subject CBCI in its name. of seeing Jones, employed Kauffman who later on employed Sloan, a
 CA affirmed RTC: subsequent assignment in favor of Traders broker, to purchase the shares of Strong. Jones sold the 800 shares of
Royal Bank null and void and of no force and effect. Strong for 16,000 Mexican currency. For this sale transaction a check of
o Philfinance acquired no title or rights under CBCI one Rueda Ramos was issued.
which it could assign or transfer to Traders and which
it can register with the Central Bank Later on, the negotiations for the purchase of the Dominican
o instrument is payable only to Filriters, the registered lands were concluded and a contract of sale was subsequently executed.
owner This sale transaction increased the value of the shares of stocks originally
ISSUE: owned by Strong from 16,000 Mexican currency to 76,256 US currency.
During the negotiations regarding the purchase of the shares of stock of
W/N the CBCI is a negotiable instrument Strong, not one word of the facts affecting the value of this stock was
made known to her nor her agent, Jones. After the sale of Dominican lands
HELD: and after the purchase of the 800 shares of Strong, Repide became the
owner of 30,400 out of the 42,030 shares of Philippine Sugar Estates.
NO. Petition is dismissed. CA affirmed.
Strong filed a complaint for the recovery of her 800 shares. She
 CBCI is not a negotiable instrument in the absence of words of argued that her agent Jones had no authority to sell her shares and that
negotiability within the meaning of the Repide fraudulently concealed the facts affecting their value.
negotiable instruments law (Act 2031)
 certificate of indebtedness ISSUE:
o = certificates for the creation and maintenance of a Was there fraud in effecting the purchase of Strong’s shares?
permanent improvement revolving fund
o similar to a "bond" RULING:
 properly understood as acknowledgment of Yes. With the factual circumstances of this case, it became the
an obligation to pay a fixed sum of money duty of Repide, acting in good faith, to state the facts before making the
 usually used for the purpose of long term purchase of Strong’s shares. That Repide was one of the directors of
loans Philippine Sugar Estates was but one of the facts upon which liability is
 Philfinance merely borrowed the CBCI from Filriters, a sister asserted. He was not only a director, but he owned three-fourths of the
corporation. shares of its stock, and was, at the time of the purchase of the stock,
o lack of any consideration = assignment is a complete administrator general of the company with large powers and engaged in
nullity the negotiations which finally led to the sale of the company’s lands at a
price which greatly enhanced the value of the stock. He was the negotiator
 Filriters to Philfinance did not conform to the "Rules and
for the sale of the Dominican lands and was acting substantially as the
Regulations Governing Central Bank Certificates of
agent of the shareholders of Philippine Sugar Estates by reason of his
Indebtedness" (Central Bank Circular No. 769, series of
ownership of the shares in the company. Because of such ownership and
1980) under which the note was issued.
agency, no one knew as well as he does about the exact condition of the
o Published in the Official Gazette on November 19, negotiations. He was the only one who knew of the probability of the sale
1980, Section 3 thereof provides that any assignment
of the Dominican lands to the government and of the probable purchase
of registered certificates shall not be valid unless made
price. Under these circumstances, Repide employed an agent to purchase
. . . by the registered owner thereof in person or by his
the stock of Strong, concealed his own identity and his knowledge of the
representative duly authorized in writing
state of negotiations and their probable result. The concealment of his
 Alfredo O. Banaria, who signed the deed of identity while procuring the purchase of the stock, by his agent, was in
assignment purportedly for and on behalf of itself strong evidence of fraud on the part of Repide. By such means, the
Filriters, did not have the necessary written more easily was he able to avoid questions relative to the negotiations for
authorization from the BOD the sale of Dominican lands and actual misrepresentations regarding that
o Traders, being a commercial bank, cannot feign subject. He kept up the concealment as long as he could by giving the
ignorance of Central Bank Circular 769, and check of a third person Rueda Ramos, for the purchase money. This move
its requirements.
of Repide was a studied and intentional omission to be characterized as • (1) "assign to a third party assignee to be designated by Radstock all its
part of the deceitful machinations to obtain the purchase without giving rights and
any information whatever as to the state and probable result of the interests" to the listed real properties of PNCC;
negotiations and to obtain a lower price for the shares of Strong. After the • (2) issue to Radstock or its assignee common shares of the capital stock
purchase of stock, he continued negotiations for the sale of the Dominican of PNCC; and
lands as the administrator general and eventually entered into a contract of (3) assign to Radstock or its assignee 50% of PNCC’s 6% share, for the
sale. The whole transaction gives conclusive evidence of the next 27 years, in
overwhelming influence Repide had in the negotiations and it is clear that the gross toll revenues of the Manila North Tollways Corporation.
the final consummation was in his hands at all times. • Strategic Alliance Development Corporation (STRADEC) moved for
reconsideration.
OBITER DICTUM: STRADEC alleged that it has a claim against PNCC as a bidder of the
The directors are declared to be mandatories of the society and National
that they are prohibited from acquiring by purchase, even at public or Government’s shares, receivables, securities and interests in PNCC.
judicial auction, the property the administration or sale of which, may Issue
have been entrusted to them, and that this is the extent of the prohibition. Whether or not the Compromise Agreement between PNCC (Board?) and
Radstock is
Tam Wing Tak vs Ramon Makasiar valid
Business Organization – Corporation Law – Ultra Vires Acts of !
Corporate Officers – Derivative Suit • NO
Sometime before November 1992, Vic Ang Siong issued a check to • Radstock is a private corporation incorporated in the British Virgin
Concord-World Properties, Inc. The check amounted to P83.5 million. Islands. As a foreign
The check however bounced. In November 1992, Tam Wing Tak filed an corporation, with unknown owners whose nationalities are also unknown,
affidavit-complaint for violation of the Anti-Bouncing Checks Law Radstock is not
against Ang Siong. The fiscal did not file a criminal information against qualified to own land in the Philippines.
Ang Siong because apparently Concord-World and Ang Siong are settling • Radstock is also disqualified to own the rights to ownership of lands and
out of court (in fact Ang Siong already paid P19 million); and that Tam transfer rights to
Wing Tak was not authorized by the Board of Directors of Concord-World ownership of lands in the Phils. !
to sue Ang Siong. Tam Wing Tak then filed a petition for mandamus to DOCTRINE:
compel the fiscal to file the information. Judge Ramon Makasiar • In this jurisdiction, the members of the board of directors have a three-
dismissed the petition. fold duty: duty of
ISSUE: Whether or not the petition should be granted. obedience, duty of diligence, and duty of loyalty.
HELD: No. The petition for mandamus shall not lie. There was no grave • Accordingly, the members of the board of directors
abuse of discretion when the fiscal refused to file the information. • (1) shall direct the affairs of the corporation only in accordance with the
Concord-World is the named payee in the check that bounced. As payee, purposes for
Concord-World is the injured party hence only Concord-World can file the which it was organized;
criminal case against Ang Siong but it did not do so because it chose to • (2) shall not willfully and knowingly vote for or assent to patently
amicably settle the issue with Ang Siong. Where a corporation is an unlawful acts of the
injured party, its power to sue is lodged with its board of directors or corporation or act in bad faith or with gross negligence in directing
trustees. This can be delegated but Tam Wing Tak never proved that he the affairs of the corporation; and
was authorized by the Board of Concord-World. •(3) shall not acquire any personal or pecuniary interest in conflict with
But may the suit be considered a derivative suit where the Board’s their duty as such directors or trustees.
authorization may not be had?
No. For a derivative suit to prosper, it is required that the minority G.R. No. L-30460
stockholder suing for and on behalf of the corporation must allege in his March 12, 1929
complaint that he is suing on a derivative cause of action on behalf of the Steinberg v. Velasco
corporation and all other stockholders similarly situated who may wish to
join him in the suit. In this case, this was not complied with. Hence, Tam Nature: Petition for review of lower court decision to dismiss P’s
Wing Tak cannot sue Ang Siong. complaint since it held that the Board of Driectors could legally declare
dividend of P3000
Strategic Alliance vs. Radstock Securities ! Facts:
Steinberg (P) is the receiver of Sibuguey Trading Corp. P alleges that
• Construction Development Corporation of the Philippines (CDCP) was [Velasco, as president, Del Castillo, as vice-president, Navallo, as
incorporated in secretary-treasurer, and Manuel, as director of the trading Company][R],
1966. It was granted a franchise to construct, operate and maintain toll at a meeting of the board of directors held on July 24, 1922, approved and
facilities in the North authorized various lawful purchases already made of a large portion of the
and South Luzon Tollways and Metro Manila Expressway. capital stock of the company from its various stockholders, thereby
• CDCP Mining Corporation, an affiliate of CDCP, obtained loans from diverting its funds to the injury of the creditors of the corp. At the time of
Marubeni such purchase, the corporation had debts amounting to P13,807.50, most
Corporation of Japan. A CDCP official issued letters of guarantee for the of which were unpaid at the time petition for the dissolution of the
loans although corporation was financial condition, in contemplation of an insolvency and
there was no CDCP Board Resolution authorizing the issuance of such dissolution. P also alleges that R approved a resolution for the payment of
letters of guarantee. ! P3,000 as dividends to its stockholders, which was done in bad faith, and
• CDCP Mining secured the Marubeni loans when CDCP and CDCP to the injury and fraud of its creditors since it had accounts less in amount
Mining were still than the accounts receivable. R argues that the distribution of dividends
privately owned and managed. was authorized by the board of directors and they constitute surplus profit
• Later on, CDCP’s name was changed to Philippine National of the corp. The lower court found out that R authorized the purchase of,
Construction Corporation purchased and paid for, 330 shares of the capital stock of the corporation
(PNCC) in order to reflect that the Government already owned 90.3% of at the agreed price of P3,300, and that at the time the purchase was made,
PNCC and only the corporation was indebted in the sum of P13,807.50, and that according
9.70% is under private ownership. ! to its books, it had accounts receivable in the sum of P19,126.02.
• Meanwhile, the Marubeni loans to CDCP Mining remained unpaid. Issue:
• PNCC Board passed Board Resolutions admitting PNCC’s liability to Whether R can declare dividends.
Marubeni. Ruling:
Previously, for two decades the PNCC Board consistently refused to admit No. The action of the board in purchasing the stock from the corporation
any liability for and in declaring the dividends on the stock was all done at the same
the Marubeni loans. meeting of the board of directors. The directors were permitted to resign
• Later, Marubeni assigned its entire credit to Radstock Securities Limited so that they could sell their stock to the corporation. The authorized capital
(Radstock), a stock was P20,000 divided into 2,000 shares of the par value of P10 each,
foreign corporation. Radstock immediately sent a notice and demand letter which only P10,030 was subscribed and paid. Deducting the P3,300 paid
to PNCC. for the purchase of the stock, there would be left P7,000 of paid up stock,
• PNCC and Radstock entered into a Compromise Agreement. Under this from which deduct P3,000 paid in dividends, there would be left P4,000
agreement, only. R acted on assumption that it appeared from the books of the
PNCC shall pay Radstock the reduced amount of P6,185,000,000.00 from corporation that it had accounts receivable of the face value of P19,126.02,
P17,040,843,968.00 ! therefore it had a surplus over and above its debts and liabilities. However,
• To satisfy its reduced obligation, PNCC undertakes to there is no stipulation as to the actual cash value of those accounts, and it
does appear from the stipulation that, P12,512.47 of those accounts had
but little value. The corporation did not then have an actual bona fide compliance.[18]
surplus from which the dividends could be paid, and that the payment of
them in full at the time would affect the financial condition of the In the case at bar, the Court holds that there has been substantial
corporation. Because of this, the directors did not act in good faith or that compliance with Sections 4 and 5, Rule 7 of the 1997 Revised Rules on
they were grossly ignorant of their duties. Creditors of a corporation have Civil Procedure on the petitioners’ part in consonance with our ruling in
the right to assume that so long as there are outstanding debts and the Lepanto Consolidated Mining Company v. WMC Resources
liabilities, the board of directors will not use the assets of the corporation International PTY LTD.[19] that we laid down in 2003 with the rationale
to purchase its own stock, and that it will not declare dividends to that the President of petitioner-corporation is in a position to verify the
stockholders when the corporation is insolvent. truthfulness and correctness of the allegations in the petition. Petitioner
Pleadings; powers of corporate officers. Benzonan clearly satisfies the aforementioned jurisprudential requirement
because he is the President of petitioner South Cotabato Communications
G.R. No. 173326 Corporation. Moreover, he is also named as co-respondent of petitioner-
SOUTH COTABATO COMMUNICATIONS CORPORATION and corporation in the labor case which is the subject matter of the special civil
GAUVAIN J. BENZONAN vs. HON. PATRICIA A. STO. TOMAS, action for certiorari filed in the Court of Appeals.
SECRETARY OF LABOR AND EMPLOYMENT, et. al., G.R. No.
173326, December 15, 2010 Clearly, it was error on the part of the Court of Appeals to dismiss
petitioners’ special civil action for certiorari despite substantial
DECISION compliance with the rules on procedure. For unduly upholding
LEONARDO-DE CASTRO, J.: technicalities at the expense of a just resolution of the case, normal
x x x. procedure dictates that the Court of Appeals should be tasked with
properly disposing the petition, a second time around, on the merits.
Anent the first procedural issue, the Court had summarized the
jurisprudential principles on the matter in Cagayan Valley Drug The Court is mindful of previous rulings which instructs us that when
Corporation v. Commissioner of Internal Revenue.[15] In said case, we there is enough basis on which a proper evaluation of the merits can be
held that a President of a corporation, among other enumerated corporate made, we may dispense with the time-consuming procedure in order to
officers and employees, can sign the verification and certification against prevent further delays in the disposition of the case.[20] However, based
of non-forum shopping in behalf of the said corporation without the on the nature of the two remaining issues propounded before the Court
benefit of a board resolution. We quote the pertinent portion of the which involve factual issues and given the inadequacy of the records,
decision here: pleadings, and other evidence available before us to properly resolve those
questions, we are constrained to refrain from passing upon them.
It must be borne in mind that Sec. 23, in relation to Sec. 25 of the
Corporation Code, clearly enunciates that all corporate powers are After all, the Court has stressed that its jurisdiction in a petition for review
exercised, all business conducted, and all properties controlled by the on certiorari under Rule 45 of the Rules of Court is limited to reviewing
board of directors. A corporation has a separate and distinct personality only errors of law, not of fact, unless the findings of fact complained of
from its directors and officers and can only exercise its corporate powers are devoid of support by the evidence on record, or the assailed judgment
through the board of directors. Thus, it is clear that an individual corporate is based on the misapprehension of facts.[21]
officer cannot solely exercise any corporate power pertaining to the
corporation without authority from the board of directors. This has been WHEREFORE, the petition is PARTIALLY GRANTED. The assailed
our constant holding in cases instituted by a corporation. Resolutions of the Court of Appeals are REVERSED and SET ASIDE.
The case is REMANDED to the Court of Appeals for proper disposition of
In a slew of cases, however, we have recognized the authority of some CA-G.R. SP No. 00179-MIN.
corporate officers to sign the verification and certification against forum
shopping. In Mactan-Cebu International Airport Authority v. CA, we SO ORDERED.
recognized the authority of a general manager or acting general manager
to sign the verification and certificate against forum shopping; in Pfizer v. SOLER
Galan, we upheld the validity of a verification signed by an “employment VS.
specialist” who had not even presented any proof of her authority to COURT OF APPEALS
represent the company; in Novelty Philippines, Inc. v. CA, we ruled that a G.R. No. 123892
personnel officer who signed the petition but did not attach the authority
from the company is authorized to sign the verification and non-forum FACTS OF THE CASE
shopping certificate; and in Lepanto Consolidated Mining Company v. Petitioner Jazmin Soler is a well-known licensed professional interior
WMC Resources International Pty. Ltd. (Lepanto), we ruled that the designer. Her friend Rosario Pardo asked her to talk to Nida Lopez, who
Chairperson of the Board and President of the Company can sign the was manager of the COMBANK Ermita Branch for they were planning to
verification and certificate against non-forum shopping even without the renovate the branch offices. During their meeting, petitioner was hesitant
submission of the board’s authorization. to accept the job because of her many out of town commitments, and also
considering that Ms. Lopez was asking that the designs be submitted by
In sum, we have held that the following officials or employees of the December 1986, which was such a short notice. Ms. Lopez insisted,
company can sign the verification and certification without need of a however, because she really wanted petitioner to do the design for
board resolution: (1) the Chairperson of the Board of Directors, (2) the renovation. Petitioner acceded to the request. Petitioner even told Ms.
President of a corporation, (3) the General Manager or Acting General Lopez that her professional fee was P10, 000.00, to which Ms. Lopez
Manager, (4) Personnel Officer, and (5) an Employment Specialist in a acceded.
labor case. After a few days, petitioner requested for the blueprint of the building so
that the proper design, plans and specifications could be given to Ms.
While the above cases do not provide a complete listing of authorized Lopez in time for the board meeting in December 1986. So come
signatories to the verification and certification required by the rules, the December 1986, the lay out and the design were submitted to Ms. Lopez.
determination of the sufficiency of the authority was done on a case to She even told petitioner that she liked the designs.
case basis. The rationale applied in the foregoing cases is to justify the Subsequently, petitioner repeatedly demanded payment for her services
authority of corporate officers or representatives of the corporation to sign but Ms. Lopez just ignored the demands. To settle the controversy,
the verification or certificate against forum shopping, being “in a position petitioner referred the matter to her lawyers, who wrote Ms. Lopez,
to verify the truthfulness and correctness of the allegations in the demanding payment for her professional fees in the amount of P10, 000.00
petition.”[16] (Emphases supplied.) which Ms. Lopez ignored. Hence, the lawyers wrote Ms. Lopez once
again demanding the return of the blueprint copies petitioner submitted
It must be stressed, however, that the Cagayan ruling qualified that the which Ms. Lopez refused to return.
better procedure is still to append a board resolution to the complaint or Petitioner filed at the RTC Pasig, a complaint against COMBANK and
petition to obviate questions regarding the authority of the signatory of the Ms. Lopez for collection of professional fees and damages.
verification and certification.[17] In its answer, COMBANK stated that there was no contract between
COMBANK and petitioner; that Ms. Lopez merely invited petitioner to
Nonetheless, under the circumstances of this case, it bears reiterating that participate in a bid for the renovation of the COMBANK Ermita Branch;
the requirement of the certification of non-forum shopping is rooted in the that any proposal was still subject to the approval of the COMBANK’s
principle that a party-litigant shall not be allowed to pursue simultaneous head office.
remedies in different fora, as this practice is detrimental to an orderly RTC ruled in favor of Soler. The Court of Appeals reversed the decision.
judicial procedure. However, the Court has relaxed, under justifiable ISSUE
circumstances, the rule requiring the submission of such certification Whether Nida Lopez, the manager of the bank branch, had authority to
considering that, although it is obligatory, it is not jurisdictional. Not being bind the bank in the transaction.
jurisdictional, it can be relaxed under the rule of substantial RULING
Yes.
It is familiar doctrine that if a corporation knowingly permits one of its confidence in him and did not mind disclosing their plans to him,
officers, or any other agent, to act within the scope of an apparent concerning the purchase of the aforesaid estate and the progress of their
authority, it 222 holds him out to the public as possessing the power to do negotiations with Maria Gay.
those acts; and thus, the corporation will, as against anyone who has in It also sufficiently established in the records that in one of the conferences
good faith dealt with it through such agent, be estopped from denying the held by the plaintiffs among themselves, relative to the purchase of the
agent’s authority. aforesaid estate, at which the defendant was present, the latter remarked
Petitioner believed that once she submitted the designs she would be paid that it would be advisable to let some days elapse before accepting the
her professional fees. Ms. Lopez assured petitioner that she would be paid. terms of the transfer as proposed by Maria Gay, in order that the latter
The discussion between petitioner and Ms. Lopez was to the effect that might not think that they were coveting said property. This mere remark
she had authority to engage the services of petitioner. During their along in itself cannot be taken to mean any wrongful intent on the part of
meeting, she even gave petitioner specifications as to what was to be said defendant, but it ceases to be innocent when taken in connection with
renovated in the branch premises and when petitioners requested for the the fact, also proven, that when the defendant met Alipio de los Santos
blueprints of the building, Ms. Lopez supplied the same. after the latter's return to Iloilo, sent by the plaintiffs to examine the estate
Ms. Lopez even insisted that the designs be rushed in time for presentation and satisfy himself of its condition, and Alipio de los Santos told him of
to the bank. With all these discussion and transactions, it was apparent to his favorable impression of the estate, he advised De los Santos not to
petitioner that Ms. Lopez indeed had authority to engage the services of report the estate to the plaintiffs as being so highly valuable, for if it
petitioner. proved failure they might blame him, De los Santos. One becomes more
strongly convinced that this defendant has been unfaithful to his
Smith v. Van Gorkom principals, the plaintiffs, when these circumstances are considered in
Citation. Smith v. Van Gorkom, 488 A.2d 858, 1985 Del. LEXIS 421, connection with the fact at an early hour in the morning of June 17, 1919,
46 A.L.R.4th 821, Fed. Sec. L. Rep. (CCH) P91,921 (Del. Jan. 29, only the midday of which the term of plaintiff's when these circumstances
1985) are considered in connection with the fact that at an early hour in the
morning of June 17, 1919, on the midday of which the term of plaintiff's
Brief Fact Summary. Plaintiffs, Alden Smith and John Gosselin, brought option to purchase was to expire, said defendant Antonio Sunyantong
a class action suit against Defendant corporation, Trans Union, and its called at the house of Mari Gay when she was having breakfast, and
directors, after the Board approved a merger proposal submitted by the offered to buy the estate on the same terms proposed by her not yet
CEO of Trans Union, fellow Defendant Jerome Van Gorkom. accepted by the plaintiffs, making the offer to buy not for the benefit of
the plaintiff's, but for own wife, his codefendant Vicenta Llorente de
Synopsis of Rule of Law. Under the business judgment rule, a business Sunyatong. In view of the opportunity that offered itself, but respecting
judgment is presumed to be an informed judgment, but the judgment will the option granted the plaintiffs, Maria Gay communicated by telephone
not be shielded under the rule if the decision was unadvised. with Manuel Sotelo, who was communicated by telephone with Manuel
Sotelo, who was acting as broker for the plaintiffs in these transactions,
Facts. Trans Union had large investment tax credits (ITCs) coupled with and told him that another buyer of the estate had presented himself who
accelerated depreciation deductions with no offsetting taxable income. would accept the terms proposed by her and that she would like to know
Their short term solution was to acquire companies that would offset the immediately what decision had been reached by the plaintiffs on the
ITCs, but the Chief Financial Officer, Donald Romans, suggested that matter. In view of Maria Gay's insistence that the plaintiff give a
Trans Union should undergo a leveraged buyout to an entity that could categorical answer, Sing Bengco, one of the plaintiffs who happened to be
offset the ITCs. The suggestion came without any substantial research, but present at the time the telephone conversation between Maria Gay and
Romans thought that a $50-60 share price (on stock currently valued at a Manuel Sotelo took place, instructed Sotelo to inform her at the time that
high of $39 ½) would be acceptable. Van Gorkom did not demonstrate any if she did not care to wait until 12 o'clock, "ella cuidado": (she could do as
interest in the suggestion, but shortly thereafter pursued the idea with a she pleased). This is a purely Philippine phrase, an exact translation of the
takeover specialist, Jay Pritzker. With only Romans’ unresearched Tagalog, "siya ang bahala" and approximately of the Visayan "ambut sa
numbers at his disposal, Van Gorkom set up an agreement with Pritzker to iya," which has very different, and even contradictory, meanings. It might
sell Pritzker Trans Union shares at $55 per share. Van Gorkom also agreed be interpreted in several different ways, such as a threat on the part of Sing
to sell Pritzker one million shares of Trans Union at $39 per share if Bengco to take legal action against Maria Gay in case she did not wait
Pritzker was outbid. Van Gorkom also agreed not to solicit other bids and until the expiration of the option, or that they would waive all claims to
agreed not to provide proprietary information to other bidders. Van the option and be agreeable to whatever action she might take. Interpreting
Gorkom only included a couple people in the negotiations with Pritzker, the phrase to mean that the plaintiffs waived their option to buy, Maria
and most of the senior management and the Board of Directors found out Gay closed the sale of the estate in favor of the defendant Antonio
about the deal on the day they had to vote to approve the deal. Van Sunyantong.
Gorkom did not distribute any information at the voting, so the Board had Even supposing that this latter interpretation of the phrase in question was
only the word of Van Gorkom, the word of the President of Trans Union actual intention of Sing Bengco, the action of the defendant Sunyantong in
(who was privy to the earlier discussions with Pritzker), advice from an intervening in the negotiations in the manner in which he did does not
attorney who suggested that the Board might be sued if they voted against make him innocent of infidelity in view of the fact that he was an
the merger, and vague advice from Romans who told them that the $55 employee of the plaintiffs to whom he owed loyalty and faithfullnes.
was in the beginning end of the range he calculated. Van Gorkom did not Even though it be concede that when he closed the contract of sale with
disclose how he came to the $55 amount. On this advice, the Board Maria Gay the plaintiffs' option had expired, but the fact cannot be denied
approved the merger, and it was also later approved by shareholders. that he was the cause of the option having precipitously come to such an
Held. The Delaware Supreme Court held the business judgment to be end. His disloyalty to his employers was responsible for Maria Gay not
gross negligence, which is the standard for determining whether the accepting the terms proposed by the plaintiffs, because of being certain of
judgment was informed. The Board has a duty to give an informed another less exigent buyer. Without such intervention on the part of the
decision on an important decision such as a merger and can not escape the defendant it is presumed, taking into account all the circumstances of the
responsibility by claiming that the shareholders also approved the merger. case, that the sale of the estate in question would have been consummated
The directors are protected if they relied in good faith on reports submitted between Maria Gay and the plaintiffs, perhaps with such advantages to the
by officers, but there was no report that would qualify as a report under plaintiffs, as they expected to obtain by prolonging negotiations.
the statute. The directors can not rely upon the share price as it contrasted Such an act of infidelity committed by a trusted employee calculated to
with the market value. And because the Board did not disclose a lack of redound to his own benefit and to the detriment of his employers cannot
valuation information to the shareholders, the Board breached their pass without legal sanction. Nemo debet aliena jactura locupletari; nemo
fiduciary duty to disclose all germane facts. ex suo delicto meliorem suam conditionem facere potest. It is an illicit act
committed with culpa and, therefore, its agent is liable (art. 1089, Civil
G.R. No. L-17131 June 30, 1922 Code), for the damage caused (art. 1902, ibidem). Not identical, but
SING JUCO and SING BENGCO, plaintiffs-appellees, similar, to this infidelity is the abuse of the confidence sanctioned in our
vs. Penal Code as a generic circumstances, nay as specific aggravating one,
ANTONIO SUNYANTONG and his wife VICENTA LLORENTE DE and even as an essential element of certain crimes.
SUNYANTONG, defendants-appellants. This reparation provided for in the Civil Code and applied to the case at
Montinola, Montinola and Hontiveros for appellants. bar seems to be limited to the indemnification of damages, as we are not
Fisher and De Witt for appellees. aware of any express provision in said Code which imposes upon the
ROMUALDEZ, J.: person thus held liable, any obligation, such as that of transferring to
On May 20, 1919, the plaintiffs obtained from Maria Gay a written option plaintiffs the estate in question.
to purchase an estate known as "San Antonio Estate," containing more Such principle, however, in case of this nature is generally recognized in
than 2,000 hectares situated in the municipality of Passi, Province of our laws, since the case of commercial agents ( factories) it is expressly
Iloilo, together with the large cattle existing on said estate. The term of the established. Undoubtedly, formerly under the circumstances then
option expired, but the plaintiffs had it extended verbally until 12 o 'clock prevailing such sanction was not necessary in the field of civil law,
noon of June 17, 1919. because is sphere of action is the general relations of society; but event
The defendant Antonio Sunyantong was at the time an employee of the then it was deemed necessary expressly to protect with such sanction the
plaintiffs, and the preponderance of evidence shows that they reposed commercial relations wherein the question of gain was involved, which is
sometimes so imperatives as to ignore everything, even the very principles Development Authority under RA No. 7227; (3) Plaintiff’s cause of action
of loyalty, honesty, and fidelity. is barred by prescription; (4) twenty-five years having lapsed since the
This specific relief, however, has already come to be applied in this issuance of the writ of execution, no action for revival of judgment may be
jurisdiction in similar cases, among which can be cited that of Camacho instituted because under Paragraph 3 of Article 1144 of the Civil Code,
vs. Municipality of Baliuag (28 Phil., 466.) such action may be brought only within ten (10) years from the time the
And in the North American law such sanction is expressly recognized, and judgement had been rendered.
the transaction of this nature might be regarded as an "equitable trust" by On August 31, 1999, the trial court denied petitioner’s motion to dismiss
virtue of which the things acquired by an employee is deemed not to have and on October 14, 1999, its motion for reconsideration was likewise
been acquired for his own benefit or that of any other person but for his turned down.
principal, and held in trust for the latter (21 R. C. L., 825; 2 Corpus Juris, On October 21, 1999, petitioner instituted a petition for certiorari and
353). prohibition with the Court of Appeals, docketed therein as CA-G.R. SP
After examination and consideration of the case we do not find in the No. 55535, on the ground that the orders of the trial court denying its
appealed judgment any of errors assigned to it; wherefore the same is motion to dismiss and its subsequent motion for reconsideration were
affirmed with costs against the appellants. So ordered. issued in excess of jurisdiction.
Araullo, C.J., Malcolm, Avanceña and Ostrand, JJ., concur. On Novemeber 4, 1999, the court of Appeals dismissed the petition in CA-
G.R. SP No. 55535 on the ground that the verification and certification in
SHIPSIDE INCORPORATED, petitioner, vs. THE HON. COURT the petition, under the signature of Lorenzo Balbin, Jr., was made without
OF APPEALS [Special Former Twelfth Division], HON. REGIONAL authority, there being no proof therein that Balbin was authorized to
TRIAL COURT, BRANCH 26 (San Fernando City, La Union) & institute the petition for and in behalf and of petitioner.
THE REPUBLIC OF THE PHILIPPINES, respondents. On May 23, 2000, the Court of Appeals denied petitioner’s motion for
G.R. No. 143377, February 20, 2001 reconsideration on the grounds that: (1) a complaint filed on behalf of a
corporation can be made only if authorized by its Board of Directors, and
Facts: in the absence thereof, the petition cannot prosper and be granted due
On October 29, 1958, Original Certificate No. 0-381 was issued in favour course; and (2) the petitioner was unable to show that it had substantially
of Rafael Galvez, over four parcels of land. complied with the rule requiring proof of authority to institute an action or
On April 11, 1960, Lots No. 1 and 4 were sold by Rafael Galvez to proceeding.
Filipina Mamaril, Cleopatra Llana, Regina Bustos, and Erlinda Balatbat, In support of its petition, Shipside, Inc. asseverates that:
with deed of sale inscribed as entry no. 9115OCT 0-381 on August 1. The honourable Court of Appeals gravely abused its discretion
10,1960. Consequently, Transfer Certificate No. T-4304 was issued in in dismissing the petition when it made a conclusive legal
favour of the buyers covering Lots No. 1 and 4. presumption that Mr. Balbin had no authority to sign the petition
On August 16, 1960, Mamaril, et al. sold Lots No. 1 and 4 to Lepanto despite the clarity of laws, jurisprudence and Secretary’
Consolidated Mining Company. The deed of sale covering the aforesaid certificate to the contrary.
property was inscribed as Entry No. 9173 on TCT No. T-4304. 2. The honourable Court of Appeals abused its discretion when it
Subsequently, Transfer Certificate No. T-4314 was issued in the name of dismissed the petition, in effect affirming the grave abuse of
Lepanto Consolidated Mining Company as owner of Lots 1 and 4. discretion committed by the lower court, when it refused to
On February 1, 1963, unknown to Lepanto Consolidated Mining dismiss the 1999 Complaint for Revival of a 1973 judgment, in
Company, the Court of First Instance of La Union, Second Judicial violation of clear laws and jurisprudence.
District, issued an order in Land registration Case No. N-361 entitled
“Rafael Galvez, Applicant, Eliza Bustos, et al., Parties-In-Interest; Issues:
Republic of the Philippines, Movant” declaring OCT No. 0-381 of the (1) Whether an authorization from petitioner’s Board of Directors is
Registry of Deeds for the Province of La Union issued in the name of still required in order for its resident manager to institute or
Rafel Galvez, null and void, and ordered the cancellation thereof. commence a legal action for and in behalf of the corporation;
On October 28, 1963, Lepanto Consolidated Mining Company sold to the (2) Whether the instant petition should be allowed; and
petitioner Lots No. 1 and 4, with the deed being entered in TCT No. 4314 (3) Whether the republic of the Philippines can maintain action for
as entry No. 12381. Transfer Certificate of Title No T-5710 was thus revival of judgment therein.
issued in favour of the petitioner which starting since then exercised
proprietary rights over Lots No. 1 and 4. Held:
In the meantime, Rafael Galvez filed his motion for reconsideration (1) Yes. The court of Appeals dismissed the petition for certiorari
against the order by the trial court declaring OCT No. 0-381 null and void. on the ground that Lorenzo Balbin, the resident manager for
The motion was denied on January 25, 1965. On appeal, the court of petitioner, who was the signatory in the verification and
Appeals ruled in favor of the Republic of the Philippines in a resolution certification on non-forum shopping, failed to show proof that
promulgated on August 14, 1973 in CA-G.R. No. 36061`-R. he was authorized by petitioner’s board of directors to file such
Thereafter, the court of Appeals, issued an Entry of judgement, certifying a petition.
that its decision dated August 14, 1973 became final and executor on
October 23, 1973. A corporation, such as petitioner, has no power except those expressly
On April 22, 1974, the trial court in L.R.C. Case No. N-361 is sued a writ conferred on it by the Corporation Code and those that are implied or
of execution of the judgement which was served on the Register of Deeds, incidental to its existence. In turn, a corporation exercises said powers
San Fernando, La Union on April 29, 1974 through its board of directors and /or its duly authorized officers and
On January 14, 1999, the office of the Solicitor General received a letter agents. Thus, it has been observed that the power of a corporation to sue
dated January 11, 1999, from Mr. Victor Floresca, Vice-President, John and be sued in any court is lodged with the board of directors that
Hay Poro Point Development Corporation, stating that the aforementioned exercises its corporate powers. In turn, physical acts of the corporation,
orders and decision of the trial court in L.R.C. No. N-361 have not been like the signing of documents, can be performed only by natural persons
executed by the Register of Deeds, San Fernando, La Union despite duly authorized for the purpose by the corporate by-laws or by a specific
receipt of the writ of execution. act of the board of directors to file said petition.
On April 21, 1999, the Office of the Solicitor General filed a complaint for
the revival of judgment and cancellation of titles before the Regional Trial On October 21, 1999, when Balbin filed the petition, there was no proof
Court of the First judicial Region (Branch 26, San Fernando, La Union) attached thereto that Balbin was authorized to sign the verification and
docketed therein as Civil Case No., 6346 entitled, “Republic of the non-forum shopping certification therein. As a consequence, the petition
Philippines, Plaintiff, vs. Heirs of Rafael Galvez, represented by Teresita was dismissed by the Court of Appeals. However, subsequent to such
Tan, Reynaldo Mamaril, Elisa Bustos, Erlinda Balatbat, Regina Bustos, dismissal, petitioner filed a motion for reconsideration, attaching to said
Shipside Incorporated and the Register of Deeds of La Union, motion a certificate issued by its board secretary stating that on October
defendants.” 11, 1999, or ten days prior to the filing of the petition, Balbin had been
In its complaint in Civil Case No. 6346, the Solicitor General argued that authorized by petitioner’s board of directors to file said petition.
since the trial court in LRC Case no. 361 had ruled and declared OCT No.
0381 to be null and void, which ruling was subsequently affirmed by the Verification is simply intended to secure an assurance that the allegations
court of appeals, the defendants-successors-in-interest of Rafael Galvez in the pleading are true and correct and not the product of the imagination
have no valid title over the property covers by OCT No. 0-381, and the or a matter of speculation, and that the pleading is filed in good faith. The
subsequent Torrens titles issued in their names should be consequently court may order the correction of the pleading if verification is lacking or
cancelled. act on the pleading although it is not verified, if the attending
On July 22, 1999, petitioner Shipside, Inc. Filed its Motion to Dismiss, circumstances are such that strict compliance with the rules may be
based on the following grounds: (1) the complaint stated no cause of dispensed with in order that the ends of justice may thereby be served.
action because only final and executor judgements may be subject of an
action for revival for judgment; (2) the plaintiff is not the real party-in- On the other hand, the lack of certification against forum shopping is
interest because the real property covered by the Torrens titles sought to generally incurable by the submission thereof after filing of the petition.
be cancelled, allegedly part of Camp Wallace (Wallace Air Station), were Section 5, Rule 45 of the 1997 Rules of Civil Procedure provides that the
under the ownership and administration of the Bases Conversion failure of the petitioner to submit the required documents that should
accompany the petition, including the certification against forum Represented by the Department of Promulgated:
shopping, shall be sufficient ground for the dismissal thereof. The same Education, Culture and Sports,
rule applies to certifications against forum shopping signed by a person on Respondent. October 9, 2009
behalf of a corporation which are unaccompanied by proof that said
signatory is authorized to file a petition on behalf of the corporation. x ---------------------------------------------------------------------------------------
-x
(2) Yes. In the instant case, the merits of the petitioner’s case should
be considered special circumstances or compelling reasons that DECISION
justify tempering the requirement in regard to the certificate of
non-forum shopping. With more reason should the instant ABAD, J.:
petition be allowed since the petitioner did submit a certification
on non-forum shopping, failing only to show proof that the
signatory was authorized to do so. That petitioner subsequently This petition for review on certiorari assails the February 21, 2006
submitted a secretary’s certificate attesting that Balbin was Decision[1] of the Court of Appeals in CA-G.R. CV 83648 and its
authorized to file an action. Resolution[2] of May 29, 2006, which dismissed the petitioners appeal
from the decision of Branch 71 of the Regional Trial Court (RTC) of Pasig
It must also be kept in mind that while the requirement of the certificate of City in Civil Case 66852.
non-forum shopping is mandatory, nonetheless the requirements should
not be interpreted literally and thus defeat the objective of preventing the The Facts and the Case
undesirable practice of forum- shopping. Lastly, technical rules of
procedure should be used to promote, not frustrate justice. While the swift In 1980, during the regime of President Ferdinand E. Marcos, the
unclogging of court dockets is a laudable objective, the granting of government-owned Human Settlements Development Corporation
substantial justice is an even more urgent ideal. (HSDC) built with public funds and on government land the St. Martin
Technical Institute Complex at Barangay Ugong, Pasig City. This later on
(3) No. The action instituted by the Solicitor General in the trial became known as the University of Life Complex.
court is one for revival of judgment which is governed by
Article 1144 (3) of the Civil Code and Section 6, Rule 39 of the In July 1980, First Lady Imelda R. Marcos and others organized
1997 Rules on Civil Procedure. Article1144 (3) provides that an the University of Life Foundation, Inc. (ULFI), a private non-stock, non-
action upon a judgement “must be brought within 10 years from profit corporation devoted to non-formal education. On August 26,
the time the right of action accrues.” On the other hand, Section 1980 the government gave the management and operation of the Complex
6, Rule 39 provides that a final and executor judgment or order to ULFI but HSDC was to continue to construct facilities and acquire
may be executed on motion within five (5) years from the date equipment for it. Although ULFI was to get all the incomes of the
of its entry, but that after the lapse of such time, and before it is Complex, ULFI had to pay HSDC an annual fee of 14 percent of HSDCs
barred by the statute of limitations, a judgement may be investments in it.
enforced by action. Taking those two provisions into
consideration, it is plain that an action for revival of judgment After the fall of the Marcos regime in 1986, the new government
must be brought within ten years from the time said judgment reorganized HSDC into the Strategic Investment Development
becomes final. Corporation (SIDCOR) under the supervision of the Office of the
President. Realizing that ULFI never paid the 14 percent annual fee due to
From the records of the case, it is clear that the judgment sought to be HSDC, now totaling about P316 million, on July 25, 1989 SIDCOR
revived became final on October 23, 1973. On the other hand, the action rescinded the HSDC-ULFI agreement. Ironically, in its place, SIDCOR
for revival of judgment was instituted only in 1999, or more than 25 entered into an Interim Management Agreement with ULFI, allowing it to
uyears after the judgment had become final. Hence, the action is abarred continue managing and operating the Complex.
by extinctive prescription considering that such an action can be instituted
only within ten (10) years from the time the cause of action accrues. Meantime, in October 1989, the government transferred the ownership of
ULFIs properties to the Department of Education, Culture and Sports
The Solicitor-general’s contention that the state’s cause of action in the (DECS). Later in January 1990, Republic Act 6847 transferred full control
cancellation of the land title issued to petitioner’s predecessor-in-interest and management of the Complex to DECS with effect two years from the
is imprescriptible because it is included in Camp Wallace, which belong to laws enactment. The DECS transferred its offices to the Complex in
the government, is misleading. While it is true that the prescription does December 1990. On January 29, 1991, SIDCOR transferred all its rights in
not run against the State, the same may not be invoked by the government the Complex to the National Government which in turn transferred the
in this case since it is no longer interested in the subject matter. While same to the DECS.
Camp Wallace may have belonged to the Government at the time Rafael
Galvez’s title was ordered cancelled in Land Registration Case no N-361, On January 31, 1991 DECS and ULFI entered into a Management
the same no longer holds true today. Agreement, granting ULFI the authority to manage and operate the
Complex until the end of that year.During this period, ULFI was expressly
With the transfer of Camp Wallace to the BCDA, the government no mandated under the said Management Agreement to remit to the Bureau
longer has a right or interest to protect. Consequently, the republic is not a of the Treasury, through the DECS, all incomes from the Complex, net of
real party in interest and it may not institute the instant action. Nor may it allowable expenses.[3] At the end of 1991, the DECS gave ULFI notice to
raise the defense of imprescriptibility the same being applicable only in immediately vacate the Complex. But ULFI declined, prompting the
cases where the government is a party in interest. Under section 2 of Rule DECS to file an action for unlawful detainer against it in Civil Case 2959
3 of the 1997 Rules of Civil procedure, “every action must be prosecuted of the Metropolitan Trial Court (MeTC) of Pasig City. After hearing,
or defined in the name of the real party in interest.” And to qualify a MeTC dismissed the action for lack of merit. On the DECSs appeal to the
person to be a real party in interest whose name in action must be RTC, the latter affirmed the order of dismissal.
prosecuted, he must appear to be the present real owner of the right sought
to be enforced (Pioneer Insurance v. CA, 175 SCRA 668 [1989]). A real On appeal of the DECS to the Court of Appeals by petition for
party in interest is the party who stands to be benefitted or injured by the review,[4] however, the latter rendered judgment on January 17, 1995,
judgment in the suit, or the party entitled to the avails of the suit. And by reversing the MeTC and RTC decisions. The appeals court ordered ULFI
real interest is meant a present substantial interest, as distinguished from a to vacate the Complex and pay such reasonable rentals as the MeTC might
mere expectancy, or a future, contingent, subordinate or consequential fix. This Court dismissed ULFIs recourse to it from the judgment of the
interest (Ibonilla v. Province of Cebu, 210 SCRA 526 [1992]). Being the Court of Appeals.[5]
owner of the areas covered by Camp Wallace, it is the Bases Conversion
and Development Authority, not the Government, which stands to be On April 15, 1996 the MeTC fixed, after hearing, the rents that ULFI had
benefited if the land covered by TCT No. T5710 issued in the name of to pay the DECS at P22,559,215.14 (due from February 1992 to January
petitioner is cancelled. 1996) plus P6,325.00 per month until it shall have vacated the
premises.[6] The DECS succeeded in ejecting ULFI but the latter did not
MANUEL LUIS S. SANCHEZ, G.R. No. 172885 pay the amounts due from it.
Petitioner,
Present: On June 15, 1998 the DECS filed a complaint[7] before the RTC of Pasig
Corona, J.,* City in Civil Case 66852 for collection of the P22,559,215.14 in
- versus - Carpio Morales,** unremitted rents and damages against Henri Kahn, ULFIs President, and
Acting Chairperson, petitioner Manuel Luis S. Sanchez, its Executive Vice-President, based on
Chico-Nazario,*** their personal liability under Section 31 of the Corporation Code. The
Brion, and latter two were Managing Director and Finance Director, respectively, of
Abad, JJ. the corporation.[8]
REPUBLIC OF THE PHILIPPINES,
The complaint alleged that Kahn and petitioner Sanchez, as key ULFI directing the affairs of the corporation or acquire any personal or
officers, were remiss in safekeeping ULFIs corporate incomes and in pecuniary interest in conflict with their duty as such directors or
accounting for them.[9] They neither placed the incomes derived from the trustees shall be liable jointly and severally for all damages resulting
Complex in ULFIs deposit account nor submitted the required financial therefrom suffered by the corporation, its stockholders or members
statements detailing their transactions. The underlying theory of the case is and other persons. (Emphasis supplied)
that Kahn and Sanchez operated ULFI as if it were their own property,
handled the collections and spent the money as if it were their personal xxxx
belonging.[10] The DECS asked the RTC to order Kahn and Sanchez
personally to pay it the P22,559,215.14 in rents due from ULFI with legal
interest, exemplary damages of P1,000,000.00, attorneys fees The DECS does not have to invoke the doctrine of piercing the veil of
of P500,000.00, and costs. corporate fiction. Section 31 above expressly lays down petitioner
Sanchez and Kahns liability for damages arising from their gross
In his answer, petitioner Sanchez alleged that, being a mere officer of negligence or bad faith in directing corporate affairs. The doctrine
ULFI, he cannot be made personally liable for its adjudged corporate mentioned, on the other hand, is an equitable remedy resorted to only
liability. He took exception to the complaint, characterizing it as an when the corporate fiction is used, among others, to defeat public
attempt to pierce the corporate veil that cloaked ULFI. convenience, justify wrong, protect fraud or defend a crime. [13]

Satisfied that the DECS fully established its case, on October 14, 2002, the Moreover, in a piercing case, the test is complete control or domination,
RTC rendered judgment, ordering Kahn and petitioner Sanchez to pay the not only of finances, but of policy and business practice in respect of the
DECS, jointly and severally, P22,559,215.14 with legal interest from April transaction attacked.[14]This is not the case here. Section 31, under which
1, 1996 until they shall have fully paid the same, P500,000.00 in this case was brought, makes a corporate directorwho may or may not
exemplary damages, and P200,000.00 in attorneys fees, plus costs.[11] even be a stockholder or memberaccountable for his management of the
affairs of the corporation.
Both Kahn and petitioner Sanchez appealed to the Court of Appeals. The
latter court gave due course to Sanchezs appeal but denied that of Kahn Bad faith implies breach of faith and willful failure to respond to plain and
since it was filed out of time. On February 21, 2006 the Court of Appeals well understood obligation.[15] It does not simply connote bad judgment or
rendered judgment, wholly affirming the trial courts decision, [12] hence, negligence; it imports a dishonest purpose or some moral obliquity and
this petition. conscious doing of wrong; it means breach of a known duty through some
motive or interest or ill will.[16] It partakes of the nature of fraud.[17]
In a nutshell, Sanchez argues that he cannot be made personally liable for
ULFIs corporate obligations absent specific allegations in the complaint Gross negligence, on the other hand, is the want of even slight care, acting
and evidence adduced during trial that would warrant a piercing of the or omitting to act in a situation where there is duty to act, not inadvertently
corporate veil. He further argues that the DECS is barred by res but willfully and intentionally, with a conscious indifference to
judicata and forum shopping from collecting from him what it could not consequences insofar as other persons may be affected.[18] It evinces
get by execution from ULFI under the judgment in the ejectment a thoughtless disregard of consequences without exerting any effort to
case. Finally, he claims that because ULFI suffered losses in operations avoid them;[19] the want or absence of or failure to exercise slight care or
during the period 1992 up to 1996, there could have been nothing left of diligence, or the entire absence of care.[20]
the rentals it collected from the lessees of the Complex.
In resolving the issue of whether or not petitioner Sanchez, a director and
The DECS points out, on the other hand, that since Kahn and petitioner chief executive officer of ULFI, can be held liable in damages under
Sanchez were guilty of fraud and bad faith in managing the funds of ULFI, Section 31 of the Corporation Code for bad faith or gross neglect in
they can be made to personally answer for those funds and to pay its directing the corporations affairs, the Court will consider only the Court of
corporate obligations pursuant to Section 31 of the Corporation Appeals findings of facts. This Courts jurisdiction in a petition for review
Code. They collected money from rents but did not, as was their duty, on certiorari under Rule 45 is limited to reviewing only errors of law. It is
remit this to the DECS pursuant to the DECS-ULFI agreement. bound by the findings of fact of the Court of Appeals.

The Issues The Court of Appeals found that from January 1992 to January 1996, after
ULFIs authority to manage the Complex expired and despite the ejectment
The case before this Court presents the following issues: suit that the DECS filed against it, petitioner Sanchez and Kahn still
continued to lease spaces in those facilities to third persons. And they
1. Whether or not petitioner Sanchez, a director and chief executive officer collected and kept all the rents although they knew that these primarily
of ULFI, can be held liable in damages under Section 31 of the belonged to the DECS. ULFI had merely managed the facilities and
Corporation Code for gross neglect or bad faith in directing the collected earnings from them for the DECS. What is more, Sanchez and
corporations affairs; and Kahn were aware that they had to submit written accounts of those rents
and remit the net earnings from them to the Bureau of Treasury, through
2. Whether or not the action in Civil Case 66852 is barred by res the DECS, at the end of the year. Yet, Sanchez and Kahn, acting in bad
judicata and constitutes forum shopping by the DECS. faith or with gross neglect did not turn over even one centavo of rent to the
DECS nor render an accounting of their collections. Nor did they account
Rulings of the Court for the money they collected by submitting to the Securities and Exchange
Commission the required financial statements covering such collections.
Petitioner Sanchez points out that the Court of Appeals decision arbitrarily
changed the DECSs theory of the case from one based on his and Kahns Parenthetically, a witness for the defense, Evangeline Naniong, ULFIs
alleged failure to deposit for the account of ULFI whatever rentals they bookkeeper, testified that the revenues from the rents were deposited in
have collected to another based on their alleged failure to remit to the the bank in the names of Sanchez and ULFIs accountant. And so only they
DECS the incomes of the facilities they managed. But Sanchez is drawing could withdraw and spend those revenues.[21]
insignificant distinctions from what the DECS claims and what the court
below finds. Both essentially rest on Kahn and Sanchezs failure to account Petitioner Sanchez of course claims that the funds they had collected
for the rent incomes that they collected from lease of spaces in the proved inadequate even to meet expenses. But, as the appellate court held,
facilities of the Complex beyond the one-year management authority that he had been unable to substantiate such claims. As the officer charged
the DECS granted ULFI in 1991. with approving and implementing corporate disbursements, Sanchez had
the duty to present documents showing how the incomes of the foundation
Petitioner Sanchez claims that there is no ground for the courts below to were spent. But he failed to do so even after the DECS, which took
pierce the veil of corporate identity and hold him and Kahn, who were custody of the records, asked Kahn to submit a list of the documents they
mere corporate officers, personally liable for ULFIs obligations to the needed for establishing their defenses so these may be made available to
DECS. But this is not a case of piercing the veil of corporate fiction. The them.[22] Under the circumstances, the indubitable conclusion is that
DECS brought its action against Sanchez and Kahn under Section 31 of petitioner Sanchez and Kahn acted with bad faith, if not with gross
the Corporation Code, which should not be confused with actions intended negligence, in failing to perform their duty to remit to DECS or keep in
to pierce the corporate fiction. safe hands ULFIs incomes from the leases.

Section 31 of the Corporation Code makes directors-officers of Section 31 lays down the doctrine of corporate opportunity and holds
corporations jointly and severally liable even to third parties for their gross personally liable corporate directors found guilty of gross negligence or
negligence or bad faith in directing the affairs of their corporations. Thus: bad faith in directing the affairs of the corporation, which results in
damage or injury to the corporation, its stockholders or members, and
Sec. 31. Liability of directors, trustees or officers. - Directors or trustees other persons. The ejectment suit that held only ULFI liable to the DECS
who willfully and knowingly vote for or assent to patently unlawful acts of for unpaid rents does not constitute res judicata to the issue of personal
the corporation or who are guilty of gross negligence or bad faith in liabilities of Kahn and petitioner Sanchez under the circumstances to pay
such obligations, given that the unaccounted funds would have settled the We stress that the corporate fiction should be set aside when it becomes a
same. shield against liability for fraud, or an illegal act on inequity committed on
third person. The question of piercing the veil of corporate fiction is
Petitioners allegations of forum shopping must fail as well. The essence of essentially, then a matter of proof.
forum shopping is the filing of multiple suits involving the same parties In the present case, however, the court finds no reason to pierce the
for the same cause of action, either simultaneously or successively, for the corporate veil of respondent Motorich. Petitioner utterly failed to establish
purpose of obtaining a favorable judgment.[23] This is not the case with the said corporation was formed, or that it is operated for the purpose of
respect to the ejectment suit vis--vis the action for damages. shielding any alleged fraudulent or illegal activities of its officers or
stockholders; or that the said veil was used to conceal fraud, illegality or
WHEREFORE, the Court DENIES the petition inequity at the expense of third persons like petitioner.
and AFFIRMS the February 21, 2006 Decision of the Court of Appeals in
CA-G.R. CV 83648 and its Resolution of May 29, 2006. Additional Notes:
Definition of Close Corporation
SO ORDERED. Petitioner claims that Motorich is a close corporation. We rule that it is
not. Section 96 of the Corporation Code defines a close corporation as
San Juan Structural and Steel Fabricators, Inc. vs. Court of Appeals follows:
G.R. No. 129459. September 29, 1998 “SEC. 96. Definition and Applicability of Title.—A close corporation,
SAN JUAN STRUCTURAL AND STEEL FABRICATORS, INC., within the meaning of this Code, is one whose articles of incorporation
petitioner, vs. COURT OF APPEALS, MOTORICH SALES provide that: (1) All of the corporation’s issued stock of all classes,
CORPORATION, NENITA LEE GRUENBERG, ACL exclusive of treasury shares, shall be held of record by not more than a
DEVELOPMENT CORP. and JNM REALTY AND specified number of persons, not exceeding twenty (20); (2) All of the
DEVELOPMENT CORP., respondents. issued stock of all classes shall be subject to one or more specified
restrictions on transfer permitted by this Title; and (3) The corporation
Nature of the Case: shall not list in any stock exchange or make any public offering of any of
The Case is a Petition for Review on Certiorari, assailing the March 18, its stock of any class. Notwithstanding the foregoing, a corporation shall
1997 Decision of the Court of Appeals in CA GR CV No. 46801 which, in be deemed not a close corporation when at least two-thirds (2/3) of its
turn, modified the July 18, 1994 Decision of the Regional Trial Court of voting stock or voting rights is owned or controlled by another corporation
Makati, Metro Manila, Branch 633 in Civil Case No. 89-3511 which which is not a close corporation within the meaning of this Code. x x x.”
dismissed both the Complaint and the Counterclaim filed by the parties. The articles of incorporation of Motorich Sales Corporation does not
Facts: contain any provision stating that (1) the number of stockholders shall not
Plaintiff San Juan structural and Steel fabricators Inc.’s amended exceed 20, or (2) a preemption of shares is restricted in favor of any
complaint alleged that on February 14, 1989, plaintiff-appellant entered stockholder or of the corporation, or (3) listing its stocks in any stock
into an agreement with defendant Motorich Sales Corporation for the sale exchange or making a public offering of such stocks is prohibited. From
of a parcel of land identified as lot 30, Block 1 of the Acropolis Greens its articles, it is clear that Respondent Motorich is not a close corporation.
Subdivision located in the district of Murphy, Quezon City, Metro Manila Motorich does not become one either, just because Spouses Reynaldo and
containing an area of 414 square meters, covered by TCT No. 362909; Nenita Gruenberg owned 99.866% of its subscribed capital stock. The
through the latter’s treasurer, Nenita Gruenberg. San Juan advanced “mere ownership by a single stockholder or by another corporation of all
P100,000.00 to Nenita as earnest money, the balance to be paid on or or nearly all of the capital stock of a corporation is not of itself sufficient
before March 2, 1989; ground for disregarding the separate corporate personalities.”
On the day agreed upon on which Nenita was supposed to deliver the title
of the land to Motorich, Nenita did not show up. Nenita and Motorich did Safic Alcan & Cie vs Imperial Vegetable Oil Co., Inc.
not heed the subsequent demand of San Juan to comply with the contract Business Organization – Corporation Law – Ultra Vires Acts of
hence San Juan sued Motorich. Motorich, in its defense, argued that it is Corporate Officers
not bound by the acts of its treasurer, Nenita, since her act in contracting In 1985, Safic Alcan & Cie (SAC), a corporation, entered into an
with San Juan was not authorized by the corporate board. agreement with Imperial Vegetable Oil Co., Inc. (IVO) whereby the latter
San Juan raised the issue that Nenita was actually the wife of the President shall deliver tones of coconut oil to SAC. Both parties complied. IVO was
of Motorich; that Nenita and her husband owns 99.866% of the represented by its president, Dominador Monteverde. In 1986, SAC again
corporation’s capital stocks; that as such, it is a close corporation and that entered into an several agreements with IVO but this time it was agreed
makes Nenita and the President as principal stockholders who do not need that IVO shall deliver the coconut oil 8 months from the agreement or
any authorization from the corporate board; that in this case, the corporate sometime in 1987. This time, IVO failed to deliver and SAC sued IVO.
veil may be properly pierced. IVO in its defense aver that Monteverde was acting beyond his power as
Issues: president when he made the 1986 agreement with SAC; that Monteverde
1) Whether or not the corporation’s treasurer act can bind the is acting beyond his power because the 1986 contracts were speculative in
corporation. nature and speculative contracts are prohibited by the by-laws of IVO.
2) Whether or not the doctrine of piercing the veil of corporate SAC insists that there is an implied agency between IVO and Monteverde
entity is applicable. because SAC and Monteverde has been transacting since 1985 and that
Held: IVO benefited from said transactions.
No. Such contract cannot bind Motorich, because it never authorized or ISSUE: Whether or not Monteverde’s act in entering into the 1986
ratified such sale. contracts is ultra vires.
A corporation is a juridical person separate and distinct from its HELD: Yes. It was proven by IVO, when they presented a copy of their
stockholders or members. Accordingly, the property of the corporation is by-laws, that Monteverde acted beyond his authority when he entered into
not the property of its stockholders or members and may not be sold by the speculative contracts with SAC in 1986. The 1986 contracts are
stockholders or members without express authorization from the speculative because at the time of the contracts, the coconuts are not even
corporation’s board of directors. growing at that time and are yet to be harvested. Hence, the 1986 contracts
Section 23 of BP 68 provides the Board of Directors or Trustees – Unless are sales of mere expectations – and this is something prohibited by the
otherwise provided in this code, the corporate powers of all corporations by-laws and the Board of Directors of IVO.
formed under this code shall be exercised, all business conducted, and all There can be no implied agency too simply because there has been a
property of such corporations controlled and held by the board of directors previous transaction between SAC and IVO where IVO was represented
or trustees to be elected from among the stockholders of stocks, or where by Monteverde. This is because the 1985 contract and the 1986 contracts
there is no stock, from among the members of the corporations, who shall are very different. The 1985 contract is not speculative while the 1986
hold office for 1 year and until their successors are elected and qualified. contracts are speculative hence, SAC should have secured the
As a general rule, the acts of corporate officers within the scope of their confirmation by IVO’s Board that Monteverde is indeed authorized to
authority are binding on the corporation. But when these officers exceed enter into such agreements. Further, Monteverde did not even present the
their authority, their actions, cannot bind the corporation, unless it has said 1986 agreements before the Board of Directors so there was, in fact,
ratified such acts as is estopped from disclaiming them. no occasion at all for ratification. The contracts were not even reported in
Because Motorich had never given a written authorization to respondent IVO’s export sales book and turn-out book. Neither were they reflected in
Nenita Gruenbeg to sell its parcel of land, we hold that the February 14, other books and records of the corporation. It must be pointed out that the
1989 agreement entered into by the latter with petitioner is void under Board of Directors, not Monteverde, exercises corporate power. Clearly,
Article 1874 of the Civil Code. Being inexistent and void from the Monteverde’s speculative contracts with Safic never bound IVO and Safic
beginning, said contract cannot be ratified. cannot therefore enforce those contracts against IVO.
The statutorily granted privilege of a corporate veil may be used only for
legitimate purposes. On equitable consideration, the veil can be Rural Bank of Milaor vs. Francisca Ocfemia et. al G.R. No 137686,
disregarded when it is utilized as a shield to commit fraud, illegality or February 8, 2000
inequity, defeat public convenience; confuse legitimate issues; or serve as Thursday, January 29, 2009 Posted by Coffeeholic Writes
a mere alter ego or business conduit of a person or an instrumentality, Labels: Case Digests, Commercial Law
agency or adjunct of another corporation.
FACTS: Several parcels of land were mortgaged by the respondents shares, was able to nominate and elect a board of directors to his own
during the lifetime of the respondent’s grandparents to the Rural bank of liking, without opposition from the minority. After the board of directors
Milaor as shown by the Deed of Real Estate Mortgage and the Promissory had been thus elected and had qualified, they chose a set of officers
Note. Spouses Felicisimo Ocfemia and Juanita Ocfemia, one of the constituting of Jose M. Yusay, president, Timoteo Unson, vice-president,
respondents, were not able to redeem the mortgaged properties consisting Jose G. Montalvo, secretary-treasurer, and H. W. Corp and Agustin
of seven parcels of land and so the mortgage was foreclosed and thereafter Coruna, as members. Said officials immediately entered upon the
ownership was transferred to the petitioner bank. Out of the seven parcels discharged of their duties and have continued in possession of their
of land that were foreclosed, five of them are in the possession of the respective offices until the present time.
respondents because these five parcels of land were sold by the petitioner Since the creation of the voting trust there have been a number of
bank to the respondents as evidenced by a Deed of Sale. However, the five vacancies caused by resignation or the absence of members from the
parcels of land cannot be transferred in the name of the parents of Merife Philippine Islands, with the result that various substitutions have been
Nino, one of the respondents, because there is a need to have the made in the personnel of the voting trust. At the present time the
document of sale registered. The Register of deeds, however, said that the petitioners Roxas, Echaus, and Lacson presumably constitute its
document of sale cannot be registered without the board resolution of the membership. We say presumably, because in the present proceedings an
petitioner bank confirming both the Deed of sale and the authority of issue of fact is made by the respondents upon the point whether the three
thebank manager, Fe S. Tena, to enter such transaction. individuals named have been regularly substituted for their several
predecessors. In the view we take of the case it is not necessary to
The petitioner bank refused her request for a board resolution and made determine this issue; and we shall assume provisionally that the three
many alibis. Respondents initiated the present proceedings so that they petitioners are the lawful components of the voting trust.
could transfer to their names the subject five parcel of land and Although the present officers of the Binalbagan Estate, Inc., were elected
subsequently mortgage said lots and to use the loan proceeds for the by the representative of the voting trust, the present trustee are apparently
medical expenses of their ailing mother. desirous of ousting said officers, without awaiting the termination of their
official terms at the expiration of one year from the date of their election.
ISSUE: May the Board of Directors of a rural banking corporation be In other to effect this purpose the petitioners in their character as members
compelled to confirm a deed of absolute sale of real property owned by of the voting trust, on August 2, 1926, caused the secretary of the
the corporation which deed of sale was executed by the bank manager Binalbagan Estate, Inc., to issue to the shareholders a notice calling for a
without prior authority of the board of directors of the rural banking special general meeting of shareholders to be held at 10 a. m., on August
corporation? 16, 1926, "for the election of the board of directors, for the amendment of
the By-Laws, and for any other business that can be dealt with in said
HELD: YES. The bank acknowledges, by its own acts or failure to act, the meeting."
authority of Fe S. Tena to enter into binding contracts. After the execution Within a few days after said notice was issued Agustin Coruña, as member
of the Deed of Sale, respondents occupied the properties indispute and of the existing board, and Mauro Ledesma, as a simple shareholder of the
paid the real estate taxes. If the bank management believed that it had title corporation, instituted a civil action (No. 3840) in the Court of First
to the property, it should have taken measured to prevent the infringement Instance of Occidental Negros against the trustees and the Binalbagan
and invasion of title thereto and possession thereof. Likewise, Tena had Estate, Inc., for the purpose of enjoining the meeting completed in the
previously transacted business on behalf of the bank, and the latter had notice above-mentioned.
acknowledged her authority. A bank is liable to innocent third persons In response to a proper for a preliminary injunction, in connection with
where representation is made in the course of its normal business by an said action, the respondent judge issued the restraining order, or
agent like Manager Tena even though such agent is abusing her authority. preliminary injunction, which gave rise to the present petition for the writ
Clearly, persons dealing with her could not be blamed for believing that of certiorari. In the dispositive part of said order the Binalbagan Estate,
she was authorized to transact business for and on behalf of the bank. Inc., its lawyers, agents, representatives, and all others who may be
assisting or corroborating with them, are restrained from holding the
The bank is estopped from questioning the authority of the bank to enter general shareholders' meeting called for the date mentioned and from
into contract of sale. If a corporation knowingly permits one of its officers electing new directors for the company in substitution of the present
or any other agent to act within the scope of an apparent authority, it holds incumbents, said injunction to be effective until further order of the court.
the agent out to the public as possessing the power to do those acts; thus, it is now asserted here by the petitioners that the making of this order was
the corporation will, as against anyone who has in good faith dealt with it beyond the legitimate powers of the respondent judge, and it is
through such agent, be estopped from denying the agent’s authority. accordingly prayed that said order be set aside.
We are of the opinion that this contention is untenable and that the
G.R. No. L-26555 November 16, 1926 respondent judge acted within his legitimate powers in making the order
BALDOMERO ROXAS, ENRIQUE ECHAUS and ROMAN J. against which relief is sought. In order to expose the true inwardness of
LACSON, petitioners, the situation before us it is necessary to take not of the fact that under the
vs. law the directors of a corporation can only be removed from office by a
Honorable MARIANO DE LA ROSA, Auxiliary Judge of First vote of the stockholders representing at least two-thirds of the subscribed
Instance of Occidental Negros, AGUSTIN CORUNA, MAURO capital stock entitled to vote (Act No. 1459, sec. 34); while vacancies in
LEDESMA and BINALBAGAN ESTATE, INC., respondents. the board, when they exist, can be filled by mere majority vote, (Act No.
Roman J. Lacson, for petitioners. 1459, sec. 25). Moreover, the law requires that when action is to be taken
The respondent judge in his own behalf. at a special meeting to remove the directors, such purpose shall be
The respondent corporation in its own behalf. indicated in the call (Act No. 1459, sec. 34).
R. Nolan and Feria and La O for the respondents Coruna and Ledesma. Now, upon examining into the number of shares controlled by the voting
trust, it will be seen that, while the trust controls a majority of the stock, it
STREET, J.: does not have a clear two-thirds majority. It was therefore impolitic for the
This is an original petition for the writ of certiorari whereby the petitioners, in forcing the call for the meeting of August 16, to come out
petitioners, Baldomeo Roxas, Enrique Echaus, and Roman J. Lacson, seek frankly and say in the notice that one of the purpose of the meeting was to
to procure the abrogation of an order of the respondent judge granting a removed the directors of the corporation from office. Instead, the call was
preliminary injunction in an action in the Court of First Instance of limited to the election of the board of directors, it being the evident
Occidental Negros, instituted by Agustin Coruna and Mauro Ledesma intention of the voting trust to elect a new board as if the directorate had
against the petitioners and the Binalbagan Estate, Inc. The cause is now been then vacant.
before us upon the issues made by the answers filed by the respondents. But the complaint in civil No. 3840 directly asserts that the members of
It appears that the Binalbagan Estate, Inc., is a corporation having its the present directorate were regularly elected at the general annual
principal plant in Occidental Negros where it is engaged in the meeting held in February, 1926; and if that assertion be true, the proposal
manufacture of raw sugar from canes grown upon farms accessible to its to elect, another directorate, as per the call of August 2, if carried into
central. In July, 1924, the possessors of a majority of the shares of the effect, would result in the election of a rival set of directors, who would
Binalbagan Estate, Inc., formed a voting trust composed of three members, probably need the assistance of judgment of court in an independent action
namely, Salvador Laguna, Segunda Monteblanco, and Arthur F. Fisher, as of quo warrantoto get them installed into office, even supposing that their
trustee. By the document constituting this voting trust the trustees were title to the office could be maintained. That the trial judge had jurisdiction
authorized to represent and vote the shares pertaining to their constituents, to forestall that step and enjoin the contemplated election is a matter about
and to this end the shareholders undertook to assign their shares to the which there cannot be the slightest doubt. The law contemplates and
trustees on the books of the company. The total number of outstanding intends that there will be one of directors at a time and that new directors
shares of the corporation is somewhat over 5,500, while the number of shall be elected only as vacancies occur in the directorate by death,
shares controlled by the voting trust is less than 3,000. resignation, removal, or otherwise. lawphil.net
On February 1, 1926, the general annual meeting of the shareholders of It is instituted that there was some irregularity or another in the election of
the Binalbagan Estate, Inc., took place, at which Mr. J. P. Heilbronn the present directorate. We see nothing upon which this suggestion can be
appeared as representative of the voting trust, his authority being safely planted; And at any rate the present board of directors are de
recognized by the holders of all the other shares present at this meeting. factoincumbents of the office whose acts will be valid until they shall be
Upon said occasion Heilbronn, by virtue of controlling the majority of the lawfully removed from the office or cease from the discharge of their
functions. In this case it is not necessary for us to agitate ourselves over basis. Consequently, the corresponding issuance of shares was without
the question whether the respondent judge properly exercised his judicial authority of the board of directors."
discretion in granting the order complained of. If suffices to know that in xxxxxxxxx
making the order he was acting within the limits of his judicial powers. "WHEREFORE, premises considered, this Commission finds and so holds
It will be noted that the order in question enjoins the defendants from that the purported board resolution of December 29, 1975, not having
holding the meeting called for August 16; and said order must not be been properly passed upon at a duly constituted board meeting,
understood as constituting any obstacle for the holding of the regular cannot be recognized as valid and hence, without legal force and
meeting at the time appointed in the by-laws of the corporation. effect. Consequently, the issuance of shares of stock to corporate
For the reasons stated the petition will be denied, and it is so ordered, with creditors of the Tagaytay Taal Tourist Development Corporation is
costs. null and void. In view thereof, the shares in question are still considered
DIVISION unissued and remain part of the authorized capital stocks of the Tagaytay
Taal Tourist Development Corporation. This is without prejudice to the
[ GR No. 136821, Oct 17, 2002 ] rights of said corporate creditors as against Tagaytay Taal Tourist
ROVELS ENTERPRISES v. EMMANUEL B. OCAMPO + Development Corporation for the latter's contractual obligations."
DECISION (emphasis added)
On appeal by Roberto Roxas and Eduardo Santos, the SEC en banc, in its
439 Phil. 777 Decision dated September 2, 1982 in SEC-AC No. 049,[9]affirmed the
Decision of the SEC Hearing Officer. This Court, in its Decision of June
20, 1983 in G.R. No. 61863,[10] likewise affirmed the Decision of the
SANDOVAL-GUTIERREZ, J.: SEC en banc. The Decision of this Court became final and executory on
Assailed in this petition for review on certiorari[1] is the Decision of the September 2, 1983.[11]
Court of Appeals dated June 5, 1998[2] in CA-G.R. SP No. 43260, Subsequently, TTTDC, Jose Silva, Emmanuel Ocampo, Victoriano
affirming the Decision of the Securities and Exchange Commission Leviste, Francisco Carreon, Jr., and Expedito Leviste, Sr., another
(SEC) in SEC Case No. 09-95-5135 dismissing the petition to be declared stockholder of TTTDC, (the SILVA GROUP, now respondents), filed
the majority stockholder of Tagaytay Taal Tourist Development with the SEC a petition against Eduardo Santos, Sylvia S. Veloso, Josefina
Corporation (TTTDC). The petition was filed by Rovels Enterprises, Inc. Carballo, Augusto del Rosario, Reynaldo Alcantara and Lauro Sandoval
(Rovels), herein petitioner. Rovels is a domestic corporation engaged in (the SANTOS GROUP), docketed as SEC Case No. 3806. (The SANTOS
construction work. Its President is Eduardo Santos. TTTDC was among GROUP were nominees of Rovels who, by virtue of the shares of stock
Rovels' clients. issued pursuant to the December 29, 1975 Resolution, proceeded to act as
In payment for the services rendered by Rovels, the Board of Directors of directors and officers of TTTDC). In their petition, the SILVA GROUP
TTTDC passed a Resolution on December 29, 1975 providing as follows: prayed that they be declared the true and lawful stockholders and
"RESOLVED, as it is hereby resolved that payment for professional fees incumbent directors and officers of TTTDC.
and services rendered by x x x Rovels' Enterprises x x x be made in cash if On July 6, 1993, SEC Hearing Officer Alberto P. Atas rendered a
funds are available, or its equivalent number of shares of stock of the Decision[12] in favor of the SILVA GROUP, thus:
corporation at par value, and should said creditors elect the latter "WHEREFORE, judgment is hereby rendered in favor of the petitioners
mode of payment, it is further resolved that the President and/or his (SILVA GROUP) and against the respondents (SANTOS GROUP), as
Secretary be authorized as they are hereby authorized, to issue the follows:
corresponding unissued shares of stock of the a. Declaring petitioners as the lawful stockholders, directors and officers
corporation."[3] (emphasis added) of Tagaytay Taal Tourist Development Corporation;
The Resolution was signed by three of TTTDC's directors, namely, b. Declaring respondents, to be not stockholders of Tagaytay Taal Tourist
Victoriano Leviste, Bienvenido Cruz, Jr., and Roberto Roxas. Roberto Development Corporation;
Roxas is the President of TTTDC and stockholder of Rovels at the same c. Declaring respondents to be not directors or officers of Tagaytay Taal
time. Noticeably, the signatures of the other two (2) TTTDC directors Jose Tourist Development Corporation;
Silva, Jr. and Emmanuel Ocampo do not appear in the subject Resolution d. The writ of preliminary injunction issued on November 6, 1990 is
despite their presence in the December 29, 1975 Board meeting. [4] hereby made permanent; and
On February 23, 1976, Eduardo Santos, President of Rovels, on behalf of e. Ordering the Records Division of this Commission to purge the records
TTTDC, filed with the SEC an application for exemption from registration of Tagaytay Taal Tourist Development Corporation of all papers and
of TTTDC's unissued shares of stock transferred to it (Rovels) as payment documents filed by respondents purportedly in behalf of Tagaytay Taal
for its services worth One Hundred Eight Thousand Pesos (P108,000.00). Tourist Development Corporation." (emphasis and words in parentheses
This was done because under Section 4 (a) of the Revised Securities Act, added)
no shares of stocks shall be transferred unless first registered with the SEC The above Decision became final and executory on September 1,
or permitted to be sold.[5] 1994[13] as no appeal was interposed by either the SILVA GROUP or the
On May 7, 1976, the SEC, in its Resolution No. 260, [6] granted Eduardo SANTOS GROUP.
Santos' application. However, Rovels, to whom the TTTDC shares of stock (worth
On March 1, 1976, the TTTDC Board of Directors passed another P108,000.00) were transferred, claimed that it became aware of the July 6,
Resolution[7] repealing its Resolution of December 29, 1975, thus: 1993 SEC Decision only in June of 1995. So on September 6, 1995, it
"RESOLVED, as it is hereby resolved, that the Resolution of December filed a petition with the SEC,[14] docketed as SEC Case No. 09-95-5135,
29, 1975 authorizing the payment of creditorswith unissued shares of praying that it be declared the majority stockholder of TTTDC as against
the corporation be as it is hereby repealed: Resolved further that the respondents Ocampo, Silva, Leviste, Sr., Calalang and Carreon (belonging
matter as well as the amount of the creditors' claims be given adequate to the SILVA GROUP). The material allegations of the petition state that:
study and consideration by the Board." (emphasis added) (1) TTTDC passed a Resolution dated December 29, 1975 authorizing the
In view of the December 29, 1975 TTTDC Board Resolution transferring transfer of its unissued shares to Rovels as the latter's construction
to Rovels the said shares of stock as construction fee, TTTDC Directors fee;[15] (2) Pursuant to that Resolution, TTTDC shares of stock worth
Jose Silva, Jr. and Emmanuel Ocampo filed a complaint with the SEC P692,000.00 were transferred to Rovels;[16] (3) While TTTDC, in its
against Roberto Roxas, TTTDC President, and Eduardo Santos, Rovels' March 1, 1976 Resolution, repealed the December 29, 1975 Resolution,
President, docketed as SEC Case No. 1322. In their complaint, Silva and such repeal does not bind Rovels for lack of notice;[17] (4) Several
Ocampo alleged that there was no meeting of the TTTDC's Board of "interrelated cases" (SEC Case Nos. 1322 and 3806) were filed with the
Directors on December 29, 1975; that they did not authorize the transfer of SEC involving the SILVA and SANTOS GROUPS;[18] (5) Rovels is not
TTTDC's shares of stock to Rovels; that they never signed the alleged bound by the SEC Decisions since it was not impleaded as a party in said
minutes of the meeting; and that the signatures of the other two (2) cases.[19]
Directors, Victoriano Leviste and Bienvenido Cruz, Jr., as well as that of Forthwith, the SILVA GROUP filed a motion to dismiss[20] the petition
TTTDC's Secretary Francisco Carreon, Jr., were obtained through fraud on the following grounds: (1) Rovels has no cause of action since
and misrepresentation. They also alleged that the TTTDC Board TTTDC's December 29, 1975 Board Resolution was repealed by its March
Resolution dated December 29, 1975 was repealed by the March 1, 1976 1, 1976 Resolution;[21] (2) the petition is barred by the prior SEC
Resolution. They thus prayed that the transfer of TTTDC's shares of stock Decisions in SEC Case No. 1322 declaring that the issuance of TTTDC's
to Rovels pursuant to Resolution dated December 29, 1975 be annulled. shares of stock to Rovels is valid, and the SEC Decision in 3806 declaring
On March 17, 1979, SEC Hearing Officer Eugenio E. Reyes issued a the SILVA GROUP as the lawful stockholders of TTTDC;[22] and (3) the
Decision[8] in favor of Silva and Ocampo, the dispositive portion of which petition is barred by estoppel, prescription and laches since it was filed
reads: long after Rovels was notified of the repeal of the December 29, 1975
"Considering that the (December 29, 1975) board resolution which TTTDC Resolution.[23]
authorizes the corporation to pay its creditors with its unissued shares of In an Order dated April 22, 1996[24] in SEC Case No. 09-95-5135, SEC
stock x x x had been expressly revoked or repealed on March 1, Hearing Officer Manuel P. Perea dismissed Rovel's petition on the
1976 as earlier pointed out, Commission Resolution No. 260 (granting grounds of lack of cause of action, res judicata, estoppel, laches and
Santos' application for exemption from registration of the unissued prescription. This Order was affirmed by the SEC en banc in its Decision
shares), when issued on May 7, 1976 x x x had lost its legal dated January 20, 1997[25] in SEC AC No. 560.
Upon a petition for review, docketed as CA-G.R. SP. No. 43260, the Court A reading of the above petition (paragraph 5) shows that Rovels' prayer to
of Appeals, in its Decision dated June 5, 1998,[26] affirmed the January 20, be declared the majority stockholder of TTTDC is anchored on the
1997 SEC en banc Decision. Rovels' motion for reconsideration was December 29, 1975 TTTDC Board Resolution transferring its shares of
likewise denied.[27] stock to Rovels as construction fee. This Resolution could have vested in
Hence, the instant petition for review on certiorari,[28] alleging that the Rovels a right to be declared a stockholder of TTTDC. However, the same
Court of Appeals erred: petition (paragraphs 9 and 10) concedes that the December 29, 1975
I Resolution was repealed by the March 1, 1976 Resolution. The petition
likewise alleges (paragraphs 12 and 19) that there were prior "interrelated
IN HOLDING THAT PETITIONER ROVELS HAS NO CAUSE OF cases" filed with the SEC between the SILVA and SANTOS GROUPS,
ACTION AGAINST PRIVATE RESPONDENTS; and namely: (1) SEC Case No. 1322(wherein the SEC en banc in its Decision
II dated September 2, 1982 nullified the TTTDC Board Resolution dated
IN HOLDING THAT THE PETITION IN SEC CASE NO. 09-95-5135 IS December 29, 1975, which Decision was affirmed with finality by this
BARRED BY PRIOR JUDGMENT (RES JUDICATA), LACHES, Court in G.R. No. 61863) and (2) SEC Case No. 3806 (wherein the SEC
PRESCRIPTION AND ESTOPPEL.[29] declared the SILVA GROUP as the legitimate stockholders of TTTDC,
The petition is unmeritorious. not Rovels' nominees [the SANTOS GROUP]). Clearly, on the face of its
On the first assigned error, we find that the Court of Appeals is correct in petition, Rovels cannot claim to be the majority stockholder of TTTDC.
affirming the dismissal of Rovels' petition in SEC Case No. 09-955135 for Relative to the second assigned error, Rovels contends that it is not bound
lack of cause of action. by the SEC Decision in SEC Case Nos. 1322 and 3806 and in G.R. No.
A cause of action is defined as the delict or wrongful act or omission 61863 as it was "never a party in any of these cases." This contention
committed by a person in violation of the right of another.[30] A cause of brings us to the issue of res judicata.
action exists if the following elements are present: (1) a right in favor of The requisites of res judicata,[34] also known as the rule on bar by prior
the plaintiff, (2) the correlative obligation of the defendant to respect such judgment, are:
right, and (3) the act or omission of the defendant in violation of plaintiff's 1) the former judgment must be final;
right.[31] The test is whether the material allegations of the complaint, 2) the court which rendered it had jurisdiction over the subject matter and
assuming them to be true, state ultimate facts which constitute plaintiff's the parties;
cause of action, such that plaintiff is entitled to a favorable judgment as a 3) the judgment must be on the merits; and
matter of law.[32] 4) there must be between the first and the second actions, identity of
The pertinent portions of Rovels petition filed with the SEC read: parties, subject matter and causes of action.
xxxxxxxxx The first three (3) requisites of res judicata are present in this case. This is
"5. x x x. On December 29, 1975, TTTDC in a Resolution signed by not disputed by the parties and is, in fact, established by the record. The
majority members of the Board of Directors resolved that TTTDC pay its controversy arises as to whether there is identity of the parties in the
creditors through a 'debt-to-equity swap;' present SEC Case No. 09-95-5135, on the one hand, and in prior SEC
xxxxxxxxx Case Nos. 1322 and 3806, on the other.
"9. x x x the relation between the Silva faction and the Santos Contrary to its claim, Rovels is bound by the previous SEC Decisions. It
faction became adversarial. The Silva faction attempted to form an must be noted that Eduardo Santos, President of Rovels, was one of the
alleged new board of directors and repealed the Board Resolution dated respondents in both SEC Case Nos. 1322 and 3806. Clearly, Rovels and
December 29, 1975 Resolution regarding the 'debt' to equity swap. Thus, Eduardo Santos, being its President, share an identity of
it resolved: interests sufficient to make them privies-in-law, as correctly found by
'RESOLVED, as it is hereby resolved, that the Resolution of December the Court of Appeals in its assailed Decision, thus:
29, 1975 authorizing the payment of creditors with unissued shares of "In the case at bench, there can be no question that the rights claimed by
the corporation be as it is hereby repealed: Resolved further that the petitioner and its stockholders/directors/officers who were parties in SEC
matter as well as the amount of the creditor's claims be given adequate Case Nos. 1322 and 3806 are identical in that they are both based on the
study and consideration by the Board. x x x' December 29, 1975 Resolution. Stated differently, they shared an
"10. That what is clear from the above Resolution of March 1, 1976 is identity of interest from which flowed an identity of relief sought,
the admission that indeed TTTDC owes certain amount of money from its namely, to be declared owners of the stocks of TTTDC, premised on the
creditors. The creditors became stockholders of record as a result of shares same December 29, 1975 Resolution. x x x. This 'identity of interest is
of stock issued in implementation of the 'debt to equity' conversion. sufficient to make them privies-in-law, one to the other, and meets the
Corresponding shares of stock were issued and signed by then president of requisite of substantial identity of parties.'"[35]
the corporation Roberto Roxas and then corporate secretary Francisco N. It bears stressing that absolute identity of parties is not required for the
Carreon, Jr. principle of res judicata, or the rule on bar by prior judgment, to apply.
"Copy of said Certificate of Stocks are hereto attached and marked as Mere substantial identity of parties, or a community of interests between
Annexes 'D' to 'P' and made an integral part hereof. a party in the first case and a party in the subsequent case even if the latter
xxxxxxxxx was not impleaded in the first case, is sufficient.[36]
"12. That several interrelated cases were filed by Eduardo L. Santos Rovels cannot take refuge in the argument that, as a corporation, it is
(SEC Case No. 1322), on one hand, and Expedito M. Leviste, imbued with personality separate and distinct from that of the respondents
Francisco Carreon, Felicisimo Ocampo and Jose M. Silva (SEC Case in SEC Case Nos. 1322 and 3806. The legal fiction of separate corporate
No. 3806) and vice versa on the other. Petitioner, Rovels Enterprises, existence is not at all times invincible and the same may be pierced when
Inc. was never made a party in any of these cases and its nominees in employed as a means to perpetrate a fraud, confuse legitimate issues, or
the Board of Directors of TTTDC continued to exercise its function from used as a vehicle to promote unfair objectives or to shield an otherwise
1976. blatant violation of the prohibition against forum-shopping. While it
xxxxxxxxx is settled that the piercing of the corporate veil has to be done with
"19. That to implement the decision in SEC CASE 3806, which declared caution, this corporate fiction may be disregarded when necessary in the
the Silva Group as the duly authorized directors and officers, without interest of justice.[37]
looking deeply into the records of the case, i.e. the sub-poened authentic The doctrine of res judicata states that a final judgment on the merits
'Stock and Transfer Book' of TTTDC and the earlier decision in PED Case rendered by a court of competent jurisdiction is conclusive as to the rights
No. 89-0644, will constitute irreparable damage to the petitioner. Specially of the parties and their privies, and constitutes an absolute bar to
so, Silva executed an affidavit showing 5 Directors of TTTDC but the subsequent actions involving the same claim, demand or cause of
stock certificates were not signed by the corporate secretary who died in action.[38] This is founded on public policy and necessity, which makes it
1982. to the interest of the State that there should be an end to litigations, and on
xxxxxxxxx the principle that an individual should not be vexed twice for the same
"21. That petitioner which became duly registered majority stockholder cause.[39]
thru 'debt to equity swap' had been an innocent party to such controversy Just recently, we emphatically declared in In Re: Petition Seeking for
between the aforesaid 2 ruling thereof, hence, petitioner remains as is on a Clarification as to the Validity and Forceful Effect of Two (2) Final and
status quo basis as majority stockholder of TTTDC. Executory but Conflicting Decisions of the Honorable Supreme
xxxxxxxxx Court:[40] "Every litigation must come to an end once a judgment
"PRAYER becomes final, executory and unappealable. This is a fundamental and
"WHEREFORE, premises considered, petitioner prays that this Honorable immutable legal principle. For '(j)ust as a losing party has the right to file
Commission render judgment in favor of petitioner and against an appeal within the prescribed period, the winning party also has the
respondents (SILVA GROUP): correlative right to enjoy the finality of the resolution of his case' by the
xxxxxxxxx execution and satisfaction of the judgment, which is the 'life of the law.'
"2. After due notice and hearing, re-declaring petitioner lawful registered Any attempt to thwart this rigid rule and deny the prevailing litigant his
majority stockholder of TTTDC x x x; right to savour the fruit of his victory, must immediately be struck down."
"3. Ordering respondents to desist from sitting in the Board of Directors of Finally, this Court sustains the Appellate Court's finding that the filing of
TTTDC as they are not lawful registered stockholders in the books of the Rovels petition in the instant SEC Case No. 09-95-5135 is barred by
said corporation. estoppel, prescription and laches. There is no merit to Rovels' claim that it
x x x x x x x x x"[33] was only in June of 1995[41] when it became aware of the repeal of the
December 29, 1975 TTTDC Resolution and of the consequent even. And the three directors present were not even listed as current
nullification of the transfer of its shares of stock. directors of PAMBUSCO.
It is undisputed that Eduardo Santos was present in the March 1, 1976
TTTDC Board meeting wherein the December 29, 1975 Resolution was Strong and Strong vs. Repide
repealed. We hold that Eduardo Santos, being the President of Rovels, is 41 Phil. 947 3 May 1909
considered as its (Rovels') agent. As such, his knowledge of the repeal of
the December 29, 1975 Resolution, under the theory of imputed PONENTE: Justice Peckham
knowledge, is ascribed to his principal (Rovels). FACTS:
It was only on September 6, 1995, or almost twenty (20) years from the Among the lands comprising the friar lands are the Dominican
time Eduardo Santos learned of the March 1, 1976 Resolution, that Rovels lands, the only valuable asset owned by the corporation Philippine Sugar
filed its petition in SEC Case No. 09-95-5135. Within that long period of Estates Development Company Limited (Philippine Sugar Estates).
time, Rovels did nothing to contest the March 1, 1976 TTTDC Resolution Francisco Gutierrez Repide (Repide), defendant, was the majority
to protect its rights, if any. Obviously, such inaction constitutes estoppel, stockholder and one of the five directors of Philippine Sugar Estates. He
prescription and laches. As stated by Rovels itself, Article 1149 of the was likewise elected by the board as the agent and administrator general of
New Civil Code limits the filing of actions, whose periods are not fixed such company.
therein or in any other laws, to only five (5) years. In addition, the
principle of laches or "stale demands" provides that the failure or neglect, The factual backdrop being during US occupation, the US
for an unreasonable and unexplained length of time, to do that which by Government wanted to secure title over the friar lands. To accomplish this
exercising due diligence could or should have been done earlier, or the objective, Governor for the Philippines entered into negotiations for the
negligence or omission to assert a right within a reasonable time, warrants purchase of the Dominican lands, during which Repide represented
a presumption that the party entitled to assert it either has abandoned it or Philippine Sugar Estates. The first offer of the Governor was to purchase
declined to assert it.[42] the subject lands in the amount of $6,043,219.47. As the majority
In sum, this Court finds that the Court of Appeals did not commit any stockholder of Philippine Sugar Estates and without prior consultation
reversible error in its challenged Decision. with the other stockholders, Repide rejected the offer. For the second
WHEREFORE, the petition is DENIED. The assailed Decision of the offer, the purchase price was increased to $7,535,000.
Court of Appeals dated June 5, 1998 and its Resolution dated December
21, 1998 in CA-G.R. SP. No. 43260, are AFFIRMED. While negotiations for the second offer were ongoing and while
SO ORDERED. still holding out for a higher price of the Dominican lands, Repide took
steps to purchase the 800 shares of stock of Philippine Sugar Estates.
Rosita Peña vs Court of Appeals These shares were owned by Mrs. Eleanor Strong (Strong) which were
193 SCRA 717 – Business Organization – Corporation Law – Board then in the possession of her agent, F. Stuart Jones (Jones). Repide, instead
resolutions may be questioned by third persons – Board Meetings – of seeing Jones, employed Kauffman who later on employed Sloan, a
Quorum – Sale of Corporate Properties broker, to purchase the shares of Strong. Jones sold the 800 shares of
In 1962, the Pampanga Bus Company (PAMBUSCO) took out a loan from Strong for 16,000 Mexican currency. For this sale transaction a check of
the Development Bank of the Philippines (DBP). PAMBUSCO used the one Rueda Ramos was issued.
parcels of land it owns to secure the loan. In October 1974, due to
PAMBUSCO’s nonpayment, DBP foreclosed the parcels of land. Rosita Later on, the negotiations for the purchase of the Dominican
Peña was the highest bidder. Meanwhile, in November 1974, the Board of lands were concluded and a contract of sale was subsequently executed.
Directors of PAMBUSCO had a meeting. The meeting was attended by This sale transaction increased the value of the shares of stocks originally
only 3 out of the 5 Directors. In the said meeting, the Board, through a owned by Strong from 16,000 Mexican currency to 76,256 US currency.
resolution, authorized one of the directors, Atty. Joaquin Briones, to During the negotiations regarding the purchase of the shares of stock of
assign the properties of PAMBUSCO. Pursuant to the resolution, Briones Strong, not one word of the facts affecting the value of this stock was
assigned PAMBUSCO’s assets to Marcelino Enriquez. Enriquez, knowing made known to her nor her agent, Jones. After the sale of Dominican lands
that the properties were previously mortgaged and foreclosed, exercised and after the purchase of the 800 shares of Strong, Repide became the
PAMBUSCO’s right to redeem. So in August 1975, he redeemed the said owner of 30,400 out of the 42,030 shares of Philippine Sugar Estates.
properties and thereafter he sold them to Rising Yap.
Yap then registered the properties under his name. He then demanded Strong filed a complaint for the recovery of her 800 shares. She
Peña to vacate the properties. Peña refused to do hence Yap filed a argued that her agent Jones had no authority to sell her shares and that
complaint. In her defense, Peña averred that Yap acquired no legal title Repide fraudulently concealed the facts affecting their value.
over the property because the board resolution issued by PAMBUSCO in
November 1974 is void; that it is void because the resolution was issued ISSUE:
without a quorum; that there was no quorum because under the by-laws of Was there fraud in effecting the purchase of Strong’s shares?
PAMBUSCO, a quorum constitutes the presence of 4 out of 5 directors yet
the meeting was only attended by three directors. As such, the authority RULING:
granted to Briones to assign the properties is void; that the subsequent Yes. With the factual circumstances of this case, it became the
assignment by Briones to Enriquez is void; that Enriquez acquired no title duty of Repide, acting in good faith, to state the facts before making the
hence, likewise, Yap acquired no title. Yap insists that Peña has no legal purchase of Strong’s shares. That Repide was one of the directors of
standing to question the board resolution because she is not a stockholder. Philippine Sugar Estates was but one of the facts upon which liability is
ISSUE: Whether or not the board resolution is valid. asserted. He was not only a director, but he owned three-fourths of the
HELD: No, it is void. The by-laws are the laws of the corporation. shares of its stock, and was, at the time of the purchase of the stock,
PAMBUSCO’s by-laws provides that a quorum consists of at least four administrator general of the company with large powers and engaged in
directors. Hence, the meeting attended by only three directors did not the negotiations which finally led to the sale of the company’s lands at a
comply with the required quorum. As such, the three directors were not price which greatly enhanced the value of the stock. He was the negotiator
able to come up with a valid resolution which could bind the corporation. for the sale of the Dominican lands and was acting substantially as the
Anent the issue of Peña being a third person, she can question the board agent of the shareholders of Philippine Sugar Estates by reason of his
resolution. The resolution here is liken to a contract. Under the law, a ownership of the shares in the company. Because of such ownership and
person who is not a party obliged principally or subsidiarily in a contract agency, no one knew as well as he does about the exact condition of the
may exercise an action for nullity of the contract if he or she is prejudiced negotiations. He was the only one who knew of the probability of the sale
in his or her rights with respect to one of the contracting parties, and can of the Dominican lands to the government and of the probable purchase
show the detriment which would positively result to him or her from the price. Under these circumstances, Repide employed an agent to purchase
contract in which he or she had no intervention. the stock of Strong, concealed his own identity and his knowledge of the
Further, the sale of the properties of PAMBUSCO did not comply with the state of negotiations and their probable result. The concealment of his
procedure laid down by the Corporation Law. Under the law, the sale or identity while procuring the purchase of the stock, by his agent, was in
disposition of an and/or substantially all properties of the corporation itself strong evidence of fraud on the part of Repide. By such means, the
requires, in addition to a proper board resolution, the affirmative votes of more easily was he able to avoid questions relative to the negotiations for
the stockholders holding at least two-thirds (2/3) of the voting power in the sale of Dominican lands and actual misrepresentations regarding that
the corporation in a meeting duly called for that purpose. No doubt, the subject. He kept up the concealment as long as he could by giving the
questioned resolution was not confirmed at a subsequent stockholders check of a third person Rueda Ramos, for the purchase money. This move
meeting duly called for the purpose by the affirmative votes of the of Repide was a studied and intentional omission to be characterized as
stockholders holding at least two-thirds (2/3) of the voting power in the part of the deceitful machinations to obtain the purchase without giving
corporation. any information whatever as to the state and probable result of the
Further still, the Supreme Court discovers a few other anomalies with negotiations and to obtain a lower price for the shares of Strong. After the
PAMBUSCO. One is that PAMBUSCO has been inactive since 1949 as purchase of stock, he continued negotiations for the sale of the Dominican
per the records provided by the Securities and Exchange Commission. Its lands as the administrator general and eventually entered into a contract of
general information sheet with the SEC has not been updated regularly sale. The whole transaction gives conclusive evidence of the
overwhelming influence Repide had in the negotiations and it is clear that Following a pre-trial conference which failed to bring the parties to an
the final consummation was in his hands at all times. amicable settlement, a Request for Admission[6] of material and relevant
facts therein stated was served by the petitioner on the respondent. In its
OBITER DICTUM: Reply[7] thereto, the respondent denied the alleged facts sought to be
The directors are declared to be mandatories of the society and admitted, claiming that the same are irrelevant and immaterial to the suit
that they are prohibited from acquiring by purchase, even at public or filed against it.
judicial auction, the property the administration or sale of which, may In the ensuing trial on the merits, the parties adduced their respective
have been entrusted to them, and that this is the extent of the prohibition. testimonial and documentary evidence.
SECOND DIVISION To buttress her allegation that the respondent as defendant in the case is
liable to her under the subject promissory note, petitioner, as plaintiff,
[ G.R. No. 146535, August 18, 2006 ] proffered in evidence the promissory note itself and her oral testimonies
NATIVIDAD G. REYES,PETITIONER, VS. RCPI EMPLOYEES that when the respondent credit union defaulted in the payment of its
CREDIT UNION, INC., RESPONDENT. obligation under said promissory note, David F. Halican, respondent's
President and Chairman of its Board, issued four (4) postdated PCIB
DECISION checks in her favor, which checks were all dishonored by the drawee bank
GARCIA, J.: when presented for payment; that when requested to make good the
Assailed and sought to be set aside in this petition for review dishonored checks, the respondent instead filed a complaint for estafa
on certiorari is the Decision[1] dated December 27, 2000 of the Court of against her before the City Prosecutor's Office of Quezon City (I.S. No.
Appeals (CA) in CA-G.R. CV No. 49720, reversing an earlier decision of 88-2693), thereunder alleging her misappropriation of corporate funds
the Regional Trial Court (RTC) of Caloocan City, Branch 125, in an while still treasurer of the respondent, which complaint was recommended
action for a sum of money with damages thereat commenced by petitioner for dismissal by the investigating fiscal for insufficiency of evidence, a
Natividad Reyes against the respondent, RCPI Employees Credit Union, recommendation duly approved by the City Prosecutor and against which
Inc., a duly registered credit union of RCPI employees organized and a petition for review was dismissed by the Department of Justice; that
existing under Philippine laws. thereafter, the respondent filed another estafa case against her (I.S. No. 90-
The facts: 108555) which was likewise dismissed by the City Prosecutor; that she
On December 8, 1986, David F. Halican, President and Chairman of the filed with the same office a criminal complaint for violation of Batas
Board of Directors of the respondent credit union, together with Nestor Pambansa Big. 22 (I.S. No. 91-7502) against David Halican in connection
Estremera, respondent's Accounting Officer, executed in favor of with the same dishonored postdated PCIB checks, but her complaint was
petitioner Natividad Reyes a promissory note[2] purportedly for and in dismissed because she, as the payee in those checks, is also a signatory
behaif of the respondent. In full, the note reads: thereto; and that the respondent's board of directors was informed of the
PROMISSORY NOTE loan covered by the promissory note in question.
P 162. 338.52 Anent the respondent's counterclaim, which is for the same amount
For value received, we promise to pay Mrs. Natividad G. Reyes the sum of allegedly misappropriated by her. the petitioner testified that it is the
ONE HUNDRED SIXTY-TWO THOUSAND THREE HUNDRED respondent which has an account payable to her since 1987 as shown by
THIRTY-EIGHT & 52/100 (P162,338.52) only payable on or before the very same Financial Audit Report[8] referred to by the respondent in its
March 8, 1987. Answer.
In case of default, interest at the rate of 2% a month will be charged and For its part, the respondent credit union, maintaining its main defense that
liquidated damages in the amount of P15,000.00 will be paid by the David F. Halican had no authority to sign the subject promissory note for
maker/s plus 10% of the entire amount as and for attorney's fees, should and in its behalf, adduced in evidence the testimonies of two (2) of the
the matter be referred to a members of its board of directors, namely Rolando Babar and Hector
lawyer. Bolano. to disprove petitioner's claim that its board knew of the loan
contracted by Halican and of the latter's execution of the subject
RCPI promissory note. In support of its counterclaim, the respondent presented
Building, the four (4) postdated PCIB checks drawn and signed by Halican and
Cubao, payable to the petitioner, as well as the Financial Audit Report earlier
Quezon City. adverted to, showing that the petitioner incurred an accountability in the
December 8, amount of P-l,049,515.70 while still the respondent's treasurer in custody
1986. of its funds and checks and herself a signatory to its checks together with
Halican as the respondent's former president.
RCPI On November 2, 1994, the trial court came out with its
EMPLOYEES decision[9] rendering judgment for the petitioner, thus:
CREDIT WHEREFORE, premises considered, judgment is hereby rendered as
UNION, INC. follows:
By: Ordering defendant [respondent] to pay plaintiff [petitioner] the sum of
P162,338.52 representing the principal obligation;
(SIGNED)
DAVID Ordering defendant to pay plaintiff the sum of P1 84,680.00 representing
F.HALICAN stipulated interest from April 1987 to. December 1991, plus the sum
President & equivalent to 2% monthly interest from January. 1992 until the entire
Chairman of the amount is fully paid;
Board
Ordering the defendant to pay plaintiff the sum of P15,000.00 as
ROLANDO O. liquidated damages;
BABAR
Vice Chairman Ordering the defendant to pay plaintiff the sum of P36,2]0.00 as attorney's
fees; and
(SIGNED)
NESTOR Costs of suit.
E.ESTREMERA SO ORDERED. (Words in brackets added).
Accounting In its decision, the trial court ruled that Halican's authority to execute the
Officer same promissory note for and in behalf of the respondent was impliedly
Unable to collect the amount of the note on its due date despite repeated admitted by the latter when it failed to deny under oath the matters alleged
demands therefor, the petitioner filed with the RTC of Caloocan City a in the petitioner's Request for Admission, supra, particularly paragraph B
complaint[3] for a sum of money with damages against the respondent thereof, reading as follows:
credit union. xxx xxx xxx
In its Answer,[4] the respondent, as defendant, denied any obligation to the
petitioner asserting that it "did not authorize" the signatories to the
promissory note sued upon "to act for and in behalf of the association," On December 8, 1986, the RCPI Employees Credit
hence the plaintiff has no cause of action against it. In the same Answer, Union, Inc. through Mr. David F. Halican and Nestor
the respondent interposed a counterclaim in the sum of P1,049,51 Estremera, executed a Promissory Note wherein they
5.70,[5] representing the amount allegedly misappropriated by the latter B. promised to pay Mrs. Natividad G. Reyes the sum of
while serving as respondent's treasurer from 1981 to 1987, as allegedly P162,338.00 on or before March 8, 1987. (The copy
shown in the Financial Audit Report prepared and submitted by the of this document is attached to the complaint as
Philippine Federation of Credit Cooperatives, inc., which conducted an Annex "A").
audit of respondent's financial condition.
In the same decision, the trial court virtually dismissed the respondent's Petitioner insists, however, that the respondent credit union is in estoppel
counterclaim for lack of factual and legal bases. to disclaim Halican's authority to secure the loan and execute the
From the adverse decision of the trial court, the respondent credit union promissory note evidencing it. In this respect, petitioner would want us to
went on appeal to the CA in CA-G.R. CV No. 49720. take into account the following: (1) the respondent's admission that at the
As stated at the threshold hereof, the CA, in its Decision [10] dated time of the execution of the note,-David Halican was its president and
December 27, 2000, reversed that of the trial court and ordered the chairman of its board of directors; (2) the undisputed fact that Halican
dismissal of the petitioner's complaint, and likewise adjudged the signed the four (4) dishonored postdated PCIB checks as payment for the
petitioner liable to the respondent on the latter's counterclaim. TotheCA, promissory note; and (3) that in the case for violation of B.P. Big. 22 filed
the petitioner failed (l)to prove that David F. Halican was authorized by by her against Halican, the latter did not state by way of defense that he
the board of directors of the respondent credit union to borrow money, was not authorized to sign the dishonored checks therein involved but
much less execute the disputed promissory note in its behalf; and (2) to merely relied on the fact that petitioner cannot sue him under B.P. Big. 22
refute by convincing evidence the existence of her liability to the because she herself is also a signatory to those checks.[15]
respondent by way of counterclaim which the latter has proven by As a general rule, the acts of corporate officers within the scope of their
preponderance of evidence. Specifically, the CA decision dispositively authority are binding on the corporation, but when these officers exceeded
reads: their authority, their actions cannot bind the corporation, unless it has
WHEREFORE, premises considered, the appealed decision of the ratified such acts or is estopped from disclaiming them.[16]
Regional Trial Court of Calookan City, Branch 125 in Civil Case No. Again, petitioner has not shown that the respondent credit union ratified,
15427 is hereby REVERSED AND SET ASIDE and a new one entered expressly or impliedly, the act of Halican in executing and signing in its
DISMISSING the Complaint against the defendant-appellant and finding behalf the promissory note in dispute. Indeed, such ratification cannot be
the plaintiff-appellee LIABLE to pay to the former the amount of inferred from the aforementioned circumstances proffered by the
P1,041,595.70 plus legal interest at the rate of 6% per annum to be petitioner. The fact that the respondent admitted Halican to be its president
computed from judicial demand, i.e., September 11, 1992 until the same at the time the promissory note was executed; that Halican signed the
shall have been fully paid. postdated PCIB checks as payment for the note; that the said checks were
Costs against the plaintiff-appellee. dishonored by the drawee bank upon presentment for payment; and that
SO ORDERED. Halican failed to raise as a defense his lack of authority to sign the note in
Hence, petitioner's present recourse on her main submission that the the B.P. Big. 22 case filed against him, does not necessarily mean that the
challengedCA decision is contrary to law and the evidence on record. respondent credit union has thereby already ratified Halican's act of
The petition is partly meritorious. contracting the obligation under the same promissory note.
To the mind of the Court, two (2) issues commend themselves for its Stressing her posture of implied ratification, petitioner argues that the
resolution, to wit: (1) whether or not the respondent credit union is liable respondent is deemed to have impliedly admitted Halican's authority when
to the petitioner on the subject promissory note executed and signed by its it did not deny under oath the matters sought to be admitted in her Request
officers, namely, its president, David F. Halican and its accounting officer, for Admission, supra.
Nestor F. Estremera; and (2) whether or not the petitioner is liable to the The argument does not persuade.
respondent on the latter's counterclaim. A perusal of the pertinent paragraph of the petitioner's aforementioned
The Court resolves both issues in the negative. pleading reveals that the same is a mere reiteration of that alleged in
Indisputably, the respondent is a credit cooperative duly organized and paragraph B of her complaint[17] which the respondent credit union has
existing under Philippine laws. As such corporate entity, it has its own acts already traversed and denied in its Answer. Thus, the rule laid down in Po
and liabilities and exercises corporate powers, including the power to enter v. Court of Appeals[18] is apropos, viz:
into all contracts, through its board of directors pursuant to Section 23 of "A party should not be compelled to admit matters of fact already
the Corporation Code.[11] admitted by his pleading and concerning which there is no issue (Sherr vs.
Hornbook is the rule that a corporation, like the respondent, may act only East, 71 A2d, 752, Terry 260, cited in 27 C.J.S. 91), nor should he be
through its board of directors or, when authorized either by its by-laws or required to make a second denial of those already denied in his
by board resolution, through its officers or agents in the normal course of answer to the complaint. A request for admission is not intended to
business.[12] It is important, however, that for such corporate officers or merely reproduce or reiterate the allegations of the requesting party's
agents to be deemed fully clothed by the corporation to exercise a power pleading but should set forth relevant evidentiary matters of fact, or
of the board, the latter must specially authorize them to do so. [13] documents described in and exhibited with the request, whose purpose is
Here, the respondent denies that it ever authorized its officers, in to establish said party's cause of action or defense. Unless it serves that
particular, its president, David Halican, to contract a loan with the purpose, it is, as correctly observed by the Court of Appeals, "pointless,
petitioner and execute a promissory note in connection therewith. With useless," and "a mere redundancy."' (Emphasis supplied.)
that denial, it behooves upon the petitioner to establish by the requisite We are, however, with the petitioner insofar as the CA adjudged her liable
quantum of proof that Halican was in fact authorized by the respondent to to the respondent on the latter's counterclaim. In this respect, the Court
represent and bind it in the questioned transaction. Unfortunately for the finds the trial court's evaluation of the evidence more in accord with those
petitioner, she failed to discharge her burden. As it is, the evidence extant on record. Says the trial court in its decision:
adduced by her is bereft of any proof of authority on the part of Halican A review of the evidence submitted by the [respondent] revealed that the
and Estremera, either by way a provision on the respondent's by-laws or a conclusion of the Auditors who examined the documents were based
board resolution, to contract the alleged loan and to execute relative mainly on conjectures and inferences and stripped of factual and legal
thereto the promissory note in dispute. This being so, Halican's act of basis as correctly established by the investigating fiscal m his resolution
executing and signing the subject promissory note cannot bind the Exhibit "F", taking into account the procedures adopted and followed by
respondent credit union. So it is that in People's Aircargo and the credit cooperative in the conduct of its business and the relevant
Warehousing Co., Inc. v. CA,[14]the Court made it clear that in the absence evidence submitted in this case notably those reports of the supervisory
of authority from the board of directors, no person, not even its officers, committee, the internal auditor who conducted regular monthly audit. The
can validly bind a corporation. We thus lend concurrence to the CA's apt conclusion of the Auditors that the [petitioner] has incurred deficiencies
observations in the decision under review, thus: because from their audit there are no records which supports some of the
Indeed, the evidence submitted by the [petitioner] to prove her claim is disbursements are not worthy of credence because if the reco'rds were not
insufficient to establish the fact that [respondent] is indebted to it for x x x sufficient it should not have passed unnoticed by the credit committee, the
it has been held that the power to borrow money is one of those cases supervisory committee, the accounting officer and the Chairman, unless a
where even a special power of attorney is required. Such being the case, conspiracy existed between them. Indeed, it will not be unnoticed by the
there is invariably a need of an enabling act of the corporation to be internal auditor of the [respondent] who has conducted regular monthly
approved by its Board of Directors. This fact is what the trial court omitted and at times on the spot audit.
to consider. It failed to recognize the fact that while [petitioner] Finally, assuming without conceding that the records of the transaction of
sufficiently established the fact that the President and Chairman of the the business of the [respondent] was not sufficient to support some
Board of Directors of the [respondent] as well as its Accounting Officer, disbursements at the time the External Auditor conducted its audit, but
had signed the promissory note, she however dismally failed to prove that were found sufficient at the time the Internal Auditor (Supervisory
Halican was, in the first place, authorized to borrow money by the Board Committee) has conducted its own audit no blame much less liability
of Directors of the defendant corporation. Much less, execute a promissory maybe imputed to the [petitioner] inasmuch as she is not the record
note in behalf of the said corporation promising to pay the loaned amount custodian of the [respondent].[19] (Words in brackets supplied.)
at a stipulated date. We note that [petitioner] was also a member of the WHEREFORE, with the MODIFICATION that the petitioner
Board of Directors which allegedly resolved to allow the corporation to is ABSOLVED of any liability to the respondent as regards the latter s
borrow money from outside sources and such being the case, she could counterclaim, the challenged CA decision is hereby AFFIRMED.
have just presented said board resolution to prove that Halican was No pronouncement as to costs.
authorized to borrow money as it can be fairly presumed that she had SO ORDERED.
access to copies of the defendant corporation's board resolution. Failing in
this respect, [petitioner's] action was left without any leg to stand on RANIEL VS. JOCHICO
insofar as the claimed liability of the [respondent] is concerned. (Words in (G.R. No. 153413, March 2, 2007)
brackets added).
DOCTRINE: The directors may appoint officers and agents and as undertake to supply from Paris the films. The board of directors approved
incident to this power of appointment, they may discharge those and accepted the offer. The most important portion of the two letters of
appointed. acceptance written by Fernandez to Ramirez is in the following terms:
“These communications were signed in the following form, in which it
FACTS will be noted the separate signature of RJ Fernandez, as an individual, is
placed somewhat below and to the left of the signature of the Orientalist
• Nectarina Raniel and Victoria Pag-ong, are 2 out of the 5 Company, as signed by RJ Fernandez, in the capacity of treasurer:
directors of Nephro Systems Dialysis Center.
THE
Note: Raniel was Corporate Secretary, Treasurer, and Administrator of the
ORIENTALIST COMPANY,
Dialysis Clinic.
By RJ
• Petitioners questioned respondents’ plan to enter into a joint
venture with the Butuan Doctors’ Hospital and College. Respondents Fernandez
allegedly tried to compel them to waive and assign their shares with
Nephro but petitioners refused.
Treasurer.
• Raniel sought an indefinite leave of absence. Paul Jochico
disapproved the request, but Raniel nonetheless stopped reporting for RJ Fernandez”
work. When asked for an explanation for her absence, Raniel expressed
her sentiments over the disapproval of leave, and the joint venture with
Butuan.
Note: Without Raniel, holding three important positions, the company’s The record showed that JF Ramirez himself procured the films upon his
operations were disrupted. Such also warranted loss of the Board’s own responsibility. Thus, the only contracting parties in this case are JF
confidence in her. - SC Ramirez (first party), and Orientalist with RJ Fernandez (second party).
The films arrived in Manila but Orientalist had no funds to meet its
• Jochico issued a Notice of Special Board Meeting. Petitioners obligations. Hence, the first few drafts were accepted in the name of
were notified, but they did not attend. The board passed several Orientalist by its president B Hernandez, and were taken up by him with
resolutions ratifying the disapproval of Raniel’s request for leave, his own funds. As the drafts had been paid by Hernandez, he treated the
dismissing her as Administrator of Nephro, declaring the position of films as his own property, and they never came into the actual possession
Corporate Secretary vacant. of Orientalist as owner at all. Hernandez rented the films to Orientalist and
they were exhibited by it in the Oriental Theater under an arrangement
• Otelio Jochico was appointed as the new Corporate Secretary, made between him and the theater’s manager. Several remittances of films
and a Special Stockholders’ Meeting was held. from Paris arrived. All of the drafts accompanying these films were drawn
upon the Orientalist Company; and all were accepted in the name of
• Again, petitioners did not attend. The stockholders that were Orientalist by its president, B Hernandez, except the last which was
present removed the petitioners as directors of Nephro. accepted by Hernandez individually. None of the drafts thus accepted
Note: Raniel could have explained herself during these meetings, but she were taken up by the drawee or by Hernandez when they fell due; and it
chose not to attend. - SC was finally necessary for Ramirez to take them up as dishonoured by non-
payment.
• Side-note: the ownership of the outstanding capital stock is
distributed in this manner; Ramirez instituted an action against Orientalist and RJ Fernandez. Upon
Jochico – 200 Shares application of Ramirez, the films were sold and the amount realized from
Steffens – 100 Shares the sale was applied to the satisfaction of the plaintiff’s claim. Judgment
Viriya – 100 Shares was given for the balance due to Ramiez. Orientalist was declared to be a
Raniel – 25 Shares principal debtor and Fernandez was declared to be subsidiarily liable as
Pag-ong – 75 Shares guarantor. Defendants appealed. The Court noted that the action is
= 500 Shares primarily founded upon the liability created by the two acceptance letters.
2/3 of OCS is 333.33 Shares
400 Shares voted for petitioners’ removal ISSUES:
1. WON Fernandez’s actions bound the company .
• Petitioners filed a case with the SEC, which held that the 2. WON the company is still liable, assuming that the company
removal of petitioners was valid. Appeal made to the CA, which affirmed was able to deny the authority of Fernandez.
the SEC decision. 3. What is the character of liability assumed by Fernandez?

ISSUE HELD:
1. YES. The corporation was not able to deny the genuineness and due
WON the petitioners’ removal was valid? YES. execution of the contracts in question and the authority of Fernandez
to bind the Orientalist Company. Sec. 103 of the Code of Civil
HELD Procedure requires that the Answer setting up the defense of lack of
authority of an officer of a corporation to bind it by a contract should
• The SC ruled in favor of Jochico. be verified and the denial contemplated must be specific. In this case,
the failure of the corporation to make any issue in its answer with
• The directors may appoint officers and agents and as incident to regard to the authority of Fernandez to bind it, and particularly its
this power of appointment, they may discharge those appointed. failure to deny specifically under oath the genuineness and due
Note: Raniel was removed from her positions before she was ousted from execution of the contracts sued upon, have the effect of eliminating
her status as a director. the question of his authority from the case.

• The unanimous resolutions carried by the board during such Whether a particular officer actually possesses the authority which he
meetings are valid and binding against complainants. assumes to exercise is frequently known to very, very few and the proof of
Note: There was quorum, given that there were only 5 directors, 3 voted to it usually is not readily accessible to the stranger who deals with the
remove petitioners. corporation on the faith of the ostensible authority exercised by some of
the corporate officers.
PETITION DENIED
2. YES. If a corporation knowingly permits one of its officers, or any
Ramirez v. Orientalist Co. and Fernandez (1918) – Street, J. other agent, to do acts within the scope of an apparent authority and
Concept: Control and Management of Corporation thus holds him out to the public as possessing power to do those acts,
the corporation will, as against anyone who has in good faith dealt
FACTS: with the corporation through such agent, be estopped from denying
Orientalist Company (Orientalist for brevity) exhibited films in a theatre in his authority; and where it is said “if the corporation permits” this
Manila. Plaintiff JF Ramirez, a resident of Paris and represented in Manila means the same as “if the thing is permitted by the directing power of
by his son Jose Ramirez, was engaged in business of marketing films for the corporation.”
manufacturers and in the production or distribution of cinematographic
material. In 1913, there were negotiations between the officials of The stockholders adopted a resolution to the effect that the agencies of the
Orientalist and Jose Ramirez, as agent of JF Ramirez, for the exclusive two films should be accepted if the corporation could obtain the money
agency of two films in the hands of Orientalist. Jose Ramirez placed a with which to meet the expenditure involved, and to this end appointed a
formal offer stating in detail the terms upon which Ramirez would
committee to apply to the bank for a credit. An attempt to obtain credit application for listing in the PSE of the private respondent PALI, since
was made, but failed. Another special meeting of stockholders was held this is a matter addressed to the sound discretion of the PSE, a corporation
and a resolution was passed to the effect that the company should pay to entity, whose business judgments are respected in the absence of bad faith.
Hernandez, Fernandez, Monroy and Papa an amount equal to 10% of their
outlay in importing the films, said payment to be made in shares of the The Court also finds that the private respondent PALI, on at least two
company. At the time this meeting was held three shipments of the film points (nos. 1 and 5) has failed to support the propriety of the issue of its
had already been received in Manila. Therefore, the body was then shares with unfailing clarity, thereby lending support to the conclusion
cognizant that the offer had already been accepted in the name of that the PSE acted correctly in refusing the listing of PALI in its stock
Orientalist Company and that the films which were then expected to arrive exchange.
were being imported by virtue of such acceptance.
(1) The registration statement is on its face incomplete or inaccurate
3. In affixing his signature to the contracts, Fernandez was a guarantor. in any material respect or includes any untrue statement of a material fact
From the testimony of both Ramirez and Fernandez, the Court was or omits to state a material facts required to be stated therein or necessary
convinced that the responsibility of the later was that of a guarantor. to make the statements therein not misleading;
Fernandez said that his name was signed as a guaranty that the (5) The issuer or registrant has not shown to the satisfaction of the
contract would be approved by the corporation, while Ramirez said Commission that the sale of its security would not work to the prejudice to
that the name was put on the contract for the purpose of guaranteeing the public interest or as a fraud upon the purchaser or investors.
its performance. The Court believed that the latter was the real
intention of the parties. Dispositive: The decisions of the Court of Appeals and the Securities and
Exchage Commission dated July 27, 1996 and April 24, 1996,
PSE v. Court of Appeals respectively, are hereby REVERSED and SET ASIDE, and a new
G.R. No. 125469 281 SCRA 232 Judgment is hereby ENTERED, affirming the decision of the Philippine
October 27, 1997 Stock Exchange to deny the application for listing of the private
By: Karen P. Lustica respondent Puerto Azul Land, Inc.

Facts: The Puerto Azul Land, Inc. (PALI) is a domestic real estate Prime White Cement Corporation vs Intermediate Appellate Court
corporation. PALI sought to offer its shares to the public in order to raise 220 SCRA 103 – Commercial Law – Corporation Code – Award of
funds allegedly to develop its properties and pay its loans with several Moral Damages to Corporations – Self-Dealing Director
banking institutions. In July 1969, Zosimo Falcon and Justo Trazo entered into an agreement
with Alejandro Te whereby it was agreed that from 1970 to 1976, Te shall
PALI was issued a Permit to Sell its shares to the public by the Securities be the sole dealer of 20,000 bags Prime White cement in Mindanao.
and Exchange Commission (SEC). To facilitate the trading of its shares Falcon was the president of Prime White Cement Corporation (PWCC)
among investors, PALI sought to course the trading of its shares through and Trazo was a board member thereof. Te was likewise a board member
the Philippine Stock Exchange, Inc. (PSE), for which purpose it filed with of PWCC. It was agreed that the selling price for a bag of cement shall be
the said stock exchange an application to list its shares, with supporting P9.70.
documents attached. Before the bags of cement can be delivered, Te already made known to the
public that he is the sole dealer of cements in Mindanao. Various
The Listing Committee of the PSE, upon a perusal of PALI's application, hardwares then approached him to be his sub-dealers, hence, Te entered
recommended to the PSE's Board of Governors the approval of PALI's into various contracts with them.
listing application. But then apparently, Falcon and Trazo were not authorized by the Board
of PWCC to enter into such contract. Nevertheless, the Board wished to
Before it could act upon PALI's application, the Board of Governors of the retain the contract but they wanted some amendment which includes the
PSE received a letter from the heirs of Ferdinand E. Marcos, claiming that increase of the selling price per bag to P13.30 and the decrease of the total
the late President Marcos was the legal and beneficial owner of certain amount of cement bags from 20k to 8k only plus the contract shall only be
assets of PALI which likewise appears to have been held and continue to effective for a period of three months and not 6 years.
be held in trust by one Rebecco Panlilio for then President Marcos. Te refused the counter-offer. PWCC then awarded the contract to someone
else.
PALI wrote a letter to the SEC addressed to the then Acting Chairman, Te then sued PWCC for damages. PWCC filed a counterclaim and in said
Perfecto R. Yasay, Jr., bringing to the SEC's attention the action taken by counterclaim, it is claiming for moral damages the basis of which is the
the PSE. SEC rendered its Order, reversing the PSE's decision. SEC claim that Te’s filing of a civil case against PWCC destroyed the
ordered to immediately cause the listing of the PALI shares in the company’s goodwill. The lower court ruled in favor Te.
Exchange. ISSUE: Whether or not the ruling of the lower court is correct.
HELD: No. Te is what can be called as a self-dealing director – he deals
The CA rendered the decision that SEC had both jurisdiction and authority business with the same corporation in which he is a director. There is
to look into the decision of the petitioner PSE, for the purpose of ensuring nothing wrong per se with that. However, Sec. 32 provides that:
fair administration of the exchange. Both as a corporation and as a stock SEC. 32. Dealings of directors, trustees or officers with the corporation.
exchange, the petitioner is subject to public respondent's jurisdiction, —- A contract of the corporation with one or more of its directors or
regulation and control. PALI complied with all the requirements for trustees or officers is voidable, at the option of such corporation, unless all
public listing, affirming the SEC's ruling. the following conditions are present:
1. That the presence of such director or trustee in the board meeting in
which the contract was approved was not necessary to constitute a quorum
Issue: WON SEC has the authority to order the PSE to list the shares of for such meeting;
PALI in the stock exchange. 2. That the vote of such director or trustee was not necessary for the
approval of the contract;
Held: YES. 3. That the contract is fair and reasonable under the circumstances; and
4. That in the case of an officer, the contract with the officer has been
Ratio: A corporation is but an association of individuals, allowed to previously authorized by the Board of Directors.
transact under an assumed corporate name, and with a distinct legal In this particular case, the Supreme Court focused on the fact that the
personality. In organizing itself as a collective body, it waives no contract between PWCC and Te through Falcon and Trazo was not
constitutional immunities and perquisites appropriate to such a body. reasonable. Hence, PWCC has all the rights to void the contract and look
for someone else, which it did. The contract is unreasonable because of the
Section 3 of Presidential Decree 902-A, standing alone, is enough very low selling price. The Price at that time was at least P13.00 per bag
authority to uphold the SECs challenged control authority over the and the original contract only stipulates P9.70. Also, the original contract
petitioner PSE even as it provides that the Commission shall have absolute was for 6 years and there’s no clause in the contract which protects PWCC
jurisdiction, supervision, and control over all corporations, partnerships or from inflation. As a director, Te in this transaction should protect the
associations, who are the grantees of primary franchises and/or a license or corporation’s interest more than his personal interest. His failure to do so
permit issued by the government to operate in the Philippines. is disloyalty to the corporation.
Anent the issue of moral damages, there is no question that PWCC’s
The SECs power to look into the subject ruling of the PSE, therefore, may goodwill and reputation had been prejudiced due to the filing of this case.
be implied from or be considered as necessary or incidental to the carrying However, there can be no award for moral damages under Article 2217 of
out of the SECs express power to insure fair dealing in securities traded the Civil Code in favor of a corporation.
upon a stock exchange or to ensure the fair administration of such NOTE: In a later case, Coastal Pacific Trading, Inc. vs Southern Rolling
exchange. Mills Co., Inc. (July 28, 2006), it was ruled that a corporation may be
entitled to moral damages provided that its good reputation was debased
However, in the present case, the Court finds that the SEC had acted resulting in its humiliation in the business realm.
arbitrarily in arrogating unto itself the discretion of approving the
Premium Marble Resources Inc. v. Court of Appeals Please be informed that we have approved the loan application of
GR NO 96551 November 4, 1996 ARIZONA TRANSPORT CORP. and PANACOR MARKETING
CORPORATION. Both represented by MR. PEDRO P. PANALIGAN
Facts: (hereinafter the BORROWERS) in the principal amount of PESOS:
Premium Marble Resources, Inc. (PREMIUM for brevity), assisted by SEVEN MILLION FIVE HUNDRED THOUSAND ONLY
Atty. Dumadag as counsel, filed an action for damages against (P7,500,000.00) Philippine Currency. The loan shall be secured by a Real
International Corporate Bank (ICB). Estate Mortgage over a parcel of land located at #777 Nueve de Pebrero
Meantime, the same corporation, i.e., PREMIUM, but this time St. Bo. Mauway, Mandaluyong City, Metro Manila covered by TCT No.
represented by Siguion Reyna, Montecillio and Ongsiako Law Office as 3475 and registered under the name of Arizona Haulers, Inc. which is
counsel, filed a motion to dismiss (MTD) on the ground that the filing of presently mortgaged with your bank.
the case was without authority from its duly constituted board of directors
as shown by the excerpt of the minutes of PREMIUM’s board of directors’ The borrowers have authorized IBA FINANCE CORP. to pay Premiere
meeting. In its opposition to the MTD, PREMIUM thru Atty. Dumadag Bank from the proceeds of their loan. The disbursement of the loan,
contended that the persons who signed the board resolution are not however is subject to the annotation of our mortgage lien on the said
directors of the corporation and were allegedly former officers and property and final verification that said title is free from any other lien or
stockholders of PREMIUM who were dismissed for various irregularities encumbrance other than that of your company and IBA Finance
and fraudulent acts; Corporation.
On the other hand, Siguion Reyna Law firm as counsel of PREMIUM in a
rejoinder, asserted that it is the general information sheet filed with the In order to register the mortgage, please entrust to us the owner's duplicate
SEC, among others, that is the best evidence that would show who are the copy of TCT No. 3475, current tax declaration, realty tax receipts for the
stockholders of a corporation and not the AOI since the latter does not current year and other documents necessary to affect annotation thereof.
keep track of the many changes that take place after new stockholders
subscribe to corporate shares of stocks. Upon registration of our mortgage, we undertake to remit directly to you
or your authorized representative the amount equivalent to the Borrower's
Issue: outstanding indebtedness to Premiere Bank as duly certified by your
Whether the filing of the action was with authorization from the BOD goodselves provided such an amount shall not exceed PESOS: SIX
MILLION ONLY (P6,000,000.00) and any amount in excess of the
Held: aforestated shall be for the account of the borrowers. It is understood that
NO. By express mandate of the Corporation Code, all corporations duly upon receipt of payment, you will release to us the corresponding
organized pursuant thereof are required to file with SEC the names, cancellation of your mortgage within five (5) banking days therefrom.
nationalities and residence of the directors and officers elected. In
determining whether the filing of an action was authorized by the board, it If the foregoing terms and conditions are acceptable to you, please affix
is the list of directors in the latest general information sheet as filed with your signature provided below and furnish us a copy of the Statement of
the SEC which is controlling. Account of said borrowers.
On October 12, 1995, Premiere Bank sent a letter-reply[7] to Iba-Finance,
[ GR No. 159352, Apr 14, 2004 ] informing the latter of its refusal to turn over the requested documents on
PREMIERE DEVELOPMENT BANK v. CA the ground that Arizona had existing unpaid loan obligations and that it
was the bank's policy to require full payment of all outstanding loan
471 Phil. 704 obligations prior to the release of mortgage documents. Thereafter,
YNARES-SATIAGO, J.: Premiere Bank issued to Iba-Finance a Final Statement of
This is a petition for review under Rule 45 of the 1997 Rules on Civil Account[8] showing Arizona's total loan indebtedness. On October 19,
Procedure seeking the annulment of the Decision dated June 18, 2003 of 1995, Panacor and Arizona executed in favor of Iba-Finance a promissory
the Court of Appeals[1] which affirmed the Decision of the Regional Trial note in the amount of 7.5 million. Thereafter, Iba-Finance paid to Premiere
Court[2] in Civil Case No. 65577. Bank the amount of P6,235,754.79 representing the full outstanding loan
account of Arizona. Despite such payment, Premiere Bank still refused to
The undisputed facts show that on or about October 1994, Panacor release the requested mortgage documents specifically, the owner's
Marketing Corporation (Panacor for brevity), a newly formed corporation, duplicate copy of TCT No. T-3475.[9]
acquired an exclusive distributorship of products manufactured by Colgate
Palmolive Philippines, Inc. (Colgate for short). To meet the capital On November 2, 1995, Panacor requested Iba-Finance for the immediate
requirements of the exclusive distributorship, which required an initial approval and release of the remaining P2.5 million loan to meet the
inventory level of P7.5 million, Panacor applied for a loan of P4.1 million required monthly purchases from Colgate. Iba-Finance explained
with Premiere Development Bank. After an extensive study of Panacor's however, that the processing of the P2.5 million loan application was
creditworthiness, Premiere Bank rejected the loan application and conditioned, among others, on the submission of the owner's duplicate
suggested that its affiliate company, Arizona Transport Corporation copy of TCT No. 3475 and the cancellation by Premiere Bank of Arizona's
(Arizona for short),[3] should instead apply for the loan on condition that mortgage. Occasioned by Premiere Bank's adamant refusal to release the
the proceeds thereof shall be made available to Panacor. Eventually, mortgage cancellation document, Panacor failed to generate the required
Panacor was granted a P4.1 million credit line as evidenced by a Credit capital to meet its distribution and sales targets. On December 7, 1995,
Line Agreement.[4] As suggested, Arizona, which was an existing loan Colgate informed Panacor of its decision to terminate their distribution
client, applied for and was granted a loan of P6.1 million, P3.4 million of agreement.
which would be used to pay-off its existing loan accounts and the
remaining P2.7 million as credit line of Panacor. As security for the P6.1 On March 13, 1996, Panacor and Arizona filed a complaint for specific
million loan, Arizona, represented by its Chief Executive Officer Pedro performance and damages against Premiere Bank before the Regional
Panaligan and spouses Pedro and Marietta Panaligan in their personal Trial Court of Pasig City, docketed as Civil Case No. 65577.
capacities, executed a Real Estate Mortgage against a parcel of land
covered by TCT No. T-3475 as per Entry No. 49507 dated October 2, On June 11, 1996, Iba-Finance filed a complaint-in-intervention praying
1995.[5] that judgment be rendered ordering Premiere Bank to pay damages in its
favor.
Since the P2.7 million released by Premiere Bank fell short of the P4.1
million credit line which was previously approved, Panacor negotiated for On May 26, 1998, the trial court rendered a decision in favor of Panacor
a take-out loan with Iba Finance Corporation (hereinafter referred to as and Iba-Finance, the decretal portion of which reads:
Iba-Finance) in the sum of P10 million, P7.5 million of which will be
released outright in order to take-out the loan from Premiere Bank and the WHEREFORE, judgment is hereby rendered in favor of the plaintiff
balance of P2.5 million (to complete the needed capital of P4.1 million Panacor Marketing Corporation and against the defendant Premiere Bank,
with Colgate) to be released after the cancellation by Premiere of the ordering the latter to pay the former the following sums, namely:
collateral mortgage on the property covered by TCT No. T-3475. Pursuant
to the said take-out agreement, Iba-Finance was authorized to pay P4,520,000.00 in addition to legal interest from the time of filing of
1)
Premiere Bank the prior existing loan obligations of Arizona in an amount the complaint until full payment;
not to exceed P6 million. 2) P1,000,000.00 as and for exemplary damages;
3) P100,000.00 as and for reasonable attorney's fees; and
On October 5, 1995, Iba-Finance sent a letter to Ms. Arlene R. Martillano, 4) Costs of suit.
officer-in-charge of Premiere Bank's San Juan Branch, informing her of Similarly, judgment is hereby rendered in favor of plaintiff-in-intervention
the approved loan in favor of Panacor and Arizona, and requesting for the IBA-Finance Corporation as against defendant Premiere bank, as follows,
release of TCT No. T-3475. Martillano, after reading the letter, affixed her namely:
signature of conformity thereto and sent the original copy to Premiere
Bank's legal office. The full text of the letter reads:[6] Ordering defendant Premiere Bank to release to plaintiff-intervenor
1)
IBA-Finance Corporation the owner's duplicate copy of Transfer
Certificate of Title No. 3475 registered in the name of Arizona of the mortgage because they were rendered fait accompli by the
Haulers, Inc. including the deed of cancellation of the mortgage compromise agreement.
constituted thereon;
Ordering the defendant Premiere Bank to pay to Intervenor IBA- We are not persuaded.
2)
Finance, the following sums, to wit:
3) P1,000,000.00 as and by way of exemplary damages; and In a letter-agreement[12] dated October 5, 1995, Iba-Finance informed
4) P100,000.00 as and for reasonable attorney's fees; and Premiere Bank of its approval of Panacor's loan application in the amount
5) Costs of suit. of P10 million to be secured by a real estate mortgage over a parcel of
For lack of sufficient legal and factual basis, the counterclaim of defendant land covered by TCT No. T-3475. It was agreed that Premiere Bank shall
Premiere Bank is DISMISSED. entrust to Iba-Finance the owner's duplicate copy of TCT No. T-3475 in
order to register its mortgage, after which Iba-Finance shall pay off
SO ORDERED. Arizona's outstanding indebtedness. Accordingly, Iba-Finance remitted
Premiere Bank appealed to the Court of Appeals contending that the trial P6,235,754.79 to Premiere Bank on the understanding that said amount
court erred in finding, inter alia, that it had maliciously downgraded the represented the full payment of Arizona's loan obligations. Despite
credit-line of Panacor from P4.1 million to P2.7 million. performance by Iba-Finance of its end of the bargain, Premiere Bank
refused to deliver the mortgage document. As a consequence, Iba-Finance
In the meantime, a compromise agreement was entered into between Iba- failed to release the remaining P2.5 million loan it earlier pledged to
Finance and Premiere Bank whereby the latter agreed to return without Panacor, which finally led to the revocation of its distributorship
interest the amount of P6,235,754.79 which Iba-Finance earlier remitted to agreement with Colgate.
Premiere Bank to pay off the unpaid loans of Arizona. On March 11,
1999, the compromise agreement was approved. Undeniably, the not-so-forthright conduct of Premiere Bank in its dealings
with respondent corporations caused damage to Panacor and Iba-Finance.
On June 18, 2003, a decision was rendered by the Court of Appeals which It is error for Premiere Bank to assume that the compromise agreement it
affirmed with modification the decision of the trial court, the dispositive entered with Iba-Finance extinguished all direct and collateral incidents to
portion of which reads: the aborted take-out such that it also cancelled its obligations to Panacor.
The unjustified refusal by Premiere Bank to release the mortgage
WHEREFORE, premises considered, the present appeal is hereby document prompted Iba-Finance to withhold the release of the P2.5
DISMISSED, and the decision appealed from in Civil Case No. 65577 is million earmarked for Panacor which eventually terminated the
hereby AFFIRMED with MODIFICATION in that the award of distributorship agreement. Both Iba-Finance and Panacor, which are two
exemplary damages in favor of the appellees is hereby reduced to separate and distinct juridical entities, suffered damages due to the fault of
P500,000.00. Needless to add, in view of the Compromise Agreement Premiere Bank. Hence, it should be held liable to each of them.
plaintiff-intervenor IBA-Finance and defendant-appellant PREMIERE
between plaintiff-intervenor IBA-Finance and defendant-appellant While the compromise agreement may have resulted in the satisfaction of
PREMIERE as approved by this Court per Resolution dated March 11, Iba-Finance's legal claims, Premiere Bank's liability to Panacor remains.
1999, Our dispositive of the present appeal is only with respect to the We agree with the Court of Appeals that the "present appeal is only with
liability of appellant PREMIERE to the plaintiff-appellees. respect to the liability of appellant Premiere Bank to the plaintiffs-
appellees (Panacor and Arizona)"[13] taking into account the compromise
With costs against the defendant-appellant. agreement.

SO ORDERED.[10] For the foregoing reasons, we find that the Court of Appeals did not err in
Hence the present petition for review, which raises the following issues:[11] discussing in the assailed decision the abortive take-out and the refusal by
Premiere Bank to release the cancellation of the mortgage document.
I
Secondly, Premiere Bank asserts that it acted in good faith when it
WHETHER OR NOT THE DECISION OF HONORABLE COURT OF downgraded the credit line of Panacor from P4.1 million to P2.7 million. It
APPEALS EXCEEDED AND WENT BEYOND THE FACTS, THE cites the decision of the trial court which, albeit inconsistent with its final
ISSUES AND EVIDENCE PRESENTED IN THE APPEAL TAKING disposition, expressly recognized that the downgrading of the loan was not
INTO CONSIDERATION THE ARGUMENT OF PETITIONER BANK the proximate cause of the damages suffered by respondents.
AND ADVENT OF THE DULY APPROVED COMPROMISE
AGREEMENT BETWEEN THE PETITIONER BANK AND IBA Under the Credit Line Agreement[14] dated September 1995, Premiere
FINANCE CORPORATION. Bank agreed to extend a loan of P4.1 million to Arizona to be used by its
affiliate, Panacor, in its operations. Eventually, Premiere approved in
favor of Arizona a loan equivalent to P6.1 million, P3.4 million of which
II was allotted for the payment of Arizona's existing loan obligations and
P2.7 million as credit line of Panacor. Since only P2.7 million was made
WHETHER OR NOT THE ISSUES THAT SHOULD HAVE BEEN available to Panacor, instead of P4.1 million as previously approved,
RESOLVED BY THE HONORABLE COURT OF APPEALS, BY Panacor applied for a P2.5 loan from Iba-Finance, which, as earlier
REASON OF THE EXISTENCE OF THE COMPROMISE mentioned, was not released because of Premiere Bank's refusal to issue
AGREEMENT, IS LIMITED TO THE ISSUE OF ALLEGED BAD the mortgage cancellation.
FAITH OF PETITIONER BANK IN THE DOWNGRADING OF THE
LOAN AND SHOULD NOT INCLUDE THE RENDITION OF AN It is clear that Premiere Bank deviated from the terms of the credit line
ADVERSE PRONOUNCEMENT TO AN ALREADY FAIT agreement when it unilaterally and arbitrarily downgraded the credit line
ACCOMPLI- ISSUE ON THE REFUSAL OF THE BANK TO of Panacor from P4.1 million to P2.7 million. Having entered into a well-
RECOGNIZE THE TAKE-OUT OF THE LOAN AND THE RELEASE defined contractual relationship, it is imperative that the parties should
OF TCT NO. 3475. honor and adhere to their respective rights and obligations thereunder.
Law and jurisprudence dictate that obligations arising from contracts have
the force of law between the contracting parties and should be complied
III with in good faith.[15] The appellate court correctly observed, and we
agree, that:
WHETHER OR NOT PETITIONER ACTED IN BAD FAITH IN THE
DOWNGRADING OF THE LOAN OF RESPONDENTS TO SUPPORT Appellant's actuations, considering the actual knowledge of its officers of
AN AWARD OF ACTUAL AND EXEMPLARY DAMAGES NOW the tight financial situation of appellee PANACOR brought about
REDUCED TO P500,000.00. primarily by the appellant bank's considerable reduction of the credit line
portion of the loan, in relation to the "bail-out" efforts of IBA Finance,
whose payment of the outstanding loan account of appellee ARIZONA
IV with appellant was readily accepted by the appellant, were truly marked by
bad faith and lack of due regard to the urgency of its compliance by
WHETHER OR NOT THERE IS BASIS OR COMPETENT PIECE OF immediately releasing the mortgage cancellation document and delivery of
EVIDENCE PRESENTED DURING THE TRIAL TO SUPPORT AN the title to IBA Finance. That time is of the essence in the requested
AWARD OF ACTUAL DAMAGES OF P4,520,000.00. release of the mortgage cancellation and delivery of the subject title was
Firstly, Premiere Bank argues that considering the compromise agreement only too well-known to appellant, having only belatedly invoked the
it entered with Iba-Finance, the Court of Appeals should have ruled only cross-default provision in the Real Estate Mortgage executed in its favor
on the issue of its alleged bad faith in downgrading Panacor's credit line. It by appellee ARIZONA to resist the plain valid and just demand of IBA
further contends that the Court of Appeals should have refrained from Finance for such compliance by appellant bank. [16]
making any adverse pronouncement on the refusal of Premiere Bank to Premiere Bank cannot justify its arbitrary act of downgrading the credit
recognize the take-out and its subsequent failure to release the cancellation line on the alleged finding by its project analyst that the distributorship
was not financially feasible. Notwithstanding the alleged forewarning, In determining actual damages, the court cannot rely on mere assertions,
Premiere Bank still extended Arizona the loan of P6.1 million, albeit in speculations, conjectures or guesswork but must depend on competent
contravention of the credit line agreement. This indubitably indicates that proof and on the best evidence obtainable regarding the actual amount of
Premiere Bank had deliberately and voluntarily granted the said loan loss.[20]
despite its claim that the distributorship contract was not viable.
Even if not recoverable as compensatory damages, Panacor may still be
Neither can Premiere Bank rely on the puerile excuse that it was the bank's awarded damages in the concept of temperate or moderate damages. When
policy not to release the mortgage cancellation prior to the settlement of the court finds that some pecuniary loss has been suffered but the amount
outstanding loan obligations. Needless to say, the Final Statement of cannot, from the nature of the case, be proved with certainty, temperate
Account dated October 17, 1995 showing in no uncertain terms Arizona's damages may be recovered. Temperate damages may be allowed in cases
outstanding indebtedness, which was subsequently paid by Iba-Finance, where from the nature of the case, definite proof of pecuniary loss cannot
was the full payment of Arizona's loan obligations. Equity demands that a be adduced, although the court is convinced that the aggrieved party
party cannot disown it previous declaration to the prejudice of the other suffered some pecuniary loss.
party who relied reasonably and justifiably on such declaration.
The Code Commission, in explaining the concept of temperate damages
Thirdly, Premiere Bank avers that the appellate court's reliance on the under Article 2224, makes the following comment:[21]
credit line agreement as the basis of bad faith on its part was inadmissible
or self-serving for not being duly notarized, being unsigned in all of its left In some States of the American Union, temperate damages are allowed.
margins, and undated. According to Premiere Bank, the irregularities in There are cases where from the nature of the case, definite proof of
the execution of the credit line agreement bolsters the theory that the same pecuniary loss cannot be offered, although the court is convinced that
was the product of manipulation orchestrated by respondent corporations there has been such loss. For instance, injury to ones commercial credit or
through undue influence and pressure exerted by its officers on Martillano. to the goodwill of a business firm is often hard to show with certainty in
terms of money. Should damages be denied for that reason? The judge
Premiere Bank's posture deserves scant consideration. As found by the should be empowered to calculate moderate damages in such cases, rather
lower court, there are sufficient indicia that demonstrate that the alleged than that the plaintiff should suffer, without redress from the defendant's
unjust pressure exerted on Martillano was more imagined than real. In her wrongful act.
testimony, Martillano claims that she was persuaded and coaxed by Caday It is obvious that the wrongful acts of Premiere Bank adversely affected,
of Iba-Finance and Panaligan of Panacor to sign the letter. It was she who in one way or another, the commercial credit[22] of Panacor, greatly
provided Iba-Finance with the Final Statement of Account and accepted its contributed to, if not, decisively caused the premature stoppage of its
payment without objection or qualification. These acts show that she was business operations and the consequent loss of business opportunity. Since
vested by Premiere Bank with sufficient authority to enter into the said these losses are not susceptible to pecuniary estimation, temperate
transactions. damages may be awarded. Article 2216 of the Civil Code:

If a private corporation intentionally or negligently clothes its officers or No proof of pecuniary loss is necessary in order that moral, nominal,
agents with apparent power to perform acts for it, the corporation will be temperate, liquidated or exemplary damages may be adjudicated. The
estopped to deny that the apparent authority is real as to innocent third assessment of such damages, except liquidated ones, is left to the
persons dealing in good faith with such officers or agents.[17] As testified discretion of the Court, according to the circumstances of each case.
to by Martillano, after she received a copy of the credit line agreement and Under the circumstances, the sum of P200,000.00 as temperate damages is
affixed her signature in conformity thereto, she forwarded the same to the reasonable.
legal department of the Bank at its Head Office. Despite its knowledge,
Premiere Bank failed to disaffirm the contract. When the officers or agents WHEREFORE, the petition is DENIED. The Decision dated June 18,
of a corporation exceed their powers in entering into contracts or doing 2003 of the Court of Appeals in CA-G.R. CV No. 60750, ordering
other acts, the corporation, when it has knowledge thereof, must promptly Premiere Bank to pay Panacor Marketing Corporation P500,000.00 as
disaffirm the contract or act and allow the other party or third persons to exemplary damages, P100,000.00 as attorney's fees, and costs,
act in the belief that it was authorized or has been ratified. If it acquiesces, isAFFIRMED, with the MODIFICATION that the award of
with knowledge of the facts, or fails to disaffirm, ratification will be P4,520,000.00 as actual damages is DELETED for lack of factual basis. In
implied or else it will be estopped to deny ratification.[18] lieu thereof, Premiere Bank is ordered to pay Panacor P200,000.00 as
temperate damages.
Finally, Premiere Bank argues that the finding by the appellate court that it
was liable for actual damages in the amount of P4,520,000.00 is without SO ORDERED.
basis. It contends that the evidence presented by Panacor in support of its G.R. No. L-5883 November 28, 1953
claim for actual damages are not official receipts but self-serving DOMINGO PONCE AND BUHAY L. PONCE, petitioners,
declarations. vs.
DEMETRIO B. ENCARNACION, Judge of the Court of First
To justify an award for actual damages, there must be competent proof of Instance of Manila, Branch I, and POTENCIANO GAPOL,
the actual amount of loss. Credence can be given only to claims, which are respondents.
duly supported by receipts.[19] The burden of proof is on the party who will Marcelino Lontok for petitioners.
be defeated if no evidence is presented on either side. He must establish Zavalla, Bautista and Nuevas for respondents.
his case by a preponderance of evidence which means that the evidence, as PADILLA, J.:
a whole, adduced by one side is superior to that of the other. In other This is a petition for a writ of certiorari to annul an order of the respondent
words, damages cannot be presumed and courts, in making an award, must court granting Potenciano Gapol authority, pursuant to section 26, Act No.
point out specific facts that can afford a basis for measuring whatever 1459, otherwise known as the Corporation Law, to call a meeting of the
compensatory or actual damages are borne. stockholders of the Dagunoy Enterprises, Inc. and to preside at such
meeting by giving proper notice to the stockholders, as required by law or
Under Article 2199 of the Civil Code, actual or compensatory damages are by laws of the corporation, until after the majority of the stockholders
those awarded in satisfaction of, or in recompense for, loss or injury present and qualified to vote shall have chosen one of them to act as
sustained. They proceed from a sense of natural justice and are designed to presiding officer of the meeting; another order denying a motion of the
repair the wrong that has been done, to compensate for the injury inflicted petitioners to have the previous order set aside; and a third order denying a
and not to impose a penalty. motion to the same effect as the one previously filed.
The petitioners aver that the Daguhoy Enterprises, Inc., was duly
In the instant case, the actual damages were proven through the sole registered as such on 24 June 1948; that on 16 April 1951 at a meeting
testimony of Themistocles Ruguero, the vice president for administration duly called, the voluntary dissolution of the corporation and the
of Panacor. In his testimony, the witness affirmed that Panacor incurred appointment of Potenciano Gapol as receiver were agreed upon and to that
losses, specifically, in terms of training and seminars, leasehold end a petitioner Domingo Ponce; that instead of filing the petition for
acquisition, procurement of vehicles and office equipment without, voluntary dissolution of the of the corporation as agreed upon, the
however, adducing receipts to substantiate the same. The documentary respondent Potenciano Gapol, who is the largest stockholder, charged his
evidence marked as exhibit "W", which was an ordinary private writing mind and filed a complaint in the Court of First Instance of Manila (civil
allegedly itemizing the capital expenditures and losses from the failed No. 13753) to compel the petitioners to render an accounting of the funds
operation of Panacor, was not testified to by any witness to ascertain the and assets of the corporation, to reimburse it, jointly and severally, in the
veracity of its contents. Although the lower court fixed the sum of sum of P4,500, the purchase price of a parcel of land acquired by the
P4,520,000.00 as the total expenditures incurred by Panacor, it failed to corporation; P6,190 loaned to the wife of petitioner Domingo Ponce; and
show how and in what manner the same were substantiated by the P8,000 spent by the latter in his trip to the United States, or a total sum of
claimant with reasonable certainty. Hence, the claim for actual damages P18,690, plus interest, or such sum as may be found after the accounting
should be admitted with extreme caution since it is only based on bare shall have been rendered to have been misspent, misapplied,
assertion without support from independent evidence. Premiere's failure to missappropriated and converted by the petitioner Domingo Ponce to his
prove actual expenditure consequently conduces to a failure of its claim. own use and benefit; that on 18 May 1951 the plaintiff in that case, the
respondent Potenciano Gapol in this case, filed a motion praying that the meeting called for that purpose every even year. They had no right to a
petitioners be removed as members of the board of directors which was hold-over brought about by the failure to perform the duty incumbent
denied by the court; that on 3 January 1952 respondent Potenciano Gapol upon one of them. If they felt that they were sure to be reelected, why did
filed a petition (civil No. 15445, Exhibit L), praying for an order directing they fail, neglect, or refuse to call the meeting to elect the members of the
him to a call a meeting of the stockholders of the corporation and to board? Or, why did they not seek their reelection at the meeting called to
preside at such meeting in accordance with section 26 of the Corporation elect the directors pursuant to the order of the respondent court.
law; that two days later, without notice to the petitioners and to the other The alleged illegality of the election of one member of the board of
members of the board of directors and in violation of the Rules of Court directors at the meeting called by the respondent Potenciano Gapol as
which require that the adverse parties be notified of the hearing of the authorized by the court being subsequent to the order complained of
motion three days in advance, the respondent court issued the order as cannot affect the validity and legality of the order. If it be true that one of
prayed for (Exhibit M); that the petitioners learned only of this order of the directors elected at the meting called by the respondent Potenciano
the court on 27 February, when the Bank of America refused to recognize Gapol, as authorized by the order of the court complained of, was not
the new board of directors elected at such meeting and returned the checks qualified in accordance with the provisions of the by-laws, the remedy of
drawn upon it by the said board of directors; that the election of Juanito R. an aggrieved party would be quo a warranto. Also, the alleged previous
Tianzon as member of the board of directors of the corporation he must be agreement to dissolve the corporation does not affect or render illegal the
a member of the Legionarios del Trabajo, as required and provided for in order issued by the respondent court.
article 7 of the by-laws of the corporation; that on 5 March the petitioners The petition is denied, with costs against the petitioners.
filed a petition in the respondent court to have the order of 5 January set
aside but on April, the date set for the hearing of the petition, as the G.R. No. L-5377 December 29, 1954
respondent judge was on leave vacation judge directed its transfer to the MARIA CLARA PIROVANA ET AL., plaintiffs-appellees,
branch of the respondent judge; that without having set the motion for vs.
hearing, the respondent court denied the motion of 5 March in its order of THE DE LA RAMA STEAMSHIP CO., defendant-appellant.
7 May; that on 14 May the petitioners filed another motion inviting the Facts:
attention of the respondent court to the irregularity and illegality of its
procedure and setting the motion for hearing on 21 May, but the court Plaintiffs herein are the minor children of the late Enrico Pirovano
denied the motion by its order of 13 June. represented by their mother and judicial guardian Estefania R. Pirovano.
The only question to determine in this case is whether under and pursuant They seek to enforce certain resolutions adopted by the Board of Directors
to section 26 of Act No. 1459, known as the Corporation law, the and stockholders of the defendant company giving to said minor children
respondent court may issue the order complained of. Said section of the proceeds of the insurance policies taken on the life of their deceased
provides: — father Enrico Pirovano with the company as beneficiary. Defendant is a
Whenever, from any cause, there is no person authorized to call a meeting, corporation duly organized in accordance with law with an authorized
or when the officer authorized to do so refuses, fails or neglects to call a capital of P500,000, divided into 5,000 shares, with a par value of P100
meeting, any judge of a Court of First Instance on the showing of good each share. Enrico Pirovano became the president of the defendant
cause therefor, may issue an order to any stockholder or member of a company and under his management the company grew and progressed
corporation, directing him to call a meeting of the corporation by giving until it became a multi-million corporation by the time Pirovano was
the proper notice required by this Act or by-laws; and if there be no person executed by the Japanese during the occupation.
legally authorized to preside at such meeting, the judge of the Court of
First Instance may direct the person calling the meeting to preside at the In the meantime, Don Esteban de la Rama, who practically owned and
same until a majority of the members or stockholders representing a controlled the stock of the defendant corporation, distributed his
majority of the stock members or stockholders presenting a majority of the shareholding among his five daughters. One of the daughters was married
stock present and permitted by law to be voted have chosen one of their to Enrico Pirovano. Meanwhile, a grant was made in favour of the
number to act as presiding officer for the purposes of the meeting. Pirovano children which constitutes the proceeds of the insurance policies
On the showing of good cause therefor, the court may authorize a taken on his life by the defendant company. Out of the proceeds of these
stockholder to call a meeting and to preside threat until the majority policies the sum of P400,000 be set aside for the minor children of the
stockholders representing a majority strockholders representing a majority deceased, said sum of money to be convertible into 4,000 shares of the
of the stock present and permitted to be voted shall have chosen one stock of the Company, at par, or 1,000 shares for each child. However,
among them to preside it. And this showing of good cause therefor exists members of the family and Don Esteban did not realize that they would be
when the court is apprised of the fact that the by-laws of the corporation actually giving to the Pirovano children more than what they intended to
require the calling of a general meeting of the stockholders to elect the give. If the Pirovano children would be given shares of stock in lieu of the
board of directors but call for such meeting has not been done. amount to be donated, the voting strength of the five daughters of Don
Article 9 of the by-laws of the Daguhoy Enterprises, Inc., provides: Esteban in the company would be adversely affected in the sense that Mrs.
The Board of Directors shall compose of five (5) members who shall be Pirovano would have a voting power twice as much as that of her sisters.
elected by the stockholders in a general meeting called for that purpose
which shall be held every even year during the month of January. The Board of Directors of the De la Rama company, as a consequence of
Article 20 of the by-laws in part provides: the change of attitude of Don Esteban, adopted a resolution changing the
. . . Regular general meetings are those which shall be called for every form of the donation to the Pirovano children from a donation of 4,000
even year, . . . . shares of stock as originally planned into a renunciation in favor of the
The requirement that "on the showing of good cause therefor," the court children of all the company's "right, title, and interest as beneficiary in and
may grant to a stockholder the authority to call such meeting and to to the proceeds of the abovementioned life insurance policies", subject to
preside thereat does not mean that the petition must be set for hearing with the express condition that said proceeds should be retained by the
notice served upon the board of directors. The respondent court was company as a loan.
satisfied that there was a showing of good cause for authorizing the
respondent Potenciano Gapol to call a meeting of the stockholders for the On March 8, 1951, at a stockholders' meeting convened and majority of
purpose of electing the board of directors as required and provided for in the stockholders' voted to revoke the resolution approving the donation to
the by-laws, because the chairman of the board of directors called upon to the Pirovano children.
do so had failed, neglected, or refused to perform his duty. It may be
likened to a writ of preliminary injunction or of attachment which may be Issues:
issued ex-parte upon compliance with the requirements of the rules and
upon the court being satisfied that the same should be issue. Such 1. Whether or the donation has been perfected such that the corporation
provisional reliefs have not been deemed and held as violative of the due can no longer rescind it even if it wanted to.
process of law clause of the Constitution. 2. Can defendant corporation give by way of donation the proceeds of said
In several state of the Union1 the remedy which may be availed of our insurance policies to the minor children of the late Enrico Pirovano under
resorted to in a situation such as the one brought about in this case is the law or its articles of corporation, or is that donation an ultra vires act?
mandamus to compel the officer or incumbent board of directors to
perform a duties specifically enjoined by law or by-laws, to wit: to call a Ruling:
meeting of the stockholders. Dela ware is the estate that has a law similar
to ours and there the chancellor of a chancery court may summarily issue 1. Yes. The donation was a corporate act carried out by the corporation not
or enter an order authorizing a stockholder to call a meeting of the only with the sanction of its Board of Directors but also of its
stockholders of the corporation and preside thereat.2 It means that the stockholders. It is evident that the donation has reached the stage of
chancellor may issue such order without notice and hearing. perfection which is valid and binding upon the corporation and as such
That the relief granted by the respondent court lies within its jurisdiction is cannot be rescinded unless there is exists legal grounds for doing so. In
not disputed. Having the authority to grant the relief, the respondent court this case, we see none. The two reasons given for the rescission of said
did not exceed its jurisdiction; nor did it abuse its discretion in granting it. donation in the resolution of the corporation adopted on March 8, 1951, to
With persistency petitioners claim that they have been deprived of their wit: that the corporation failed to comply with the conditions and that in
right without due process of law. They had no right to continue as the opinion of the Securities and Exchange Commission said donation is
directors of the corporation unless reflected by the stockholders in a ultra vires, are not, in our opinion, valid and legal as to justify the
rescission of a perfected donation. These reasons cannot be invoked by the properties, or in other words, to operate as a pawn broker, engage in
corporation to rescind or set at naught the donation, and the only way by pawnshop business as defined under P.D 114.
which this can be done is to show that the donee has been in default, or
that the donation has not been validly executed, or is illegal or ultra vires, Private respondent Filipinas Pawnshop, Inc. filed a complaint against
and such is not the case as we will see hereafter. We therefore declare that petitioner, Pilipinas Loan Company with the Prosecution Enforcement
the resolution approved by the stockholders of the defendant corporation Department of the SEC alleging that the petitioner contrary to its prime
on March 8, 1951 did not and cannot have the effect of nullifying the purpose, has been operating and doing business as a pawn broker,
donation in question. pawnshop or “sanglaan” in the same area where private respondent has
had its own pawnshop for 30 years, without permission from the Central
2. We find that the corporation was given broad and almost unlimited Bank to engage in pawnshop business, thereby causing unjust and unfair
powers to carry out the purposes for which it was organized among them, competition with private respondent.
(1) "To invest and deal with the moneys of the company not immediately
required, in such manner as from time to time may be determined" and, (2) Petitioner denied that it is engage in the pawnshop business alleging that it
"to aid in any other manner any person, association, or corporation of is a lending investor duly registered with the Central Bank. Petitioner also
which any obligation or in which any interest is held by this corporation or questioned the power of the SEC to take cognizance of the complaint
in the affairs or prosperity of which this corporation has a lawful interest." involving supposed violation of the Pawnshop Regulation Act (P.D.114)
which is more properly within the jurisdiction of the Central Bank.
The world deal is broad enough to include any manner of disposition, and
refers to moneys not immediately required by the corporation, and such The SEC en banc rendered a decision ordering the petitioner to cease and
disposition may be made in such manner as from time to time may be desist from further engaging in business as a “pawnshop” or “pawnbroker”
determined by the corporations. Under the second broad power, that is, to as defined in P.D.114, until proper license shall have been secured from
aid in any other manner any person in the affairs and prosperity of whom the Central Bank.
the corporation has a lawful interest, the record of this case is replete with On appeal, the Court of Appeals affirmed the decision of the SEC en banc
instances which clearly show that the corporation knew well its scope and hence, this petition for certiorari under Rule 45 of the Rules of Court.
meaning so much so that, with the exception of the instant case, no one
has lifted a finger to dispute their validity. ISSUE
1) Whether or not petitioner had violated its Articles of Incorporation by
It may perhaps be argued that the donation given to the children of the late engaging in pawnshop business.
Enrico Pirovano is so large and disproportionate that it can hardly be 2) Whether or not the SEC has jurisdiction over complaints involving
considered a pension of gratuity that can be placed on a par with the violation of Pawnshop Regulation Act (P.D.114.) 285
instances above mentioned, but this argument overlooks one
consideration: the gratuity here given was not merely motivated by pure RULING
liberality or act of generosity, but by a deep sense of recognition of the On the first issue, a corporation has only such power as are expressly
valuable services rendered by the late Enrico Pirovano which had granted to it by the Code and by its Articles of Incorporation, those that
immensely contributed to the growth of the corporation to the extent that are incidental to such conferred powers and to its existence. In the case at
from its humble capitalization it blossomed into a multi-million bar, the limit of the powers of petitioner as a corporation is that “it is
corporation that it is today. Said donation was given not only because the prohibited from engaging in pawn broking as defined in P.D.114.”
company was so indebted to him that it saw fit and proper to make Petitioner engaged in the pawnshop business when it is not authorized to
provisions for his children, but it did so out of a sense of gratitude. do so by its Articles of Incorporation is an ultra vires act, which amounts
Another is that Enrico Pirovano was not only a high official of the to fraud, detrimental not only to the corporation but also to the public.
company but was at the same time a member of the De la Rama family,
and the recipients of the donation are the grandchildren of Don Esteban de On the issue of the jurisdiction, the SC ruled that the SEC has jurisdiction
la Rama. This we, may say, is the motivating root cause behind the grant to entertain complaints involving ultra vires acts of a corporation. Thus,
of this bounty. the complaint treats of a violation of petitioner’s primary purpose, an ultra
vires act. Moreover, by law the SEC has absolute jurisdiction, supervision
We do not see much difference between this definition of gratuity and a and control over all corporations that are enfranchised to act as corporate
remunerative donation contemplated in the Civil Code. In essence they are entities and registered under it. Furthermore, petitioner cannot invoke the
the same. A distinction should be made between corporate acts or jurisdiction of the Central Bank in view of its own avowal that it is not a
contracts which are illegal and those which are merely ultra vires. The pawnshop and neither is it engaged in the business as a pawnshop. P.D.
former contemplates the doing of an act which is contrary to law, morals, 114 provides that the supervisory power of the Central Bank extends
or public policy or public duty, and are, like similar transactions between merely to pawnshop registered with it hence, the Central Bank does not
the individuals void. On the other hand, an ultra vires act is one outside acquire jurisdiction over the petitioner.
the scope of the power conferred by the legislature, and although the term
has been used indiscriminately, it is properly distinguishable from acts People vs. Dumlao
which are illegal, in excess or abuse of power, or executed in an G.R. No. 168918
unauthorized manner, or acts within corporate powers but outside the March 2, 2009
authority of particular officers or agents.
Doctrine
Since it is not contended that the donation under consideration is illegal The fundamental test in determining the sufficiency of the material
we cannot but logically conclude, that said donation, even if ultra vires in averments of an information is whether the facts alleged therein, which are
the supposition we have adverted to, is not void, and if voidable its hypothetically admitted, would establish the essentials elements of the
infirmity has been cured by ratification and subsequent acts of the crime defined by law. Evidence aliunde, or matters extrinsic of the
defendant corporation. The defendant corporation, therefore, is now Information, are not be considered.
prevented or estopped from contesting the validity of the donation. To
allow the corporation to undo what it has done would only be most unfair Facts
but would contravene the well-settled doctrine that the defense of ultra An information was filed before the Sandiganbayan charging respondents
vires cannot be set up or availed of in completed transactions. Dumlao, La’o and others with violation of the Section 3(g) of R.A. no.
3019 or Anti-Graft and Corrupt Practices Act. The information alleged
PILIPINAS LOAN COMPANY, INC. that the respondent-members of the Board of Trustees of GSIS entered
VS into a contract of lease-purchase with respondent La’o, a private person
SECURITIES AND EXCHANCE COMMISSION whereby GSIS agreed to sell to La’o, a GSIS-acquired property consisting
GR.NO.104720, APRIL 4, 2001 of a land and building known as the Government Counsel Centre for P2
Million on an instalment basis with annual interest and amortization and
FACTS OF THE CASE grant La’o the right to sub-lease the ground floor during the period of
lease, from which he collected yearly rentals in excess of the yearly
Petitioner Pilipinas Loan Company Inc. is a lending Corporation registered amortization causing gross disadvantage to the government.
with the SEC, and its primary purpose base on its Articles of Incorporation
is “To act as a lending investor or to engage in the practice of lending During arraignment, Dumlao pleaded not guilty, and as agreed by
money or extending loans on the security of real or personal, tangible or prosecution and respondents, a Joint Stipulation of Facts and Admission of
intangible properties whether as pledge, real or chattel mortgage, xxx Exhibits was submitted to the court. The Joint Stipulation admitted
without however engaging in pawn broking as defined under P.D.114”. additional facts: (1) 3 members of the Board, Dumlao being one of them,
signed the Minutes; (2) 7 members of the Board were present during the
Private respondent Filipinas Pawnshop, Inc. is also a corporation duly board meeting; and (3) the documentary evidence of was authentic and
registered with the SEC, and its primary purpose under its Articles of duly executed. It was further decided for the pre-trial to be terminated
Incorporation, is to extend loans on the security of either personal or real limiting the course of the subsequent trial to “matters not disposed of...
unless modified by the court.”
that the other 10% is owned by another foreign corporation; that a mining
Dumlao filed a Motion to Dismiss/Quash on the ground that the facts corporation cannot be interested in another mining corporation. SJP on the
charged do not constitute an offense. He stated that the prosecution’s main other hand invoked that under the parity rights agreement (Laurel-Langley
thrust against him was the alleged approval by the GSIS Board of the Agreement), SJP, a foreign corporation, is allowed to invest in a domestic
Lease-Purchase Agreement. He argued that the Resolution was not in fact corporation.
approved by the GSIS Board. Since the signatures of fellow respondents ISSUE: Whether or not SJP is correct.
did not appear in the minutes of the meeting, these people did not HELD: No. The parity rights agreement is not applicable to SJP. The
participate in the Lease-Purchase Agreement. There was no quorum of the parity rights are only granted to American business enterprises or
board; thus no resolution approving the Agreement. Since the resolution enterprises directly or indirectly controlled by US citizens. SJP is a
was not approved, he was innocent. He added that the person liable was Panamanian corporate citizen. The other owners of SJO are Venezuelan
Atty. Javellana who actually executed the contract. corporations, not Americans. SJP was not able to show contrary evidence.
Further, the Supreme Court emphasized that the stocks of these
Sandiganbayan ruled in favor of Dumlao. It found that the minutes shows corporations are being traded in stocks exchanges abroad which renders
that the Board failed to approve the Lease-Purchase Agreement in their foreign ownership subject to change from time to time. This fact
question. As evidenced by the Joint Stipulation, of the 7 members, only 3 renders a practical impossibility to meet the requirements under the parity
signed. It did not validly pass a resolution because at least a majority of 4 rights. Hence, the tie up between SJP and SJO is illegal, SJP not being a
votes were required. Therefore prosecution had no cause of action against domestic corporation or an American business enterprise contemplated
Dumlao. under the Laurel-Langley Agreement.

Hence, this petition for certiorari under Rule 45, Rules of Court. Ong Yong V. Tiu (2003)
G.R. No. 144476 April 8, 2003
Issue
1. WON court erred in dismissing case after pre-trial and before Lessons Applicable: Pre-incorporation Subscription (Corporate Law)
prosecution could formally present its evidence
a. WON facts charged in the information actually constitute an FACTS:
offense 1994: construction of the Masagana Citimall in Pasay City was
b. WON insufficiency of evidence is a ground for Motion to threatened with stoppage, when its owner, the First Landlink Asia
Dismiss Development Corporation (FLADC), owned by the Tius, became heavily
indebted to the Philippine National Bank (PNB) for P190M
Held To save the 2 lots where the mall was being built from
1. Yes. The petition is granted. Resolution of Sandiganbayan is foreclosure, the Tius invited Ong Yong, Juanita Tan Ong, Wilson T. Ong,
reversed and set aside. Sandiganbayan is ordered for reception of evidence Anna L. Ong, William T. Ong and Julia Ong Alonzo (the Ongs), to invest
of prosecution. in FLADC.
a. Yes. Facts in the information contained elements of the crime Pre-Subscription Agreement: Ongs and the Tius agreed to
charged. maintain equal shareholdings in FLADC
b. No. Grounds for Motion to Dismiss/Quash are limited to those Ongs: subscribe to 1,000,000 shares
enumerated in Sec. 3, Rule 117 of the RRCP Tius: subscribe to an additional 549,800 shares in addition to
their already existing subscription of 450,200 shares
Ratio Tius: nominate the Vice-President and the Treasurer plus 5
directors
The elements of the crime under Section 3(g) of Republic Act No. 3019 Ongs nominate the President, the Secretary and 6 directors
are as follows: (1) that the accused is a public officer; (2) that he entered (including the chairman) to the board of directors of FLADC and right to
into a contract or transaction on behalf of the government; and (3) that manage and operate the mall.
such contract or transaction is grossly and manifestly disadvantageous to Tius: contribute to FLADC a 4-storey building P20M (for 200K
the government. shares)and 2 parcels of land P30M (for 300K shares) and P49.8M (for
49,800 shares)
The ground raised by Dumlao in his Motion to Quash/Dismiss is that the Ongs: paid P190M to settle the mortgage indebtedness of
facts charged do not constitute an offense. And after examining the FLADC to PNB (P100M in cash for their subscription to 1M shares)
information, we find that the facts alleged therein, if hypothetically February 23, 1996: Tius rescinded the Pre-Subscription
admitted, will prove all the elements of Section 3(g) as against respondent Agreement
Dumlao. Therefore, the motion to quash should not have been granted. February 27, 1996: Tius filed at the Securities and Exchange
Commission (SEC) seeking confirmation of their rescission of the Pre-
It can also be gathered from the resolution of the Sandiganbayan that it did Subscription Agreement
not consider the ground invoked by Dumlao (that the facts charged do not SEC: confirmed recission of Tius
constitute an offense); otherwise, it could have denied respondent Ongs filed reconsideration that their P70M was not a premium
Dumlao’s motion. From the reasoning given by the Sandiganbayan, it is on capital stock but an advance loan
clear that it dismissed the case because of insufficiency of evidence. SEC en banc: affirmed it was a premium on capital stock
CA: Ongs and the Tius were in pari delicto (which would not
According to Sec. 3, Rule 117 of RRCP, insufficiency of evidence is not have legally entitled them to rescission) but, "for practical considerations,"
one of the grounds of a Motion to Quash. It is only a ground for dismissal that is, their inability to work together, it was best to separate the two
of an action only after the prosecution rests its case as provided in Sec. groups by rescinding the Pre-Subscription Agreement, returning the
23, Rule 119 of RRCP on demurrer to evidence. original investment of the Ongs and awarding practically everything else
to the Tius.
In the case at bar, Sandiganbayan dismissed the case against respondent ISSUE: W/N Specific performance and NOT recission is the remedy
for insufficiency of evidence, even without giving the prosecution the
opportunity to present its evidence. In so doing, it violated the HELD: YES. Ongs granted.
prosecution’s right to due process. It deprived the prosecution of its did not justify the rescission of the contract
opportunity to prosecute its case and to prove the accused’s culpability. providing appropriate offices for David S. Tiu and Cely Y. Tiu
as Vice-President and Treasurer, respectively, had no bearing on their
It was therefore erroneous for the Sandiganbayan to dismiss the case under obligations under the Pre-Subscription Agreement since the obligation
the premises. Not only did it not consider the ground invoked by pertained to FLADC itself
respondent Dumlao; it even dismissed the case on a ground not raised by failure of the Ongs to credit shares of stock in favor of the Tius
him, and not at the appropriate time. The dismissal was thus without basis for their property contributions also pertained to the corporation and not to
and untimely. the Ongs
the principal objective of both parties in entering into the Pre-
Pedro Palting vs San Jose Petroleum, Inc. Subscription Agreement in 1994 was to raise the P190 million
law requires that the breach of contract should be so "substantial
18 SCRA 924 – Business Organization – Corporation Law – Parity Rights or fundamental" as to defeat the primary objective of the parties in making
– Nationality – Nationalized Areas of Activity the agreement
In 1956, San Jose Petroleum, Inc. (SJP), a mining corporation organized since the cash and other contributions now sought to be returned
under the laws of Panama, was allowed by the Securities and Exchange already belong to FLADC, an innocent third party, said remedy may no
Commission (SEC) to sell its shares of stocks in the Philippines. longer be availed of under the law.
Apparently, the proceeds of such sale shall be invested in San Jose Oil Any contract for the acquisition of unissued stock in an existing
Company, Inc. (SJO), a domestic mining corporation. Pedro Palting corporation or a corporation still to be formed shall be deemed a
opposed the authorization granted to SJP because said tie up between SJP subscription within the meaning of this Title, notwithstanding the fact that
and SJO is violative of the constitution; that SJO is 90% owned by SJP; the parties refer to it as a purchase or some other contract
allows the distribution of corporate capital only in three CA, CA modified the NTC decision changing the basis of the computation
instances: (1) amendment of the Articles of Incorporation to reduce the of supervision and regulation fees under sec. 40(f) of the PSA.
authorized capital stock,24 (2) purchase of redeemable shares by the
corporation, regardless of the existence of unrestricted retained ISSUE:
earnings,25 and (3) dissolution and eventual liquidation of the corporation.
They want this Court to make a corporate decision for FLADC. WHETHER THE COURT OF APPEALS ERRED IN HOLDING THAT
The Ongs' shortcomings were far from serious and certainly less THE COMPUTATION OF SUPERVISION AND REGULATION FEES
than substantial; they were in fact remediable and correctable under the UNDER SECTION 40 (F) OF THE PUBLIC SERVICE ACT SHOULD
law. It would be totally against all rules of justice, fairness and equity to BE BASED ON THE PAR VALUE OF THE SUBSCRIBED CAPITAL
deprive the Ongs of their interests on petty and tenuous grounds. STOCK.

Nyco Sales Corporation vs BA Finance Corporation RULING:


200 SCRA 637 – Mercantile Law – Negotiable Instruments Law – Notice
of Dishonor – Assignment of Credit The law in point is clear and categorical. The basis for computation of the
Nyco Sales Corporation has discounting privileges with BA Finance fee to be charged by NTC on PLDT is the capital stock subscribed or paid
Corporation. In 1978, brothers Renato Fernandez and Santiago Renato and not the property and equipment.
(officers of Sanshell Corporation) approached Nyco Sales Corporation for
a credit accommodation in order for the brothers make use of Nyco’s It bears stressing that it is not the NTC that imposed such a fee. It is the
discounting privileges. Nyco Sales agreed and so, on November 15, 1978, legislature itself. Since Congress has the power to exercise the State
Sanshell issued a post-dated (November 17, 1978) BPI check to Nyco inherent powers of Police Power, Eminent Domain and Taxation, the
Sales in the amount of P60,000.00. Following the discounting process distinction between police power and the power to tax, which could be
agreed upon, Nyco Sales, thru its president Rufino Yao, endorsed the significant if the exercising authority were mere political subdivisions,
check in favor of BA Finance. Thereafter, BA Finance issued a check would not be of any moment when, as in the case under consideration,
payable to Nyco Sales which endorsed it in favor of Sanshell. Sanshell Congress itself exercises the power. All that is to be done would be to
then made use of and/or negotiated the check. Accompanying the apply and enforce the law when sufficiently definitive and not
exchange of checks was a Deed of Assignment executed by Nyco Sales constitutional infirm.
(assignor) in favor of BA Finance (assignee) with the conformity of
Sanshell. Under the said Deed, the subject of the discounting was P60k National Power Corp. vs. Vera
BPI check. G.R. No. 83558; February 27, 1989
The check bounced. BA Finance notified Sanshell. Sanshell substituted
the BPI check with a Security Bank and Trust Company check for P60k. FACTS:
This check again bounced. BA Finance made repeated demands to Nyco Sea Lion International Port Services, private respondent, filed a
Sales and Sanshell but neither of the two settled the obligation. Hence, BA complaint for prohibition and mandamus against petitioner NPC alleging
Finance sued Nyco Sales. Nyco Sales averred that it received no notice of that it had acted in bad faith in not renewing its contract for stevedoring
dishonor when the second check was dishonored. services for its plant and in taking over its stevedoring services.
ISSUE: Whether or not Nyco Sales is liable to pay BA Finance. Respondent judge issued a restraining order against NPC enjoining the
HELD: Yes. The relationship between Nyco Sales and BA Finance is one latter from undertaking stevedoring services at its pier. Consequently,
of assignor-assignee. The assignor-vendor warrants both the credit itself NPC filed an "Urgent Motion" to dissolve the restraining order, asserting
(its existence and legality) and the person of the debtor (his solvency), if that respondent judge had no jurisdiction to issue the order and private
so stipulated, as in the case at bar. Consequently, if there be any breach of respondent, whose contract with NPC had expired prior to the
the above warranties, the assignor-vendor should be held answerable commencement of the suit, failed to establish a cause of action for a writ
therefor. There is no question then that the assignor-vendor is indeed liable of preliminary injunction. The respondent judge denied the NPC’s motion
for the invalidity of whatever he assigned to the assignee-vendee. and issued a TRO after finding that NPC was not empowered by its
Considering now the facts of the case at bar, it is beyond dispute that Nyco Charter to engage in stevedoring and arrastre services.
executed a deed of assignment in favor of BA Finance with Sanshell
Corporation as the debtor-obligor. BA Finance is actually enforcing said ISSUE:
deed and the check covered thereby is merely an incidental or collateral WON the undertaking of stevedoring services is empowered by
matter. This particular check merely evidenced the credit which was the NPC’s charter powers.
actually assigned to BA Finance. Thus, the designation is immaterial as it
could be any other check. It is only what is represented by the said checks HELD:
that Nyco is being asked to pay. YES. To carry out the national policy of total electrification of
Nyco Sales’ pretension that it had not been notified of the fact of dishonor the country, the NPC was created and empowered not only to construct,
is belied not only by the formal demand letter issued by BA Finance but operate and maintain power plants, reservoirs, transmission lines, and
also by the fact that Nyco Sales and Sanshell had frequent contacts before, other works, but also to exercise such powers and do such things as may
during and after the dishonor. More importantly, as long as the credit be reasonably necessary to carry out the business and purposes for which
remains outstanding, Nyco Sales shall continue to be liable to BA Finance it was organized, or which, from time to time, may be declared by the
as its assignor. The dishonor of an assigned check simply stresses its Board to be necessary, useful, incidental or auxiliary to accomplish said
liability and the failure to give a notice of dishonor will not discharge it purpose.
from such liability. This is because the cause of action stems from the In determining whether or not an NPC act falls within the
breach of the warranties embodied in the Deed of Assignment, and not purview of the above provision, the Court must decide whether or not a
from the dishonoring of the check alone. logical and necessary relation exists between the act questioned and the
corporate purpose expressed in the NPC charter. For if that act is one
NTC v. CA , 311 SCRA 508 (199) which is lawful in itself and not otherwise prohibited, and is done for the
FACTS: purpose of serving corporate ends, and reasonably contributes to the
promotion of those ends in a substantial and not in a remote and fanciful
NTC served on the PLDT the following assessment notices and demands sense, it may be fairly considered within the corporation's charter powers.
for payment: 1. The amount of P7,495,161.00 as supervision and
regulation fee under sec. 40(e) of the Public Service Act (PSA) of 1988, Nielson & Co. Inc. vs. Lepanto Consolidated Mining Co.
computed at P0.50 per P100.00 of the PLDT’s outstanding capital stock as GR L-21601 December 28, 1968
at Dec. 31, 1987 which then consisted of Serial Preferred Stock amounting By: Karen P. Lustica
to P1,277,934,390.00 and Common Stock of P221,097,785.00 or a total of
P1,499,032,175.00 2. The amount of P9,000,000.00 as permit fee under FACTS: Before World War II, an operating agreement was executed
Sec. 40(f) of the PSA for the approval of the PLDT increase of its between Nielson & Co. Inc. and the Lepanto Consolidated Mining Co.
authorized capital stock from P2.7B to P4.5B; and 3. The amounts of whereby the former operated and managed the mining properties owned
P12,261,600.00 and P33,472,030.00 as permit fees under sec. 40(g) of the by the latter for a management fee of P2,500.00 a month and a 10%
PSA in connection with the Commissions decisions, approving the PLDT participation in the net profits resulting from the operation of the mining
equity participation in the Fiber Optic Interpacific Cable systems and X-5 properties, for a period of 5 years.
Service Improvement and Expansion Program. PLDT challenged the
aforesaid assessments that these were being made to 1. Raise revenues and Lepanto modified a pertinent provision of the contract. This time, Nielson
not as mere reimbursements 2. The assessments should only have been on will receive (1) 10% of the dividends declared and paid, when and as paid,
the basis of the par values PLDT’s outstanding capital stock and 3. NTC during the period of the contract and at the end of each year, (2) 10% of
has no authority to compel PLDT of the assessed fees under Sec. 40(f) for any depletion reserve that may be set up, and (3) 10% of any amount
the increase since NTC did not render any supervisory or regulatory expended during the year out of surplus earnings for capital account.
activity and incurred no expenses in relation thereto. NTC denied the
protest of PLDT for lack of merit, MR denied. PLDT appealed with the Both parties agreed to renew the contract for a period of 5 years. But the
operation of the mining properties was disrupted on account of the war.
the stockholder, it tends rather to postpone said realization, in that the fund
After the mining properties were liberated from the Japanese forces, the represented by the new stock has been transferred from surplus to assets
mine operation was under Lepanto’s exclusive management. Lepanto and no longer available for actual distribution. Thus, it is apparent that
declared stock dividends worth one million in 1949 and two million in stock dividends are issued only to stockholders. This is so because only
1950. This was during the period covered by an extension in the stockholders are entitled to dividends. They are the only ones who have a
management contract. However, a disagreement arose between the right to a proportional share in that part of the surplus which is declared as
parties. dividends. % stock dividend really adds nothing to the interest of the
stockholder; the proportional interest of each stockholder remains the
Nielson claims his share in the stock dividends. 0n its motion for same. If a stockholder is deprived of his stock dividends - and this happens
reconsideration, Lepanto contends that the payment to Nielson of stock if the shares of stock forming part of the stock dividends are issued to a
dividends as compensation for its services under the management contract non-stockholder - then the proportion of the stockholder’s interest changes
is a violation of the Corporation Law, and that it was not, and it could not radically. Stock dividends are civil fruits of the original investment, and to
be, the intention of the parties that the services of Nielson should be paid the owners of the shares belong the civil fruits.
in shares of stock taken out of stock dividends declared by Lepanto.
PILIPINAS LOAN COMPANY, INC.
ISSUE: WON Nielson is entitled to his share in the stock dividends. VS
SECURITIES AND EXCHANCE COMMISSION
HELD: NO. GR.NO.104720, APRIL 4, 2001

RATIO: Stock dividends cannot be issued to a person who is not a FACTS OF THE CASE
stockholder in payment of services rendered.
Petitioner Pilipinas Loan Company Inc. is a lending Corporation registered
Section 16 of the Corporation Law, in part, provides as follows: with the SEC, and its primary purpose base on its Articles of Incorporation
is “To act as a lending investor or to engage in the practice of lending
No corporation organized under this Act shall create or issue bills, notes or money or extending loans on the security of real or personal, tangible or
other evidence of debt, for circulation as money, and no corporation shall intangible properties whether as pledge, real or chattel mortgage, xxx
issue stock or bonds except in exchange for actual cash paid to the without however engaging in pawn broking as defined under P.D.114”.
corporation or for: (1) property actually received by it at a fair valuation
equal to the par or issued value of the stock or bonds so issued; and in case Private respondent Filipinas Pawnshop, Inc. is also a corporation duly
of disagreement as to their value, the same shall be presumed to be the registered with the SEC, and its primary purpose under its Articles of
assessed value or the value appearing in invoices or other commercial Incorporation, is to extend loans on the security of either personal or real
documents, as the case may be; and the burden or proof that the real properties, or in other words, to operate as a pawn broker, engage in
present value of the property is greater than the assessed value or value pawnshop business as defined under P.D 114.
appearing in invoices or other commercial documents, as the case may be,
shall be upon the corporation, or for (2) profits earned by it but not Private respondent Filipinas Pawnshop, Inc. filed a complaint against
distributed among its stockholders or members; Provided, however, That petitioner, Pilipinas Loan Company with the Prosecution Enforcement
no stock or bond dividend shall be issued without the approval of Department of the SEC alleging that the petitioner contrary to its prime
stockholders representing not less than two-thirds of all stock then purpose, has been operating and doing business as a pawn broker,
outstanding and entitled to vote at a general meeting of the corporation or pawnshop or “sanglaan” in the same area where private respondent has
at a special meeting duly called for the purpose. had its own pawnshop for 30 years, without permission from the Central
Bank to engage in pawnshop business, thereby causing unjust and unfair
In the case at bar, Nielson cannot be paid in shares of stock which form competition with private respondent.
part of the stock dividends of Lepanto for services it rendered under the
management contract. We sustain the contention of Lepanto that the Petitioner denied that it is engage in the pawnshop business alleging that it
understanding between Lepanto and Nielson was simply to make the cash is a lending investor duly registered with the Central Bank. Petitioner also
value of the stock dividends declared as the basis for determining the questioned the power of the SEC to take cognizance of the complaint
amount of compensation that should be paid to Nielson, in the proportion involving supposed violation of the Pawnshop Regulation Act (P.D.114)
of 10 % of the cash value of the stock dividends declared. In other words, which is more properly within the jurisdiction of the Central Bank.
Nielson must still be paid his 10% fee using as the basis for computation
the cash value of the stock dividends declared. The SEC en banc rendered a decision ordering the petitioner to cease and
desist from further engaging in business as a “pawnshop” or “pawnbroker”
Moreover, from the above-quoted provision of Section 16 of the as defined in P.D.114, until proper license shall have been secured from
Corporation Law, the consideration for which shares of stock may be the Central Bank.
issued are cash, property; and undistributed profits. Shares of stock are On appeal, the Court of Appeals affirmed the decision of the SEC en banc
given the “special name,” “stock dividends,” only if they are issued in lieu hence, this petition for certiorari under Rule 45 of the Rules of Court.
of undistributed profits. If shares of stocks are issued in exchange of cash
or property then those shares do not fall under the category of “stock ISSUE
dividends”. A corporation may legally issue shares of stock in 1) Whether or not petitioner had violated its Articles of Incorporation by
consideration of services rendered to it by a person not a stockholder, or in engaging in pawnshop business.
payment of its indebtedness. But a share of stock issued to pay for 2) Whether or not the SEC has jurisdiction over complaints involving
services rendered is equivalent to a stock issued in exchange of property, violation of Pawnshop Regulation Act (P.D.114.) 285
because services is equivalent to property. Likewise a share of stock
issued in payment of indebtedness is equivalent to issuing a stock in RULING
exchange for cash. But a share of stock thus issued should be part of the On the first issue, a corporation has only such power as are expressly
original capital stock of the corporation upon its organization, or part of granted to it by the Code and by its Articles of Incorporation, those that
the stocks issued when the increase of the capitalization of a corporation is are incidental to such conferred powers and to its existence. In the case at
properly authorized. In other words, it is the shares of stock that are bar, the limit of the powers of petitioner as a corporation is that “it is
originally issued by the corporation and forming part of the capital that prohibited from engaging in pawn broking as defined in P.D.114.”
can be exchanged for cash or services rendered, or property; that is, if the Petitioner engaged in the pawnshop business when it is not authorized to
corporation has original shares of stock unsold or unsubscribed, either do so by its Articles of Incorporation is an ultra vires act, which amounts
coming from the original capitalization or from the increased to fraud, detrimental not only to the corporation but also to the public.
capitalization. Those shares of stock may be issued to a person who is not
a stockholder, or to a person already a stockholder in exchange for On the issue of the jurisdiction, the SC ruled that the SEC has jurisdiction
services rendered or for cash or property. But a share of stock coming to entertain complaints involving ultra vires acts of a corporation. Thus,
from stock dividends declared cannot be issued to one who is not a the complaint treats of a violation of petitioner’s primary purpose, an ultra
stockholder of a corporation. vires act. Moreover, by law the SEC has absolute jurisdiction, supervision
and control over all corporations that are enfranchised to act as corporate
A “stock dividend” is any dividend payable in shares of stock of the entities and registered under it. Furthermore, petitioner cannot invoke the
corporation declaring or authorizing such dividend. jurisdiction of the Central Bank in view of its own avowal that it is not a
pawnshop and neither is it engaged in the business as a pawnshop. P.D.
So, a stock dividend is actually two things. - a dividend and the enforced 114 provides that the supervisory power of the Central Bank extends
use of the dividend money to purchase additional shares of stock at par. merely to pawnshop registered with it hence, the Central Bank does not
When a corporation issues stock dividends, it shows that the corporation acquire jurisdiction over the petitioner.
accumulated profits have been capitalized instead of distributed to the
stockholders or retained as surplus available for distribution, in money or MONFORT HERMANOS AGRICULTURAL DEVELOPMENT
kind, should opportunity offer. Far from being a realization of profits for CORPORATION vs ANTONIO B. MONFORT III
Facts: Monfort Hermanos Agricultural Development Corporation, a other concessions, but extending the operation of the milling contract from
domestic private corporation, is the registered owner of a farm, fishpond the original 30 years to 45 years. The Board of Directors of the appellee
and sugar cane plantation known as Haciendas San Antonio II, Marapara, Bacolod-Murcia Milling Co., Inc., adopted a resolution granting further
Pinanoag and Tinampa-an, all situated in Cadiz City. It also owns one unit concessions to the planters over and above those contained in the printed
of motor vehicle and two units of tractors. The same allowed Ramon H. Amended Milling Contract. The appellants initiated the present action,
Monfort, its Executive Vice President, to breed and maintain fighting contending that three Negros sugar centrals with a total annual production
cocks in his personal capacity at Hacienda San Antonio. In 1997, the exceeding one-third of the production of all the sugar central mills in the
group of Antonio Monfort III, through force and intimidation, allegedly province, had already granted increased participation (of 62.5%)to their
took possession of the 4 Haciendas, the produce thereon and the motor planters, and that under the resolution the appellee had become obligated
vehicle and tractors, as well as the fighting cocks of Ramon H. Monfort. to grant similar concessions to the plaintiffs. The appellee Bacolod-Murcia
In G.R. No. 155472: The Corporation, represented by its President, Ma. Milling Co., inc., resisted the claim, and defended by urging that the
Antonia M. Salvatierra, and Ramon H. Monfort, in his personal capacity, stipulations contained in the resolution were made without consideration;
filed against the group of Antonio Monfort III, a complaint for delivery of that the resolution in question was, therefore, null and void ab initio, being
motor vehicle, tractors and 378 fighting cocks, with prayer for injunction in effect a donation that was ultra vires and beyond the powers of the
and damages. Motion to dismiss on the ground of Ma. Antonia M. corporate directors to adopt.
Salvatierra's lack of capacity to sue on behalf of the Corporation was Issue: WON the board resolution is an ultra vires act and in effect a
denied. donation from the board of directors?
In G.R. No. 152542: Ma. Antonia M. Salvatierra filed on behalf of the Held: No. There can be no doubt that the directors of the appellee
Corporation a complaint for forcible entry, preliminary mandatory company had authority to modify the proposed terms of the Amended
injunction with temporary restraining order and damages against the group Milling Contract for the purpose of making its terms more acceptable to
of Antonio Monfort III. the other contracting parties. As the resolution in question was passed in
The group of Antonio Monfort III alleged that they are possessing and good faith by the board of directors, it is valid and binding, and whether or
controlling the Haciendas and harvesting the produce therein on behalf of not it will cause losses or decrease the profits of the central, the court has
the corporation and not for themselves. They likewise raised the no authority to review them. Whether the business of a corporation should
affirmative defense of lack of legal capacity of Ma. Antonia M. Salvatierra be operated at a loss during depression, or close down at a smaller loss, is
to sue on behalf of the Corporation. a purely business and economic problem to be determined by the directors
Complaint was eventually dismissed. of the corporation and not by the court. The appellee Bacolod-Murcia
Basis of claim of Salvatierra\s lack of capacity to sue: The group of Milling Company is, under the terms of its Resolution of August 20, 1936,
Antonio Monfort III claims that the March 31, 1997 Board Resolution duty bound to grant similar increases to plaintiffs-appellants herein.
authorizing Ma. Antonia M. Salvatierra and/or Ramon H. Monfort to
represent the Corporation is void because the purported Members of the [ G.R. No. L-15092, September 29, 1962 ]
Board who passed the same were not validly elected officers of the
Corporation. ALFREDO MONTELIBANO, ET AL., PLAINTIFFS-
Issue/ Held: WON Ma. Antonia M. Salvatierra has the legal capacity APPELLANTS, VS. BACOLOD-MURCIA MILLING CO., INC.,
to sue on behalf of the Corporation. -NO. Ma. Antonia M. Salvatierra DEFENDANTS-APPELLEE.
failed to prove that four of those who authorized her to represent the
Corporation were the lawfully elected Members of the Board of the DECISION
Corporation. As such, they cannot confer valid authority for her to BARRERA, J.:
sue on behalf of the corporation. SUMMARY
Ratio:
Defendant's answer to the complaint raised defenses based on facts and
law. The trial court in dismissing the case limited itself to the resolution
Corporation failed to comply with Section 26 of the Corporation Code, of defenses based on law and ignored those based on facts. From the order
requiring submission to the SEC within thirty (30) days after the election of dismissal the plaintiffs appealed. Appellee's arguments in its brief were
the names, nationalities and residences of the elected directors, trustees limited to the issued of law and without attempting to call the attention of
and officers of the Corporation. the trial and appellate court to the necessity of considering the factual
1. In the case at bar, the fact that four of the six Members of the defenses submitted in the case for decision. Thereafter this Court rendered
Board listed in the 1996 General Information Sheet are already a decision in favor of appellants. Consequently, appellee filed two
dead at the time the March 31, 1997 Board Resolution was motions for reconsideration of the decision praying that it be set aside and
issued, does not automatically make the four signatories (i.e., an order be issued remanding1 the case either to the Court of Appeals or
Paul M. Monfort, Yvete M. Benedicto, Jaqueline M. Yusay and the Court of First Instance so that appellee's factual defenses be
Ester S. Monfort) to the said Board Resolution (whose name do considered. Appellee further argued that the decision is illegal since it
not appear in the 1996 General Information Sheet) as among the failed to specify the quantity of sugar due the appellants. The Supreme
incumbent Members of the Board. This is because it was not Court denied the motions for reconsideration.
established that they were duly elected to replace the said
deceased Board Members. HELD: During the pendency of the' appeal the appellee had ample time
To correct the alleged error in the General Information Sheet, the retained and options within which it could ventilate its grounds to reconsider either
accountant of the Corporation informed the SEC in its November 11, 1998 in the trial court or appellate court but failed to do so. Its inaction would
letter that the non-inclusion of the lawfully elected directors in the 1996 mean waiver of these factual defenses. The course suggested by appellee
General Information Sheet was attributable to its oversight and not the cannot be countenanced. It is the policy of this Court to discourage
fault of the Corporation. This belated attempt, however, did not erase the piecemeal appeals which are the constant source of delay, in the
doubt as to whether an election was indeed held. administration of justice and clogging of court dockets. Supplemental
What further militates against the purported election of those who signed proceedings in aid of execution which were neither new trial nor rehearing
the March 31, 1997 Board Resolution was the belated submission of the of the case can be had to determine the quantity of sugar due the
alleged Minutes of the October 16, 1996 meeting where the questioned appellants.
officers were elected. The issue of legal capacity of Ma. Antonia M.
Salvatierra was raised before the lower court by the group of Antonio MEDIAN CONTAINER CORPORATION VS. METROPOLITAN
Monfort III as early as 1997, but the Minutes of said October 16, 1996 BANK
meeting was presented by the Corporation only in its September 29,
1999 Comment before the Court of Appeals. Moreover, the Corporation Facts : Respondent filed a complaint for sum of money against petitioner
failed to prove that the same October 16, 1996 Minutes was submitted to for failure to settle the amount of more than P5,000,000 representing the
the SEC. outstanding balance of loans contracted by MCC. Petitioner questions the
certificate of non-forum shopping filed by respodent which was signed by
Montelibano vs Bacolod-Murcia Milling (1962) a certain Atty. Alexander P. Mendoza on May 28, 2003. Petitioner claims
February 14, 2013 markerwins Corporation Law, Mercantile that Atty. Mendoza was only given authority to execute the certificate only
Lawcorpo, merc on June 3, 2003. The trial and appellate court denied petitioner’s motion to
Facts: Plaintiffs-appellants, Alfredo Montelibano, Alejandro Montelibano, dismiss.
and the Limited co-partnership Gonzaga and Company, had been and are
sugar planters adhered to the defendant-appellee’s sugar central mill under Issue/s: W/N respondent failed to comply with the proper procedure on
identical milling contracts. Originally executed in 1919, said contracts the verification and certification of non-forum shopping.
were stipulated to be in force for 30 years starting with the 1920-21 crop,
and provided that the resulting product should be divided in the ratio of Held/Ratio: NO. Verification is a formal, not jurisdictional,
45% for the mill and 55% for the planters. Sometime in 1936, it was requirement. It is simply intended to secure an assurance that the
proposed to execute amended milling contracts, increasing the planters’ allegations in the pleading are true and correct, and that the pleading is
share to 60% of the manufactured sugar and resulting molasses, besides filed in good faith. That explains why a court may order the correction of
the pleading if verification is lacking, or act on the pleading although it is
not verified, if the attending circumstances are such that strict compliance G.R. No. 6217 December 26, 1911
with the rules may be dispensed with in order to serve the ends of justice.
Facts:
As for the required certification against forum shopping, failure to comply 1. On March 15, 1902, the plaintiff and the defendant organized the
therewith is generally not curable by its submission subsequent to the "Philippine Engineering and Construction Company," the incorporators
filing of the petition nor by amendment, and is cause for its dismissal. A being the only stockholders and also the directors of said company, with
certification against forum shopping signed by a person on behalf of a general ordinary powers.
corporation which is unaccompanied by proof that the signatory is 2. After the plaintiff left the Philippine Islands for China, the other
authorized to file the petition is generally likewise cause for dismissal. In directors, the defendants in this case, held a meeting on December 24,
several cases, however, this Court relaxed the application of these 1903, for the purpose of discussing the condition of the company at that
requirements upon appreciation of attendant special circumstances or time anddetermining what course to pursue.
compelling reasons. 3. They did on that date enter into wrecking contract with the naval
authorities with the defendant McCullough.
In the case at bar, simultaneous with the filing of the complaint, 4. When the sale or transfer took place, there were present four
Metrobank submitted both a certification of non-forum shopping and directors, all of whom gave their consent to that sale or transfer. The
proof that Atty. Mendoza who signed it on its behalf was authorized to do plaintiff was then about and his express consent to make this transfer or
so. The proof of authorization of Atty. Mendoza was dated later than the sale was not obtained. He was, before leaving, one of the directors in this
date of his signing of the certification of non-forum shopping, however, corporation, and although he had resigned as manager, he had not resigned
thus giving the impression that he, at the time he affixed his signature, was as a director.
not authorized to do so. The passing on June 3, 2004 of a Board
Resolution of authorization before the actual filing on June 23, 2004 of the Issue:
complaint, however, is deemed a ratification of Atty. Mendoza's prior Did a majority of the stockholders, who were at the same time a majority
execution on May 28, 2004 of the verification and certificate of non-forum of the directors of this corporation, have the power under the law and its
shopping, thus curing any defects thereof. articles of agreement, to sell or transfer to one of its members the assets of
said corporation?
MARIVELES SHIPYARD CORP. VS. CA et al.
G.R. No. 144134 Ruling:
November 11, 2003 He accepted the position of engineer of the Canton and Shanghai Railway
Company, knowing that his duties as such engineer would require his
FACTS: Petitioner engaged the services of Longest Force Investigation whole time and attention and prevent his returning to the Philippine
and Security Agency, Inc. to render security services at its premises. Islands for at least a year or more. The new position which he accepted in
Longest Force then deployed its security guards, herein private China was incompatible with his position as director in the Philippine
respondents, at the petitioner’s shipyard. However, petitioner found the Engineering and Construction Company, a corporation whose sphere of
services rendered by private respondents unsatisfactory and inadequate operations was limited to the Philippine Islands. These facts are sufficient
causing it to terminate its contract with Longest Force and in turn, the to constitute an abandoning or vacating of his position as director in said
latter terminated the employment of said respondents. Consequently, corporation.
private respondents filed a case for illegal dismissal, underpayment of Consequently, the transfer orsale of the corporation's assets to one of its
wages pursuant to the PNPSOSIA-PADPAO rates, non-payment of members was made by the unanimous consent of all the directors in the
overtime pay, premium pay for holiday and rest day, service incentive corporation at that time.
leave pay, 13th month pay and attorney’s fees, against both Longest Force There were only five stockholders in this corporation at any time, four of
and petitioner, before NLRC, who in turn sought the guards’ reinstatement whom were the directors who made thesale, and the other the plaintiff,
with full backwages and without loss of seniority rights. Petitioner who was absent in China when the said sale took place. The sale was,
appealed to the NLRC, contending that it should not be held jointly and therefore, made by the unanimous consent of four-fifths of all the
severally liable with Longest Force for underpayment of wages and stockholders. Under the articles of incorporation, the stockholders and
overtime pays because it had been religiously and promptly paying the directors had general ordinary powers. There is nothing in said articles
bills for the security services sent by Longest Force and that these are in which expressly prohibitsthe sale or transfer of the corporate property to
accordance with the statutory minimum wage. Also, petitioner contends one of the stockholders of said corporation.
that it should not be held liable for overtime pay as private respondents Articles 1700 to 1708 of the Civil Code deal with the manner of dissolving
failed to present proof that overtime work was actually performed. a corporation. There is nothing in thesearticles which expressly or
However, the labor tribunal affirmed in toto the decision of the Labor impliedly prohibits the sale of corporate property to one of its members,
Arbiter. Petitioner moved for reconsideration, but this was denied by the nor a dissolutionof a corporation in this manner. Neither is there anything
NLRC. The petitioner then filed a special civil action for certiorari in articles 151 to 174 of the Code of Commerce which prohibits the
assailing the NLRC judgment for having been rendered with grave abuse dissolution of a corporation by such sale or transfer.
of discretion with the Court of Appeals, which was also outright dismissed Article XIII of the corporation's statutes expressly provides that "in all the
by the respondent court due to a defective certificate of non-forum meetings of the stockholders, a majorityvote of the stockholders present
shopping and non-submission of the required documents to accompany shall be necessary to determine any question discussed."
said petition. The petitioner then moved for reconsideration of the order of The sale or transfer to one of its members was a matter which a majority
dismissal but was also denied by the appellate court, hence, this petition. of the stockholders could very properlyconsider. But it i said that if the
Issue: Whether or not petitioner is jointly and severally liable with acts and resolutions of a majority of the stockholders in a corporation are
Longest Force binding in every case upon the minority, the minority would be completely
Held: Yes, Petitioner’s liability is joint and several with that of Longest wiped out and their rights would be wholly at the mercy of the abuses of
Force, pursuant to Articles 106, 107 and 109 of the Labor Code. When the the majority.
petitioner contracted with Longest Force, petitioner became an indirect Generally speaking, the voice of a majority of the stockholders is the law
employer of private respondents pursuant to Article 107. Following of the corporation, but there are exceptionsto this rule. There must
Article 106, when the agency as contractor failed to pay the guards, the necessarily be a limit upon the power of the majority. Without such a limit
corporation as principal becomes jointly and severally liable for the the will of the majority would be absolute and irresistible and might easily
guards’ wages. This is mandated by the Labor Code to ensure compliance degenerate into an arbitrary tyranny. The reason for these limitations is
with its provisions, including payment of statutory minimum wage. The that in every contract of partnership and a corporation can be something
security agency is held liable by virtue of its status as direct employer, fundamental and unalterable which is beyond the power of the majority of
while the corporation is deemed the indirect employer of the guards for the the stockholders, and which constitutes the rule controlling their actions.
purpose of paying their wages in the event of failure of the agency to pay This rule which must be observed is to be found in the essential compacts
them. This statutory scheme gives the workers the ample protection of such partnership, which gave served as a basis upon which the members
consonant with labor and social justice provisions of the Constitution. have united, and without which it is not probable that they would have
Petitioner cannot evade its liability by claiming that it had religiously paid entered the corporation. Notwithstanding these limitations upon the power
the compensation of guards as stipulated under the contract with the of the majority of the stockholders, their (the majority's) resolutions, when
security agency. Labor standards are enacted by the legislature to alleviate passed in good faith and for a just cause, deserve careful consideration and
the plight of workers whose wages barely meet the spiraling costs of their are generally binding upon the minority.
basic needs. Labor laws are considered written in every contract. Itis well settled, first, that a private corporation, which owes no special
Stipulations in violation thereof are considered null. Similarly, legislated duty tothe public and which has not been given the right of eminent
wage increases are deemed amendments to the contract. Thus, employers domain, has the absolute right and power as against the whole world
cannot hide behind their contracts in order to evade their (or their except the state, to sell and dispose of all of its property; second, that the
contractors’ or subcontractors’) liability for noncompliance with the board of directors, has the power, without reference to the assent or
statutory minimum wage. authority of the stockholders, when the corporation is in failing
circumstances or insolvent or when it can no longer continue the business
CHARLES W. MEAD vs. E. C. McCULLOUGH, ET AL., and THE with profit, and when it is regarded as an imperative necessity; third, that a
PHILIPPINE ENGINEERING AND CONSTRUCTION COMPANY majority of the stockholders or directors, even against the protest of the
minority, have this power where, from any cause, the business is a failure property for P1,574,560.47. Any acceptance by the SAMD of petitioner's
and the best interest of the corporation and all the stockholders require it offer would not bind respondent.

We therefore conclude that the sale or transfer made by the quorum of the Section 23 of the Corporation Code:
board of directors — a majority of the stockholders — is valid and binding
upon the majority-the plaintiff. This conclusion is not in violation of the Corporate powers of all corporations shall be exercised by the board of
articles of incorporation of the Philippine Engineering and Construction directors. Just as a natural person may authorize another to do certain acts
Company. in his behalf, so may the board of directors of a corporation validly
delegate some of its functions to individual officers or agents appointed by
Manila Metal Container Corp. v. PNB, 511 SCRA 444 (2006) it. Thus, contracts or acts of a corporation must be made either by the
G.R. No. 166862 December 20, 2006 board of directors or by a corporate agent duly authorized by the board.
Absent such valid delegation/authorization, the rule is that the declarations
Facts: of an individual director relating to the affairs of the corporation, but not
Petitioner was the owner of 8,015 square meters of parcel of land located in the course of, or connected with the performance of authorized duties of
in Mandaluyong City, Metro Manila. To secure a P900,000.00 loan it had such director, are held not binding on the corporation. A corporation can
obtained from respondent Philippine National Bank, petitioner executed a only execute its powers and transact its business through its:
real estate mortgage over the lot. Respondent PNB later granted petitioner 1. Board of Directors
a new credit accommodation. On August 5, 1982, respondent PNB filed a 2. Officers and agents when authorized by:
petition for extrajudicial foreclosure of the real estate mortgage and sought 3. A board Resolution; or
to have the property sold at public auction. After due notice and 4. Its by-laws
publication, the property was sold at public action where respondent PNB
was declared the winning bidder. Petitioner sent a letter to PNB, It appears that although respondent requested petitioner to conform to its
requesting it to be granted an extension of time to redeem/repurchase the amended counter-offer, petitioner refused and instead requested
property. Some PNB personnel informed that as a matter of policy, the respondent to reconsider its amended counter-offer. Petitioner's request
bank does not accept “partial redemption”. Since petitioner failed to was ultimately rejected and respondent offered to refund its P725,000.00
redeem the property, the Register of Deeds cancelled TCT No. 32098 and deposit.
issued a new title in favor of PNB.
Meanwhile, the Special Asset Management Department (SAMD) had In sum, then, there was no perfected contract of sale between petitioner
prepared a statement of account of petitioner’s obligation. It also and respondent over the subject property. IN LIGHT OF ALL THE
recommended the management of PNB to allow petitioner to repurchase FOREGOING, the petition is DENIED.
the property for P1,574,560.oo. PNB rejected the offer and
recommendation of SAMD. It instead suggested to petitioner to purchase Luneta Motor Company v AD Santos, Inc et al
the property for P2,660,000.00, in its minimum market value. Petitioner July 31, 1962
declared that it had already agreed to SAMD’s offer to purchase
for P1,574,560.47 and deposited a P725,000.00. Petitioner warned PNB Dizon, J.
that it would seek judicial recourse should the latter decline their offer. TOPIC: Powers of Corporations; Implied and Incidental

PNB informed the petitioner that it accepted the petitioner’s offer but for SV: A CPC was originally granted to Concepcion. This CPC was made
1,931,389.53 in cash less the 725k deposited. On page 2 of the letter sent the subject of a chattel mortgage to secure the payment of Concepcion’s
by PNB, the president of petitioner should sign. However, the president loan to Luneta Motor. Subsequently, a second mortgage was executed
did not affix his signature but only confirmed that the letter was accepted. over the same CPC in favor of RFC. The CPC was later sold to Benitez,
Petitioner did not respond to the letter so PNB requested petitioner to then to Rodi Taxicab, and both sales were made with assumption of
submit an amended offer to repurchase. Petitioner rejected PNB’s offer mortgage in favor of RFC, subject to Luneta Motor’s lien. Luneta Motor
and said that PNB is already proscribed from increasing the price since it filed an action for foreclose the mortgage for failure of Concepcion to pay
already accepted the 725k deposit. his account. While this was pending, RFC also instituted foreclosure
proceedings on its 2nd chattel mortgage, and the CPC was sold in favor of
PNB filed for Annulment of Mortgage and Mortgage Foreclosure, Amador Santos, whose application for approval of sale was immediately
Delivery of Title or Specific Performance with Damages. It argued that it granted by PSC. In the action for foreclosure filed by Luneta, the CFI
had acquired ownership over the property after period to redeem had ordered the CPC to be sold, and eventually Luneta bought the CPC.
elapsed. It claimed that no contract of sale was perfected between it and Luneta Motor applied for the sale’s approval with the PSC, but this was
petitioner. While the case was pending, PNB demanded that petitioner opposed by AD Santos, Inc., claiming that under Luneta Motor’s Articles
vacate the property but it refused to do so. Petitioner offered to repurchase of Incorporation, it was not authorized to engage in the taxicab business or
for P3.5M; PNB rejected because the market value of the property was operate as a common carrier. PSC denied Luneta Motor’s application.
P30M. Petitioner offered to repurchase for P4.25M; PNB rejected.
Under the Corporation Law, , a corporation may purchase, hold, etc., and
Trial court decided in favor of PNB. otherwise deal in such real and personal property is the purpose for which
the corporation was formed may permit, and the transaction of its lawful
CA affirmed the decision of RTC business may reasonably and necessarily require. Luneta Motor’s
Corporate purposes basically allow it to operate and otherwise deal in
Thus this petition. automobiles and automobile accessories. Although it may also engage in
the transportation of persons by water, this does not mean that it may
Issue: engage in the business of land transportation, which is an entirely different
line of business. Contrary to Luneta Motor’s contentions, its Articles of
Whether or not MMCC and PNB had entered into a perfected contract for Incorporation is precisely the best evidence that it has no authority at all to
petitioner to repurchase the property from respondent. engage in the business of land transportation and operate a taxicab service.
As such it follows that it may not acquire an CPC to operate a taxicab
Held: service.

NO PERFECTION. There was no meeting of the minds. Contracts are FACTS:


perfected by mere consent which is manifested by the meeting of the offer  A certificate of public convenience (CPC) was granted before the war
and the acceptance upon the thing and the cause which are to constitute to Nicolas Concepcion to operate a taxicab service of 27 units in the
the contract. City of Manila and therefrom to any point in Luzon.
 Concepcion contracted a loan from Luneta Motor and this was
Once perfected, they bind other contracting parties and the obligations evidenced by a promissory note executed by him and guaranteed by
arising therefrom have the form of law between the parties and should be Placido Esteban.
complied with in good faith. The parties are bound not only to the o To secure payment of this note, Concepcion executed a
fulfillment of what has been expressly stipulated but also to the chattel mortgage covering the CPC in favor of Luneta
consequences which, according to their nature, may be in keeping with Motors
good faith, usage and law. A contract of sale is consensual in nature and is  Concepcion contracted a subsequent loan from Rehabilitation
perfected upon mere meeting of the minds. When there is merely an offer Finance Corporation (RFC; now known as DBP), and this was
by one party without acceptance of the other, there is no contract. secured by a second mortgage over the same CPC.
o This 2nd mortgage was approved by the Public Service
When the contract of sale is not perfected, it cannot, as an independent Commission (PSC), subject to the mortgage lien in favor of
source of obligation, serve as a binding juridical relation between the Luneta Motor
parties. There is no evidence that the SAMD was authorized by
respondent's Board of Directors to accept petitioner's offer and sell the
 The CPC was later sold to Francisco Benitez, Jr. who resold it to o If it could not thus engage in the line of business, it follows
Rodi Taxicab Company. that it may not acquire an CPC to operate a taxicab service,
o Both sales were made assumption of the mortgage in favor such as the one in question, because such acquisition would
of RFC, and were also approved provisionally by the be without purpose and would have no necessary
Commission, subject to Luneta Motor’s Lien. connection with petitioner's legitimate business.
 Petitioner filed an action to foreclose the chattel mortgage in view of Appealed decision AFFIRMED.
the failure of Concepcion and his guarantor to pay their overdue Digest by Krys
account.
o While this case was pending, RFC also instituted G.R. No. L-48237 June 30, 1987
foreclosure proceedings on its 2nd chattel mortgage. As of MADRIGAL & COMPANY, INC., petitioner,
the decision in its favor therein rendered, the CPC was sold vs.
at public auction in favor of Amador D. Santos for HON. RONALDO B. ZAMORA, PRESIDENTIAL ASSISTANT
P24,010.00. FOR LEGAL AFFAIRS, THE HON. SECRETARY OF LABOR, and
o Santos immediately applied with the Commission for the MADRIGAL CENTRAL OFFICE EMPLOYEES UNION,
approval of the sale, and the same was approved, subject to respondents.
the mortgage lien in favor of Luneta Motor.
 [CFI] rendered judgment, adjudging Concepcion indebted to Luneta Facts:
Motor in the sum of P15,197.84 with 12% interest thereon from Madrigal & Company, Inc. (MCI) manages the business of another
December 2, 1941 until full payment, plus other assessments. corporation, Rizal Cement Co., Inc. (RCC). In 1973, a labor union in MCI
o CFI ordered the the CPC subject of the chattel mortgage be sought the renewal of the collective bargaining agreement (CBA). The
sold at a public auction. union proposes a P200.00 monthly wage increase and an additional P100
o Said certificate was eventually sold to Luneta Motor, and monthly allowance. MCI refused to negotiate. Later, MCI reduced its
after 6 days, the Sheriff of the City of Manila issued the authorized capital stocks. It then wrote a letter to the Department of Labor
corresponding certificate of sale. averring that it is incurring losses and so it will enforce a retrenchment
o Thereupon, Luneta Motor filed the application for the program. The letter is however unsupported by documents and so the
approval of the sale. Department of Labor ignored it. However, MCI went on to dismiss several
o In the meantime, before he died, Amador Santos sold and employees which led the labor union to sue MCI for unfair labor practices
transferred all his rights and interests in the CPC in favor of and illegal dismissal. The labor arbiter ruled in favor of the labor union.
AD Santos, Inc., who opposed Luneta Motor’s application. The issue reached the Office of the President. The then Presidential
 In the course of hearing, AD Santos Inc filed a motion to dismiss on Assistant For Legal Affairs, Ronaldo Zamora, denied MCI’s appeal.
the grounds that: On appeal, MCI insists that it is incurring losses; that as such, it has to
1. Under Luneta Motor’s Articles of Incorporation, it was reduce its capitalization; that the profits it is earning are cash dividends
not authorized to engage in the taxicab business or from RCC; that under the law, dividends are the absolute property of a
operate as a common carrier; stockholder like MCI and cannot be compelled to share it with creditors
2. The CFI decision did not affect AD Santos Inc or its (like the employees).
predecessor inasmuch as neither of them had been
impleaded into the case; Issue:
3. What was sold to Luneta Motor were only the "rights, Whether or not cash dividends are absolute property of the stockholders
interests and participation" of Nicolas Concepcion in the and cannot be made available for disposition to a corporation’s creditors.
certificate that had been granted to him which were no
longer existing at the time of the sale. Held:
 [PSC] rendered the decision dismissing Luneta Motor’s application No. As found by the labor arbiter, MCI is in fact making significant
for approval of the sale in its favor, sustaining the first ground profits. MCI’s reduction of its capitalization is simply a scheme to avoid
(petitioner is not authorized in its Art. of Inc.) and ruling that as a negotiations with the labor union. It is therefore correct for the arbiter to
result, it could not acquire by purchase the CPC. order MCI to comply with the union’s demands.
o Luneta Motor appeals , claiming that in accordance with
the Corporation Law and its articles of incorporation, it can It is true that cash dividends are the absolute property of the stockholders
acquire by purchase the certificate of public convenience in and cannot be made available for disposition to a corporation’s creditors.
question, maintaining inferentially that, after acquiring said However, this should be viewed in context. This is only true in the case of
certificate, it could make use of it by operating a taxicab corporation distributing dividends to its stockholders. If this is the case (if
business or operate is a common carrier by land. the dividends are still with the corporation, in this case RCC), then
ISSUE: creditors cannot touch such dividends. But if the stockholder already
receives the dividends, then it becomes a profit on the part of the
Under Corporation Law and its Articles of Incorporation, could Luneta stockholder hence its creditors (like the employees) can make some
Motor acquire the CPC by purchase and thereafter hold the certificate and demands out of it. In this case, MCI is a stockholder of RCC. While RCC
operate thereunder as a common carrier by land? still has not distributed the dividends, creditors cannot demand it because
such dividends are owned by stockholders like MCI. But when MCI
HELD: (NO) already receives the dividends, then MCI’s creditors can already demand
 Under Section 13 (5) of the Corporation Law, a corporation created share from the dividends because such dividends are already the profits of
thereunder may purchase, hold, etc., and otherwise deal in such real the stockholder/MCI. So in this case, the employees can demand their
and personal property is the purpose for which the corporation was share from said profits (not strictly viewed as dividends now) by way of
formed may permit, and the transaction of its lawful business may salary increase.
reasonably and necessarily require.
 Petitioner Luneta Motor says: Its corporate purposes are Corporate Law Case Digest: Stockholders of F. Guanzon and Sons,
o to carry on a general mercantile and commercial business, Inc v. Register of Deeds of Manila (1962)
etc., and that it is authorized in its articles of incorporation G.R. No. L-18216 October 30, 1962
to operate and otherwise deal in and concerning Lessons Applicable: Strong Juridical Personality (Corporate Law)
automobiles and automobile accessories' business in all its
FACTS:
multifarious ramification and
• Sept 19, 1960: 5 stockholders of the F. Guanzon and Sons, Inc.
o to operate, etc., and otherwise dispose of vessels and boats,
executed a certificate of liquidation of the assets of the corporation,
etc., and to own and operate steamship and sailing ships
and other floating craft and deal in the same and engage in dissolution and distribution among themselves in proportion to their
shareholdings, as liquidating dividends, corporate assets, including real
the Philippine Islands and elsewhere in the transportation of
properties
persons, merchandise and chattels by water;
o all this incidental to the transportation of automobiles
• Register of Deeds of Manila denied the registration of the
 Court said that nothing in the legal provision and provisions in its
certificate of liquidation:
Articles of Incorporation could justify petitioner’s contention.
o To the contrary, they are precisely the best evidence that it
1. The number of parcels not certified to in the acknowledgment;
has no authority at all to engage in the business of land
transportation and operate a taxicab service.
2. P430.50 Reg. fees need be paid;
o That it may operate and otherwise deal in automobiles and
automobile accessories; that it may engage in the
3. P940.45 documentary stamps need be attached to the document;
transportation of persons by water does not mean that it
may engage in the business of land transportation — an
4. The judgment of the Court approving the dissolution and
entirely different line of business.
directing the disposition of the assets of the corporation need be presented
and the interests thereon. Petitioner likewise claims that laches bars
• Commissioner of Land Registration overruled ground No. 7 and respondents from recovering the subject shares.
sustained requirements Nos. 3, 5 and 6.
Issue: WON Lim Tay is the owner of the shares previously subjected to
• Stockholders appealed pledge, for him to cause the registration of said shares in his own name.

o contend that the certificate of liquidation is not a conveyance or Held: NO.


transfer but merely a distribution of the assets of the corporation which has
ceased to exist for having been dissolved Lim Tay's ownership over the shares was not yet perfected when the
Complaint was filed. The contract of pledge certainly does not make him
ISSUE: W/N certificate merely involves a distribution of the corporation's the owner of the shares pledged.
assets (or should be considered a transfer or conveyance)
Further, whether prescription effectively transferred ownership of the
HELD: NO. affirm the resolution appealed from shares, whether there was a novation of the contracts of pledge, and
• Corporation - juridical person distinct from the members whether laches had set in were difficult legal issues, which were
composing it. unpleaded and unresolved when Lim Tay asked the corporate secretary of
Go Fay to effect the transfer, in his favor, of the shares pledged to him.
o Properties registered in the name of the corporation are owned Lim Tay has failed to establish a clear legal right.
by it as an entity separate and distinct from its members.
Lim Tay's contention that he is the owner of the said shares is completely
o While shares of stock constitute personal property they do not without merit. Lim Tay does not have any ownership rights at all. At the
represent property of the corporation. time Lim Tay instituted his suit at the SEC, his ownership claim had no
prima facie leg to stand on. At best, his contention was disputable and
A share of stock only typifies an aliquot part of the corporation's uncertain.
property, or the right to share in its proceeds to that extent when
distributed according to law and equity but its holder is NOT the owner of Lim Tay cannot claim to have acquired ownership over the certificates of
any part of the capital of the corporation nor entitled to possession stock through extraordinary prescription, as provided for in Article 1132
of the Civil Code. What is required by Article 1132 is possession in the
The stockholder is not a co-owner or tenant in common of the concept of an owner. Herein, Lim Tay's possession of the stock certificates
corporate property came about because they were delivered to him pursuant to the contracts
of pledge. His possession as a pledgee cannot ripen into ownership by
Lim Tay vs. CA prescription. Lim Tay expressly repudiated the pledge, only when he filed
G.R. No. 126891 his Complaint and claimed that he was not a mere pledgee, but that he was
Date: August 5, 1998 already the owner of the shares.

Facts: Based on the foregoing, Lim Tay has not acquired the certificates of stock
through extraordinary prescription. Neither did Lim Tay acquire the shares
On 8 January 1980, Sy Guiok secured a loan from Lim Tay in the amount by virtue of a novation of the contract of pledge. Novation cannot be
of P40,000 payable within 6 months. To secure the payment of the presumed by Guiok's and Sy Lim's indorsement and delivery of the
aforesaid loan and interest thereon, Guiok executed a Contract of Pledge certificates of stock covering the 600 shares, nor Lim Tay's receipt of
in favor of Lim Tay. He pledged his 300 shares of stock in the Go Fay & dividends from 1980 to 1983, nor the fact that Guiok and Sy Lim have not
Company Inc. Guiok obliged himself to pay interest on said loan at the instituted any action to recover the shares since 1980. Novation is never
rate of 10% per annum from the date of said contract of pledge. presumed inferred.

On the same date, Alfonso Sy Lim secured a loan, from Lim Tay in the Notes: There is a contract of pledge between Guiok (respondent) and Lim
amount of P40,000 payable in 6 months. To secure the payment of his Tay (petitioner) & Sy Lim (respondent) and Lim Tay (petitioner). What
loan, Sy Lim executed a "Contract of Pledge" covering his 300 shares of was mortgaged? 300 shares of stock in the Go Fay & Company Inc. by
stock in Go Fay & Co. Under said contract, Sy Lim obliged himself to pay Guiok and 300 shares of stock in the Go Fay & Company Inc. by Sy Lim.
interest on his loan at the rate of 10% per annum from the date of the Date - January 8, 1980.
execution of said contract.
Case Digest: Majority Stockholders of Ruby v. Lim
The contract provided that Lim Tay was merely authorized to foreclose G.R. No. 165887: June 7, 2011
the pledge upon maturity of the loans, not to own them. The foreclosure is
not automatic, for it must be done in a public or private sale. MAJORITY STOCKHOLDERS OF RUBY INDUSTRIAL
CORPORATION, Petitioner, v. MIGUEL LIM et al., Respondents.
Guiok and Sy Lim endorsed their respective shares of stock in blank and
delivered the same to Lim Tay. However, Guiok and Sy Lim failed to pay VILLARAMA, JR., J.:
their respective loans and the accrued interests thereon to Lim Tay.
FACTS:
In October 1990, Lim Tay filed a "Petition for Mandamus" against Go Fay
& Co., with SEC praying that an order be issued directing the corporate Ruby Industrial Corporation (RUBY) is a domestic corporation engaged in
secretary to register the stock transfers and issue new certificates in favor glass manufacturing.Reeling from severe liquidity problems beginning in
of Lim Tay; and ordering the corporation to pay all dividends due and 1980, RUBY filed onDecember 13, 1983a petition for suspension of
unclaimed on the said certificates to Lim Tay. In the interim, Sy Lim died. payments with the Securities and Exchange Commission (SEC) docketed
Guiok and the Intestate Estate of Alfonso Sy Lim, represented by Conchita as SEC Case No. 2556.On December 20, 1983, the SEC issued an order
Lim, filed their Answer-In-Intervention with the SEC. declaring RUBY under suspension of payments and enjoining the
disposition of its properties pending hearing of the petition, except insofar
The SEC hearing officer dismissed Lim Tay's Complaint on the ground as necessary in its ordinary operations, and making payments outside of
that although the SEC had jurisdiction over the action, he failed to prove the necessary or legitimate expenses of its business.
the legal basis for the secretary of the Corporation to be compelled to
register stock transfers in favor of Lim Tay and to issue new certificates of On August 10, 1984, the SEC Hearing Panel created the management
stock under his name. His appeal was denied by SEC. He appealed with committee (MANCOM) for RUBY, composed of representatives from
CA. Allied Leasing and Finance Corporation (ALFC), Philippine Bank of
Communications (PBCOM), China Banking Corporation (China Bank),
The CA debunked Lim Tay's claim that he had acquired ownership over Pilipinas Shell Petroleum Corporation (Pilipinas Shell), and RUBY
the shares by virtue of novation, holding that Guiok's and Sy Lim's represented by Mr. Yu Kim Giang .The MANCOM was tasked to perform
endorsement and delivery of the shares were pursuant to Articles 2093 and the following functions: (1) undertake the management of RUBY; (2) take
2095 of the Civil Code and that Lim Tay's receipt of dividends was in custody and control over all existing assets and liabilities of RUBY; (3)
compliance with Article 2102 of the same Code. evaluate RUBYs existing assets and liabilities, earnings and operations;
(4) determine the best way to salvage and protect the interest of its
Arguments of Lim Tay: He contends that it has acquired ownership of the investors and creditors; and (5) study, review and evaluate the proposed
shares "through extraordinary prescription," pursuant to Article 1132 of rehabilitation plan for RUBY.
the Civil Code, and through respondents' subsequent acts, which amounted
to a novation of the contracts of pledge. Petitioner also claims that there Subsequently, two (2) rehabilitation plans were submitted to the SEC: the
was dacion en pago, in which the shares of stock were deemed sold to BENHAR/RUBY Rehabilitation Plan of the majority stockholders led by
petitioner, the consideration for which was the extinguishment of the loans
Yu Kim Giang, and the Alternative Plan of the minority stockholders minority stockholders for effecting an orderly and equitable settlement of
represented by Miguel Lim (Lim). RUBYs obligations, and compelling the majority stockholders to account
for all funds, properties and documents in their possession, and make full
Both plans were endorsed by the SEC to the MANCOM for evaluation. disclosure on the nullified credit assignments. Oblivious to these pending
incidents so crucial to the protection of the interest of the majority of
On April 26, 1991, over ninety percent (90%) of RUBYs creditors creditors and minority shareholders, the SEC simply stated that in the
objected to the Revised BENHAR/RUBY Plan and the creation of a new interim, RUBYs corporate term was validly extended, as if such extension
management committee.Instead, they endorsed the minority stockholders would provide the solution to RUBYs myriad problems.
Alternative Plan.At the hearing of the petition for the creation of a new
management committee, three (3) members of the original management Extension of corporate term requires the vote of 2/3 of the outstanding
committee (Lim, ALFC and Pilipinas Shell) opposed the Revised capital stock in a stockholders meeting called for the purpose. The actual
BENHAR/RUBY Plan on grounds that:(1) it would legitimize the entry of percentage of shareholdings in RUBY as of September 3, 1996 -- when
BENHAR, a total stranger, to RUBY as BENHAR would become the the majority stockholders allegedly ratified the board resolution approving
biggest creditor of RUBY;(2) it would put RUBYs assets beyond the reach the extension of RUBY's corporate life to another 25 years was seriously
of the unsecured creditors and the minority stockholders; and (3) it was disputed by the minority stockholders, and we find the evidence of
not approved by RUBYs stockholders in a meeting called for the purpose. compliance with the notice and quorum requirements submitted by the
majority stockholders insufficient and doubtful. Consequently, the SEC
Notwithstanding the objections of 90% of RUBYs creditors and three had no basis for its ruling denying the motion of the minority stockholders
members of the MANCOM, the SEC Hearing Panel approved on to declare as without force and effect the extension of RUBY's corporate
September 18, 1991the Revised BENHAR/RUBY Plan and dissolved the existence.
existing management committee. It also created a new management DENIED.
committee and appointed BENHAR as one of its members. In addition to
the powers originally conferred to the management committee under Ollendorf vs Abrahamson Case Digest
Presidential Decree (P.D.) No. 902-A, the new management committee William Ollendorf vs. Ira Abrahamson
was tasked to oversee the implementation by the Board of Directors of the 38 Phil. 585
revised rehabilitation plan for RUBY.
Facts: The record discloses that Ollendorf is and for a long time past has
ISSUE: Whether the minorities pre-emptive rights were violated been engaged in the city of Manila and elsewhere in the Philippines in the
business of manufacturing ladies' embroidered underwear for export.
HELD: Yes. Ollendorf imports the material from which this underwear is made and
adopts decorative designs which are embroidered upon it by Filipino
COMMERCIAL LAW: Corporation Law, Pre-emptive right needle workers from patterns selected and supplied by him. Most of the
embroidery work is done in the homes of the workers. The embroiderers
Pre-emptive right under Sec. 39 of the Corporation Code refers to the right employed by plaintiff are under contract to work for plaintiff exclusively.
of a stockholder of a stock corporation to subscribe to all issues or
disposition of shares of any class, in proportion to their respective On September 1915, plaintiff and defendant entered into a contract. Under
shareholdings. The right may be restricted or denied under the articles of the terms of this, agreement defendant entered the employ of plaintiff and
incorporation, and subject to certain exceptions and limitations. The worked for him until April 1916, when defendant, on account of ill health,
stockholder must be given a reasonable time within which to exercise their left plaintiff's employ and went to the United States. While in plaintiff's
preemptive rights. Upon the expiration of said period, any stockholder employ defendant had access to all parts of plaintiff's establishment, and
who has not exercised such right will be deemed to have waived it. had full opportunity to acquaint himself with plaintiff's business methods
and business connections. The duties performed by him were such as to
The validity of issuance of additional shares may be questioned if done in make it necessary that he should have this knowledge of plaintiff s
breach of trust by the controlling stockholders. Thus, even if the pre- business. Defendant had a general knowledge of the Philippine
emptive right does not exist, either because the issue comes within the embroidery business before his employment by plaintiff, having been
exceptions in Section 39 or because it is denied or limited in the articles of engaged in similar work for several years.
incorporation, an issue of shares may still be objectionable if the directors
acted in breach of trust and their primary purpose is to perpetuate or shift Some months after his departure, defendant returned to Manila as the
control of the corporation, or to "freeze out" the minority interest. In this manager of the Philippine Underwear Company, a corporation. This
case, the following relevant observations should have signaled greater corporation does not maintain a factory in the Philippine Islands, but sends
circumspection on the part of the SEC -- upon the third and last remand to material and embroidery designs from New York to its local representative
it pursuant to our January 20, 1998 decision -- to demand transparency and here who employs Filipino needle workers to embroider the designs and
accountability from the majority stockholders, in view of the illegal make up the garments in their homes. The only difference between
assignments and objectionable features of the Revised BENHAR/RUBY plaintiff's business and that of the firm by which the defendant is
Plan, as found by the CA and as affirmed by this Court: employed, is the method of doing the finishing work — the manufacture
of the embroidered material into finished garments.
There can be no gainsaying the well-established rule in corporate practice
and procedure that the will of the majority shall govern in all matters Shortly after defendant's return to Manila and the commencement by him
within the limits of the act of incorporation and lawfully enacted by-laws of the discharge of the duties of his position as local manager of the
not proscribed by law. It is, however, equally true that other stockholders Philippine Embroidery Company, plaintiff commenced this action, the
are afforded the right to intervene especially during critical periods in the principal purpose of which is to prevent, by injunction, any further breach
life of a corporation like reorganization, or in this case, suspension of of that part of defendant's contract of employment by plaintiff, by which
payments, more so, when the majority seek to impose their will and he agreed that he would not "enter into or engage himself directly or
through fraudulent means, attempt to siphon off Rubys valuable assets to indirectly . . . in a similar or competitive business to that of (plaintiff)
the great prejudice of Ruby itself, as well as the minority stockholders and anywhere within the Philippine Islands for a period of five years . . ." from
the unsecured creditors. the date of the agreement.

Certainly, the minority stockholders and the unsecured creditors are given Issue: Whether or not the said contract is valid.
some measure of protection by the law from the abuses and impositions of
the majority, more so in this case, considering thegive-away signs of Ruling: The contract is a valid one. The only limitation upon the freedom
private respondents perfidy strewn all over the factual landscape. Indeed, of contractual agreement is that the acts established shall not be contrary
equity cannot deprive the minority of a remedy against the abuses of the to "law, morals or public order." (Civil Code, art. 1255.)
majority, and the present action has been instituted precisely for the
purpose of protecting the true and legitimate interests of Ruby against the Public welfare is first considered, and if it be not involved, and the
Majority Stockholders. On this score, the Supreme Court, has ruled that: restraint upon one party is not greater than protection to the other party
requires, the contract may be sustained. The question is whether, under the
"Generally speaking, the voice of themajority of the stockholders is the particular circumstances of the case and the nature of the particular
law of the corporation, but there are exceptions to this rule. There must contract involved in it the contract is, or is not, unreasonable.
necessarily be a limit upon the power of the majority. Without such a limit
the will of the majority will be absolute and irresistible and might easily The Courts adopt the modern rule that the validity of restraints upon trade
degenerate into absolute tyranny.x x x" (Additional emphasis supplied.) or employment is to be determined by the intrinsic reasonableness of the
restriction in each case, rather than by any fixed rule, and that such
Lamentably, the SEC refused to heed the plea of the minority stockholders restrictions may be upheld when not contrary to the public welfare and not
and MANCOM for the SEC to order RUBY to commence liquidation greater than is necessary to afford a fair and reasonable protection to the
proceedings, which is allowed under Sec. 4-9 of the Rules on Corporate party in whose favor it is imposed.
Recovery. Under the circumstances, liquidation was the only hope of the
A business enterprise may and often does depend for its success upon the While the general rule is that the portion of a decision that becomes the
owner's relations with other dealers, his skill in establishing favorable subject of execution is that ordained or decreed in the dis-positive part
connections, his methods of buying and selling — a multitude of details, thereof, there are recognized exceptions to this rule, one of which is where
none vital if considered alone, but which in the aggregate constitute the extensive and explicit discussion and settlement of the issue is found in the
sum total of the advantages which are the result of the experience or body of the decision. The Republic of the Philippines (Republic) filed
individual aptitude and ability of the man or men by whom the business before the Sandiganbayan a “Complaint for Reconveyance, Reversion,
has been built up. Failure or success may depend upon the possession of Accounting, Restitution and Damages,” of the alleged ill-gotten wealth of
these intangible but all-important assets, and it is natural that their the Marcoses which have been invested in the Philippine Long Distance
possessor should seek to keep them from falling into the hands of his Telecommunication Corporation (PLDT). Ramon and Imelda Cojuangco
competitors. (Spouses Cojuangco) were subsequently impleaded. The Sandiganbayan
dismissed the complaint with respect to the recovery of the PLDT shares.
It is with this object in view that such restrictions as that now under The Republic appealed to the Supreme Court, and the same issued a
consideration are written into contracts of employment. Their purpose is favorable ruling. The Republic thereafter filed with the Sandiganbayan a
the protection of the employer, and if they do not go beyond what is Motion for the Issuance of a Writ of Execution, praying for the
reasonably necessary to effectuate this purpose they should be upheld. We cancellation of the shares of stock registered in the name of Prime
are of the opinion, and so hold, that in the light of the established facts the Holdings and the annotation of the change of ownership on PTIC‘s Stock
restraint imposed upon defendant by his contract is not unreasonable. and Transfer Book. The Republic further prayed for the issuance of an
order for PTIC to account for all cash and stock dividends declared by
Corporate Law Case Digest: Ponce v. Alsons Cement Corp. (2002) PLDT in favor of PTIC from 1986 up to the present including
G.R. NO. 139802 December 10, 2002 compounded interests. The Sandiganbayan granted the same, except its
Lessons Applicable: prayer for accounting of dividends. The Republic moved for
• Nature of Certificate of Stock (Corporate Law) reconsideration with respect to the denial of accounting of dividends,
• Remedy if Registration is Refused (Corporate Law) which the Sandiganbayan granted. The Cojuangcos protested, alleging that
the SC‘s decision did not include in its dispositive portion the grant of
FACTS: dividends and interests accruing to the shares adjudicated in favor of the
• February 8, 1968: Vicente C. Ponce and Fausto Gaid, Republic.
incorporator of Victory Cement Corporation (VCC), executed a “Deed of ISSUE:
Undertaking” and “Indorsement” whereby Gaid acknowledges that Ponce Whether or not the Republic is entitled to the dividends and interests
is the owner of the shares and he was therefore assigning/endorsing it to accruing to the shares despite its non-inclusion in the dis-positive portion
Ponce of the decision
• VCC was renamed Floro Cement Corporation (FCC) and then to HELD:
Alsons Cement Corporation (ACC) The Cojuangcos insist on a literal reading of the dis-positive portion of the
• Up to the present, no certificates of stock corresponding to the SC‘s Decision, excluding the dividends, interests, and earnings accruing to
239,500 subscribed and fully paid shares of Gaid were issued in the name the shares of stock from being accounted for and remitted. The SC, in
of Fausto G. Gaid and/or the plaintiff. directing the re-conveyance to the Republic of the 111,415 shares of
• Despite repeated demands, the ACC refused to issue the PLDT stock owned by PTIC in the name of Prime Holdings, declared the
certificates of stocks Republic as the owner of said shares and, necessarily, the dividends and
• SEC Hearing Officer Enrique L. Flores, Jr. granted the motion interests accruing thereto. Ownership is a relation in law by virtue of
to dismiss which a thing pertaining to one person is completely subjected to his will
• Upon appeal, the Commission En Banc reversed the decision of in everything not prohibited by law or the concurrence with the rights of
the Hearing Officer another. Its traditional elements or attributes include jus utendi or the right
• Ponce, filed a complaint with the SEC for mandamus to receive from the thing that it produces. Contrary to the Cojuangcos‘
• CA: mandamus should be dismissed for failure to state a cause contention, while the general rule is that the portion of a decision that
of action becomes the subject of execution is that ordained or decreed in the dis-
o in the absence of any allegation that the transfer of the shares positive part thereof, there are recognized exceptions to this rule, viz: (a)
was registered in the stock and transfer book where there is ambiguity or uncertainty, the body of the opinion may be
ISSUE: W/N the cert. of stocks of Gaid can be transferred to Ponce referred to for purposes of construing the judgment, because the dis-
positive part of a decision must find support from the decision‘s ratio
HELD: NO. petition Denied. decidendi; and (b) where extensive and explicit discussion and settlement
• SEC. 63. Certificate of stock and transfer of shares.–The capital of the issue is found in the body of the decision. In the Decision, although
stock of stock corporations shall be divided into shares for which the inclusion of the dividends, interests, and earnings of the 111,415 PTIC
certificates signed by the president or vice-president, countersigned by the shares as belonging to the Republic was not mentioned in the dis-positive
secretary or assistant secretary, and sealed with the seal of the corporation portion of the Court‘s Decision, it is clear from its body that what was
shall be issued in accordance with the by-laws. Shares of stock so issued being adjudicated in favor of the Republic was the whole block of shares
are personal property and may be transferred by delivery of the certificate and the fruits thereof, said shares having been found to be part of the
or certificates indorsed by the owner or his attorney-in-fact or other person Marcoses‘ ill- gotten wealth, and therefore, public money.
legally authorized to make the transfer. No transfer, however, shall be
valid, except as between the parties, until the transfer is recorded in the Republic Planters Bank vs. Agana
books of the corporation so as to show the names of the parties to the [GR 51765, 3 March 1997]
transaction, the date of the transfer, the number of the certificate or
certificates and the number of shares transferred. Facts: On 18 September 1961, the Robes-Francisco Realty &
No shares of stock against which the corporation holds any unpaid Development Corporation (RFRDC) secured a loan from the Republic
claim shall be transferable in the books of the corporation. Planters Bank in the amount of P120,000.00. As part of the proceeds of
• the stock and transfer book is the basis for ascertaining the the loan, preferred shares of stocks were issued to RFRDC through its
persons entitled to the rights and subject to the liabilities of a stockholder officers then, Adalia F. Robes and one Carlos F. Robes. In other words,
o Where a transferee is not yet recognized as a stockholder, the instead of giving the legal tender totaling to the full amount of the loan,
corporation is under no specific legal duty to issue stock certificates in the which is P120,000.00, the Bank lent such amount partially in the form of
transferee’s name. money and partially in the form of stock certificates numbered 3204 and
• in a case such as that at bar, a mandamus should not issue to 3205, each for 400 shares with a par value of P10.00 per share, or for
compel the secretary of a corporation to make a transfer of the stock on the P4,000.00 each, for a total of P8,000.00. Said stock certificates were in the
books of the company name of Adalia F. Robes and Carlos F. Robes, who subsequently,
o unless it affirmatively appears that he has failed or refused so to however, endorsed his shares in favor of Adalia F. Robes.
do, upon the demand either of the person in whose name the stock is
registered, or of some person holding a power of attorney for that purpose Said certificates of stock bear the following terms and conditions: "The
from the registered owner of the stock. Preferred Stock shall have the following rights, preferences, qualifications
mere indorsee of a stock certificate, claiming to be the owner, and limitations, to wit: 1. Of the right to receive a quarterly dividend of
will not necessarily be recognized as such by the corporation and its 1%, cumulative and participating. xxx 2. That such preferred shares may
officers, in the absence of express instructions of the registered owner to be redeemed, by the system of drawing lots, at any time after 2 years from
make such transfer to the indorsee, or a power of attorney authorizing such the date of issue at the option of the Corporation." On 31 January 1979,
transfer RFRDC and Robes proceeded against the Bank and filed a complaint
• mandamus - proper remedy to make him the rightful owner and anchored on their alleged rights to collect dividends under the preferred
holder of a stock certificate to be issued in his name shares in question and to have the bank redeem the same under the terms
and conditions of the stock certificates. The bank filed a Motion to
• IMELDA O. COJUANGCO et al. v. SANDIGANBAYAN et al. Dismiss 3 private respondents' Complaint on the following grounds: (1)
586 SCRA 790 (2009) that the trial court had no jurisdiction over the subject-matter of the action;
(2) that the action was unenforceable under substantive law; and (3) that
the action was barred by the statute of limitations and/or laches. The HELD: NO. remaining members of the board of trustees of GCHS may
bank's Motion to Dismiss was denied by the trial court in an order dated convene and fill up the vacancies in the board
16 March 1979. The bank then filed its Answer on 2 May 1979. • Except as provided, the vote necessary to approve a particular
Thereafter, the trial court gave the parties 10 days from 30 July 1979 to corporate act as provided in this Code shall be deemed to refer only to
submit their respective memoranda after the submission of which the case stocks with voting rights:
would be deemed submitted for resolution. On 7 September 1979, the trial o 1. Amendment of the articles of incorporation;
court rendered the decision in favor of RFRDC and Robes; ordering the o 2. Adoption and amendment of by-laws;
bank to pay RFRDC and Robes the face value of the stock certificates as o 3. Sale, lease, exchange, mortgage, pledge or other disposition
redemption price, plus 1% quarterly interest thereon until full payment. of all or substantially all of the corporation property;
The bank filed the petition for certiorari with the Supreme Court, o 4. Incurring, creating or increasing bonded indebtedness;
essentially on pure questions of law. o 5. Increase or decrease of capital stock;
o 6. Merger or consolidation of the corporation with another
Issue: corporation or other corporations;
1. Whether the bank can be compelled to redeem the preferred o 7. Investment of corporate funds in another corporation or
shares issued to RFRDC and Robes. business in accordance with this Code; and
2. Whether RFRDC and Robes are entitled to the payment of o 8. Dissolution of the corporation.
certain rate of interest on the stocks as a matter of right without necessity • quorum in a members’ meeting is to be reckoned as the actual
of a prior declaration of dividend. number of members of the corporation
Held: • stock corporations - shareholders may generally transfer their
shares
1. While the stock certificate does allow redemption, the option to do so o on the death of a shareholder, the executor or administrator duly
was clearly vested in the bank. The redemption therefore is clearly the appointed by the Court is vested with the legal title to the stock and
type known as "optional". Thus, except as otherwise provided in the stock entitled to vote it
certificate, the redemption rests entirely with the corporation and the o Until a settlement and division of the estate is effected, the
stockholder is without right to either compel or refuse the redemption of stocks of the decedent are held by the administrator or executor
its stock. Furthermore, the terms and conditions set forth therein use the • nonstock corporation - personal and non-transferable unless the
word "may". It is a settled doctrine in statutory construction that the word articles of incorporation or the bylaws of the corporation provide
"may" denotes discretion, and cannot be construed as having a mandatory otherwise
effect. The redemption of said shares cannot be allowed. The Central Bank o Section 91 of the Corporation Code: termination extinguishes all
made a finding that the Bank has been suffering from chronic reserve the rights of a member of the corporation, unless otherwise provided in the
deficiency, and that such finding resulted in a directive, issued on 31 articles of incorporation or the bylaws.
January 1973 by then Gov. G. S. Licaros of the Central Bank, to the o whether or not "dead members" are entitled to exercise their
President and Acting Chairman of the Board of the bank prohibiting the voting rights (through their executor or administrator), depends on those
latter from redeeming any preferred share, on the ground that said articles of incorporation or bylaws
redemption would reduce the assets of the Bank to the prejudice of its By-Laws of GCHS: membership in the corporation shall be
depositors and creditors. Redemption of preferred shares was prohibited terminated by the death of the member
for a just and valid reason. The directive issued by the Central Bank With 11 remaining members, the quorum = 6.
Governor was obviously meant to preserve the status quo, and to prevent • SECTION 29. Vacancies in the office of director or trustee. --
the financial ruin of a banking institution that would have resulted in Any vacancy occurring in the board of directors or trustees other than by
adverse repercussions, not only to its depositors and creditors, but also to removal by the stockholders or members or by expiration of term, may be
the banking industry as a whole. The directive, in limiting the exercise of a filled by the vote of at least a majority of the remaining directors or
right granted by law to a corporate entity, may thus be considered as an trustees, if still constituting a quorum; otherwise, said vacancies must be
exercise of police power. filled by the stockholders in a regular or special meeting called for that
purpose. A director or trustee so elected to fill a vacancy shall be elected
2. Both Section 16 of the Corporation Law and Section 43 of the present only for the unexpired term of his predecessor in office.
Corporation Code prohibit the issuance of any stock dividend without the o the filling of vacancies in the board by the remaining directors
approval of stockholders, representing not less than two-thirds (2/3) of the or trustees constituting a quorum is merely permissive, not mandatory
outstanding capital stock at a regular or special meeting duly called for the either by the remaining directors constituting a quorum, or by
purpose. These provisions underscore the fact that payment of dividends the stockholders or members in a regular or special meeting called for the
to a stockholder is not a matter of right but a matter of consensus. purpose
Furthermore, "interest bearing stocks", on which the corporation agrees By-Laws of GCHS prescribed the specific mode of filling up
absolutely to pay interest before dividends are paid to common existing vacancies in its board of directors; that is, by a majority vote of
stockholders, is legal only when construed as requiring payment of interest the remaining members of the board
as dividends from net earnings or surplus only. In compelling the bank to remaining member-trustees must sit as a board (as a body in a
redeem the shares and to pay the corresponding dividends, the Trial lawful meeting) in order to validly elect the new ones
committed grave abuse of discretion amounting to lack or excess of
jurisdiction in ignoring both the terms and conditions specified in the Republic vs. Cocofed
stock certificate, as well as the clear mandate of the law. REPUBLIC v. COCOFED, G.R. No. 147062-64, December 14, 2001
(Coconut levy funds are prima facie public funds which should be
Corporate Law Case Digest: Tan v. Sycip (2006) subjected to COA audit)
Facts:
G.R. No. 153468 August 17, 2006 The PCGG issued and implemented numerous sequestrations, freeze
Lessons Applicable: Release from Subscription Obligation (Corporate orders and provisional takeovers of allegedly ill-gotten companies, assets
Law) and properties, real or personal. Among the properties sequestered by the
Commission were shares of stock in the United Coconut Planters Bank
FACTS: (UCPB) registered in the names of the alleged “one million coconut
• Grace Christian High School (GCHS) is a nonstock, non-profit farmers,” the so-called Coconut Industry Investment Fund companies
educational corporation w/ 15 regular members, who also constitute the (CIIF companies) and Private Respondent Eduardo Cojuangco Jr. In
board of trustees. connection with the sequestration of the said UCPB shares, the PCGG, on
• April 6, 1998: During the annual members’ meeting only 11 July 31, 1987, instituted an action for reconveyance, reversion,
living member-trustees, as 4 had already died. accounting, restitution and damages docketed as Case No. 0033 in the
o 7 attended the meeting through their respective proxies. Sandiganbayan.
o The meeting was convened and chaired by Atty. Sabino Padilla On November 15, 1990, upon Motion of Private Respondent COCOFED,
Jr. over the objection of Atty. Antonio C. Pacis, who argued that there was the Sandiganbayan issued a Resolution lifting the sequestration of the
no quorum. subject UCPB shares on the ground that herein private respondents – in
o In the meeting, Petitioners Ernesto Tanchi, Edwin Ngo, Virginia particular, COCOFED and the so-called CIIF companies – had not been
Khoo, and Judith Tan were voted to replace the 4 deceased member- impleaded by the PCGG as parties-defendants in its July 31, 1987
trustees. Complaint for reconveyance, reversion, accounting, restitution and
• SEC: meeting void due to lack of quorum (NOT living but based damages.
on AIC) This Sandiganbayan Resolution was challenged by the PCGG in a Petition
o Sec 24 read together with Sec 89 for Certiorari docketed as GR No. 96073 in this Court. Meanwhile, upon
• CA: Dismissed due to technicalities motion of Cojuangco, the anti-graft court ordered the holding of elections
ISSUE: W/N dead members should still be counted in the quorum - NO for the Board of Directors of UCPB. However, the PCGG applied for and
based on by-laws was granted by this Court a Restraining Order enjoining the holding of the
election. Subsequently, the Court lifted the Restraining Order and ordered
the UCPB to proceed with the election of its board of directors.
Furthermore, it allowed the sequestered shares to be voted by their “The coconut levy funds being ‘clearly affected with public interest, it
registered owners. follows that the corporations formed and organized from those funds, and
On February 23, 2001, “COCOFED, et al. and Ballares, et al.” filed the all assets acquired therefrom should also be regarded as ‘clearly affected
“Class Action Omnibus Motion” referred to earlier in Sandiganbayan with public interest.’”
Civil Case Nos. 0033-A, 0033-B and 0033-F, asking the court a quo: “The utilization and proper management of the coconut levy funds, raised
“1. To enjoin the PCGG from voting the UCPB shares of stock registered as they were by the State’s police and taxing powers, are certainly the
in the respective names of the more than one million coconut farmers; and concern of the Government. It cannot be denied that it was the welfare of
“2. To enjoin the PCGG from voting the SMC shares registered in the the entire nation that provided the prime moving factor for the imposition
names of the 14 CIIF holding companies including those registered in the of the levy. It cannot be denied that the coconut industry is one of the
name of the PCGG.” major industries supporting the national economy. It is, therefore, the
Issue: State’s concern to make it a strong and secure source not only of the
Who may vote the sequestered UCPB shares while the main case for their livelihood of a significant segment of the population but also of export
reversion to the State is pending in the Sandiganbayan? earnings the sustained growth of which is one of the imperatives of
Ruling: economic stability. The coconut levy funds are clearly affected with public
This Court holds that the government should be allowed to continue interest. Until it is demonstrated satisfactorily that they have legitimately
voting those shares inasmuch as they were purchased with coconut levy become private funds, they must prima facie and by reason of the
funds – funds that are prima facie public in character or, at the very least, circumstances in which they were raised and accumulated be accounted
are “clearly affected with public interest.” subject to the measures prescribed in E.O. Nos. 1, 2, and 14 to prevent
General Rule: Sequestered Shares Are Voted by the Registered Holder their concealment, dissipation, etc., which measures include the
At the outset, it is necessary to restate the general rule that the registered sequestration and other orders of the PCGG complained of.” (Italics
owner of the shares of a corporation exercises the right and the privilege supplied)
of voting. (Sec. 24, BP 68) This principle applies even to shares that are To repeat, the foregoing juridical situation has not changed. It is still the
sequestered by the government, over which the PCGG as a mere truth today: “the coconut levy funds are clearly affected with public
conservator cannot, as a general rule, exercise acts of dominion. On the interest.”
other hand, it is authorized to vote these sequestered shares registered in To stress, the two-tiered test is applied only when the sequestered asset in
the names of private persons and acquired with allegedly ill-gotten wealth, the hands of a private person is alleged to have been acquired with ill-
if it is able to satisfy the two-tiered test devised by the Court in Cojuangco gotten wealth. Hence, in PCGG v. Cojuangco, we allowed Eduardo
v. Calpo (G.R. No. 115352, June 10, 1993) and PCGG v. Cojuangco Jr., Cojuangco Jr. to vote the sequestered shares of the San Miguel
(133197, Jan. 27, 1999) as follows: Corporation (SMC) registered in his name but alleged to have been
(1) Is there prima facie evidence showing that the said shares are ill-gotten acquired with ill-gotten wealth. We did so on his representation that he
and thus belong to the State? had acquired them with borrowed funds and upon failure of the PCGG to
(2) Is there an imminent danger of dissipation, thus necessitating their satisfy the “two-tiered” test. This test was, however, not applied to
continued sequestration and voting by the PCGG, while the main issue is sequestered SMC shares that were purchased with coco levy funds.
pending with the Sandiganbayan? In the present case, the sequestered UCPB shares are confirmed to have
Sequestered Shares Acquired with Public Funds Are an Exception been acquired with coco levies, not with alleged ill-gotten wealth. Hence,
From the foregoing general principle, the Court in Baseco v. PCGG by parity of reasoning, the right to vote them is not subject to the “two-
(“Baseco”) and Cojuangco Jr. v. Roxas, G.R. No. 91925, April 16, 1991) tiered test” but to the public character of their acquisition, which per
(“Cojuangco-Roxas”) has provided two clear “public character” Antiporda v. Sandiganbayan cited earlier, must first be determined.
exceptions under which the government is granted the authority to vote the Coconut Levy Funds Are Prima Facie Public Funds
shares: To avoid misunderstanding and confusion, this Court will even be more
(1) Where government shares are taken over by private persons or entities categorical and positive than its earlier pronouncements: the coconut levy
who/which registered them in their own names, and funds are not only affected with public interest; they are, in fact, prima
(2) Where the capitalization or shares that were acquired with public funds facie public funds. Public funds are those moneys belonging to the State or
somehow landed in private hands. to any political subdivision of the State; more specifically, taxes, customs
The exceptions are based on the common-sense principle that legal fiction duties and moneys raised by operation of law for the support of the
must yield to truth; that public property registered in the names of non- government or for the discharge of its obligations. (Beckner v.
owners is affected with trust relations; and that the prima facie beneficial Commonwealth, 5 SE2d 525, Nov. 20, 1939) Undeniably, coconut levy
owner should be given the privilege of enjoying the rights flowing from funds satisfy this general definition of public funds, because of the
the prima facie fact of ownership. following reasons:
The “public character” test was reiterated in many subsequent cases; most 1. Coconut levy funds are raised with the use of the police and taxing
recently, in Antiporda v. Sandiganbayan. (G.R. No. 116941, May 31, powers of the State.
2001) Expressly citing Cojuangco-Roxas, this Court said that in 2. They are levies imposed by the State for the benefit of the coconut
determining the issue of whether the PCGG should be allowed to vote industry and its farmers.
sequestered shares, it was crucial to find out first whether these were 3. Respondents have judicially admitted that the sequestered shares were
purchased with public funds, as follows: purchased with public funds.
“It is thus important to determine first if the sequestered corporate shares 4. The Commission on Audit (COA) reviews the use of coconut levy
came from public funds that landed in private hands.” In short, when funds.
sequestered shares registered in the names of private individuals or entities 5. The Bureau of Internal Revenue (BIR), with the acquiescence of private
are alleged to have been acquired with ill-gotten wealth, then the two- respondents, has treated them as public funds.
tiered test is applied. However, when the sequestered shares in the name of 6. The very laws governing coconut levies recognize their public
private individuals or entities are shown, prima facie, to have been (1) character.
originally government shares, or (2) purchased with public funds or those We shall now discuss each of the foregoing reasons (among others), any
affected with public interest, then the two-tiered test does not apply. one of which is enough to show their public character.
Rather, the public character exceptions in Baseco v. PCGG and Cojuangco xxx
Jr. v. Roxas prevail; that is, the government shall vote the shares. 3. Respondents Judicially Admit That the Levies Are Government Funds.
UCPB Shares Were Acquired With Coconut Levy Funds Equally important as the fact that the coconut levy funds were raised
In the present case before the Court, it is not disputed that the money used through the taxing and police powers of the State is respondents’ effective
to purchase the sequestered UCPB shares came from the Coconut judicial admission that these levies are government funds. As shown by
Consumer Stabilization Fund (CCSF), otherwise known as the coconut the attachments to their pleadings, respondents concede that the Coconut
levy funds. This fact was plainly admitted by private respondent’s counsel, Consumers Stabilization Fund (CCSF) and the Coconut Investment
Atty. Teresita J. Herbosa, during the Oral Arguments held on April 17, Development Fund “constitute government funds x x x for the benefit of
2001 in Baguio City. Indeed in Cocofed v. PCGG, this Court categorically coconut farmers.”
declared that the UCPB was acquired “with the use of the Coconut 4. The COA Audit Shows the Public Nature of the Funds.
Consumers Stabilization Fund in virtue of Presidential Decree No. 755, Under COA Office Order No. 86-9470 dated April 15, 1986, the COA
promulgated on July 29, 1975.” reviewed the expenditure and use of the coconut levies allocated for the
acquisition of the UCPB. The audit was aimed at ascertaining whether
Coconut Levy Funds Are Affected With Public Interest these were utilized for the purpose for which they had been intended.
Having conclusively shown that the sequestered UCPB shares were Because these funds have been subjected to COA audit, there can be no
purchased with coconut levies, we hold that these funds and shares are, at other conclusion than that they are prima facie public in character.
the very least, “affected with public interest.” The Resolution issued by Having shown that the coconut levy funds are not only affected with
the Court on February 16, 1993 in Republic v. Sandiganbayan (G.R. No. public interest, but are in fact prima facie public funds, this Court believes
96073, stated that coconut levy funds were “clearly affected with public that the government should be allowed to vote the questioned shares,
interest”; thus, herein private respondents – even if they are the registered because they belong to it as the prima facie beneficial and true owner.
shareholders – cannot be accorded the right to vote them. We quote the In sum, we hold that the Sandiganbayan committed grave abuse of
said Resolution in part, as follows: discretion in grossly contradicting and effectively reversing existing
jurisprudence, and in depriving the government of its right to vote the Petitioners denied violating Sec. 74 and reiterated the allegations
sequestered UCPB shares which are prima facie public in character. contained in their complaint in Civ. Case 4257-MC. They alleged that
The Petition is hereby GRANTED and the assailed Order SET ASIDE. Eduardo consistently pressured petitioner Flordeliza, his daughter, to
The PCGG shall continue voting the sequestered shares until improperly transfer ownership of the corporations’ V.A.G building to him.
Sandiganbayan Civil Case Nos. 0033-A, 0033-B and 0033-F are finally When the proposed transfer of the V.A.G. building did not materialize,
and completely resolved. petitioners claim that Eduardo instituted an action to compel donation of
said property to him. Moreover, they claim that Eduardo attempted to
GSIS v. CA (G.R. No. 183905) forcibly evict petitioner Jason from his office at VMC so he can occupy
Facts: the same; that Eduardo and his cohorts constantly created trouble by
GSIS, a major shareholder in Meralco, was distressed over the proxy intervening in the daily operations of the corporations without the
validation proceedings and the resulting certification of proxies in favor of knowledge or consent of the board of directors.
the Meralco Management. The proceedings were presided over by ISSUE: Whether the petitioners violated Sec, 74 of the Corporation Code
Meralco’s assistant corporate secretary and chief legal counsel instead of of the PH.
the person duly designated by Meralco’s Board of Directors. Thus, GSIS RULING: NO. The stockholders’ right to inspect corporate books is not
moved before the SEC to declare certain proxies, those issued to herein without limitations. It is now expressly required as a condition for such
private respondents, as invalid. Private respondents contend that dispute in examination that the one requesting it must not have been guilty of using
the validity of proxies is an election contest which falls under the trial improperly any information secured through a prior examination, or that
court’s jurisdiction. GSIS argues there was no election yet at the time it the person asking for such examination must be acting in good faith and
filed its petition with the SEC, hence no proper election contest over for a legitimate purpose in making his demand.
which the regular courts may have jurisdiction. In order therefore for the penal provision under Sec 144 of the Corporation
Issue: Code to apply in a case of violation of a stockholder or members right to
Whether or not the proxy challenge is an election contest cognizable by inspect the corporate books/records the elements provided for under Sec
the regular courts. 74 must be present.
Ruling: YES. The petition is GRANTED.
Section 2, Rule 6 of the Interim Rules broadly defines the term “election Shiou v. Chim
contest” as encompassing all plausible incidents arising from the election
of corporate directors, including: (1) any controversy or dispute involving SY TIONG SHIOU v. SY CHIM and FELICIDAD CHAN SY, (G)
title or claim to any elective office in a stock or non-stock corporation, (2) G.R. No. 174168, March 30, 2009
the validation of proxies, (3) the manner and validity of elections and (4)
the qualifications of candidates, including the proclamation of winners. FACTS:
Under Section 5(c) of Presidential Decree No. 902-A, in relation to the • February 3 2003, Juanita Tan, corporate treasurer of Sy Siy Ho
SRC, the jurisdiction of the regular trial courts with respect to election- & Sons, Inc. (the corporation), a family corporation doing business under
related controversies is specifically confined to “controversies in the the name and style Guan Yiac Hardware, submitted a letter to the
election or appointment of directors, trustees, officers or managers of corporation’s Board of Directors (Board) statingthat Felicidad Chan Sy
corporations, partnerships, or associations.” Evidently, the jurisdiction of did not make cash deposits to any of the corporation’s banks from 1
the regular courts over so-called election contests or controversies under November 2001 to 31 January 2003, thus the total bank remittances for the
Section 5(c) does not extend to every potential subject that may be voted past years were less than reflected in the corporate financial statements,
on by shareholders, but only to the election of directors or trustees, in accounting books and records. Finally, Juanita Tan sought to be free from
which stockholders are authorized to participate under Section 24 of the any responsibility over all corporate funds.
Corporation Code. • April 5, 2003, Banaria, Banaria & Company in its report, the
The power of the SEC to investigate violations of its rules on proxy accounting firm attributed to the Spouses Sy P67,117,230.30 as
solicitation is unquestioned when proxies are obtained to vote on matters unaccounted receipts and disbursements from 1994 to 2002.
unrelated to the cases enumerated under Section 5 of Presidential Decree • April 15, 2003, a demand letter was subsequently served on the
No. 902-A. However, when proxies are solicited in relation to the election Spouses Sy. On the same date, the children of the Spouses Sy allegedly
of corporate directors, the resulting controversy, even if it ostensibly stole from the corporation cash, postdated checks and other important
raised the violation of the SEC rules on proxy solicitation, should be documents. After the incident, the Spouses Sy allegedly transferred
properly seen as an election controversy within the original and exclusive residence and ceased reporting to the corporation. Thereupon, the
jurisdiction of the trial courts by virtue of Section 5.2 of the SRC in corporation filed a criminal complaint for robbery against the Spouses Sy
relation to Section 5(c) of Presidential Decree No. 902-A. before the City Prosecutor’s Office of Manila.
That the proxy challenge raised by GSIS relates to the election of the • July 1, 2003, the corporation, through Romer S. Tan, filed its
directors of Meralco is undisputed. The controversy was engendered by Amended Complaint for Accounting and Damages against the Spouses Sy
the looming annual meeting, during which the stockholders of Meralco before the RTC Manila, praying for a complete and true accounting of all
were to elect the directors of the corporation. GSIS very well knew of that the amounts paid to, received and earned by the company since 1993 and
fact. for the restitution of the said amount.The complaint also prayed for a
temporary restraining order (TRO) and or preliminary injunction to
ANG-ABAYA v. ANG G.R. No. 178511 restrain Sy Chim from calling a stockholders’ meeting on the ground of
Topic: Sec. 74 Corporation Code lack of authority.
ANG-ABAYA v. ANG G.R. No. 178511, December 4, 2008, Ynares- • September 9, 2003, the Spouses Sy filed their Motion for Leave
Santiago, J.: to File Third-Party Complaint, praying that their attached Third Party
The stockholder’s right of inspection of the corporation’s book and Complaint be allowed and admitted against Sy Tiong Shiou and his
records is based upon their ownership of the assets and property of the spouse. In the said third-party complaint, the Spouses Sy accused Sy
corporation. It is, therefore, an incident of ownership of the corporate Tiong Shiou and Juanita Tan as directly liable for the corporation’s claim
property, whether this ownership or interest be termed an equitable for misappropriating corporate funds.
ownership, a beneficial ownership, or a quasi-ownership. This right is • October 8, 2003, the trial court granted the motion for leave to
predicated upon the necessity of self-protection. The inspection has to be file the third-party complaint, and forthwith directed the issuance of
germane to the petitioner’s interest as a stockholder, and has to be proper summons against Sy Tiong Shiou and Juanita Tan.
and lawful in character and not inimical to the interest of the corporation. • January 16, 2004, their counsel allegedly discovered that Sy
FACTS: Vibelle Manufacturing Corporation (VMC) and Genato Tiong Shiou and Juanita Tan were not furnished with the copies of several
Investments, Inc. (Genato) are family-owned corporations, where pleadings, as well as a court order, which resulted in their having been
petitioners and private respondent Eduardo Ang are shareholders, officers declared in default for failure to file their answer to the third-party
and members of the board of directors. complaint; thus, they instead filed a petition for certiorari before the Court
Prior to the instant controversy, VMC, Genato, and Oriano Manufacturing of Appeals.
Corp. filed a Civil case 4257-MC against Eduardo, together with Michael, • May 26, 2004, the Court of Appeals granted the petition of Sy
and some other persons for allegedly conniving to fraudulently wrest Tiong Shiou and Juanita Tan.61The appellate court declared that a third-
control/management of the corporations. party complaint is not allowed under the Interim Rules of Procedure
During the pendency of the above-mentioned Civil Case, particularly in Governing Intra-Corporate Controversies Under R.A. No. 8799 (Interim
July 2004, Eduardo sought permission to inspect the corporate books of Rules).
VMC and Genato on account of petitioners alleged failure to update him
on the financial and business activities of these family corporations. ISSUE:
Petitioners denied the request claiming that Eduardo would use the • Whether or not a third-party complaint is prohibited by the
information obtained from said inspection for purposes inimical to the Interim Rules.
corporations’ interests. Because of petitioners’ refusal to grant Eduardo’s
request, the latter filed an Affidavit-Complaint against petitioners, HELD:
charging them with violation of two counts of Sec. 74 (Corporation Code), • No, the third-party complaint should be allowed. For while a
in relation to Sec. 144. third-party complaint is not included in the allowed pleadings, neither is it
among the prohibited ones. Nevertheless, this conflict may be resolved by
following the well-entrenched rule in statutory construction, that every Western Institute of Technology Inc. vs. Salas Case Digest
part of the statute must be interpreted with reference to the context, i.e., Western Institute of Technology Inc. vs. Salas
that every part of the statute must be considered together with the other [GR 113032, 21 August 1997]
parts, and kept subservient to the general intent of the whole enactment.
Statutes, including rules, should be construed in the light of the object to Facts: Ricardo T. Salas, Salvador T. Salas, Soledad Salas-Tubilleja,
be achieved and the evil or mischief to be suppressed and they should be Antonio S. Salas, and Richard S. Salas, belonging to the same family, are
given such construction as will advance the object, suppress the mischief the majority and controlling members of the Board of Trustees of Western
and secure the benefits intended. A statute should therefore be read with Institute of Technology, Inc. (WIT), a stock corporation engaged in the
reference to its leading idea, and its general purpose and intention should operation, among others, of an educational institution. According to
be gathered from the whole act, and this predominant purpose will prevail Homero L. Villasis, Dimas Enriquez, peston F. Villasis, and Reginald F.
over the literal import of particular terms or clauses, if plainly apparent, Villasis, the minority stockholders of WIT, sometime on 1 June 1986 in
operating as a limitation upon some and as a reason for expanding the the principal office of WIT at La Paz, Iloilo City, a Special Board Meeting
signification of others, so that the interpretation may accord with the spirit was held. In attendance were other members of the Board including
of the entire act, and so that the policy and object of the statute as a whole Reginald Villasis. Prior to said Special Board Meeting, copies of notice
may be made effectual and operative to the widest possible extent. thereof, dated 24 May 1986, were distributed to all Board Members.The
Otherwise stated, the spirit, rather than the letter of a law determines its notice allegedly indicated that the meeting to be held on 1 June 1986
construction; hence, a statute, as in the rules in this case, must be read included Item 6 which states that "Possible implementation of Art. III,
according to its spirit and intent Sec. 6 of the Amended By-Laws of Western Institute of Technology, Inc.
on compensation of all officers of the corporation." In said meeting, the
Bitong v. CA (G.R. No. 123553) Board of Trustees passed Resolution 48, series 1986, granting monthly
Facts: compensation to Salas, et. al. as corporate officers retroactive 1 June 1985,
Petitioner Bitong allegedly acting for the benefit of Mr. & Ms. Co. filed a in the following amounts: “Chairman 9,000.00/month, Vice Chairman
derivative suit before the SEC against respondent spouses Apostol, who P3,500.00/month, Corporate Treasurer P3,500.00/month and Corporate
were officers in said corporation, to hold them liable for fraud and Secretary P3,500.00/month, retroactive June 1, 1985 and the ten
mismanagement in directing its affairs. Respondent spouses moved to percentum of the net profits shall be distributed equally among the ten
dismiss on the ground that petitioner had no legal standing to bring the suit members of the Board of Trustees. This shall amend and supercede any
as she was merely a holder-in-trust of shares of JAKA Investments which previous resolution.”
continued to be the true stockholder of Mr. & Ms. Petitioner contends that
she was a holder of proper stock certificates and that the transfer was A few years later, or on 13 March 1991, Homero Villasis, Preston Villasis,
recorded. She further contends that even in the absence of the actual Reginald Villasis and Dimas Enriquez filed an affidavit-complaint against
certificate, mere recording will suffice for her to exercise all stockholder Salas, et. al. before the Office of the City Prosecutor of Iloilo, as a result
rights, including the right to file a derivative suit in the name of the of which 2 separate criminal informations, one for falsification of a public
corporation. The SEC Hearing Panel dismissed the suit. On appeal, the document under Article 171 of the Revised Penal Code and the other for
SEC En Banc found for petitioner. CA reversed the SEC En Banc estafa under Article 315, par. 1(b) of the RPC, were filed before Branch 33
decision. of the Regional Trial Court of Iloilo City. The charge for falsification of
Issue: public document was anchored on Salas, et. al.'s submission of WIT's
Whether or not petitioner is the true holder of stock certificates to be able income statement for the fiscal year 1985-1986 with the Securities and
institute a derivative suit. Exchange Commission (SEC) reflecting therein the disbursement of
Ruling: NO. corporate funds for the compensation of Salas, et. al. based on Resolution
Sec 63 of the Corporation Code envisions a formal certificate of stock 4, series of 1986, making it appear that the same was passed by the board
which can be issued only upon compliance with certain requisites. First, on 30 March 1986, when in truth, the same was actually passed on 1 June
the certificates must be signed by the president or vice-president, 1986, a date not covered by the corporation's fiscal year 1985-1986
countersigned by the secretary or assistant secretary, and sealed with the (beginning May 1, 1995 and ending April 30, 1986). Thereafter, trial for
seal of the corporation. A mere typewritten statement advising a the two criminal cases (Criminal Cases 37097 and 37098), was
stockholder of the extent of his ownership in a corporation without consolidated. After a full-blown hearing, Judge Porfirio Parian handed
qualification and/or authentication cannot be considered as a formal down a verdict of acquittal on both counts dated 6 September 1993
certificate of stock. Second, delivery of the certificate is an essential without imposing any civil liability against the accused therein. Villasis,
element of its issuance. Hence, there is no issuance of a stock certificate et. al. filed a Motion for Reconsideration of the civil aspect of the RTC
where it is never detached from the stock books although blanks therein Decision which was, however, denied in an Order dated 23 November
are properly filled up if the person whose name is inserted therein has no 1993. Villasis, et. al. filed the petition for review on certiorari.
control over the books of the company. Third, the par value, as to par Significantly on 8 December 1994, a Motion for Intervention, dated 2
value shares, or the full subscription as to no par value shares, must first December 1994, was filed before this Court by Western Institute of
be fully paid. Fourth, the original certificate must be surrendered where Technology, Inc., disowning its inclusion in the petition and submitting
the person requesting the issuance of a certificate is a transferee from a that Atty. Tranquilino R. Gale, counsel for Villasis, et. al., had no
stockholder. authority whatsoever to represent the corporation in filing the petition.
The certificate of stock itself once issued is a continuing affirmation or Intervenor likewise prayed for the dismissal of the petition for being
representation that the stock described therein is valid and genuine and is utterly without merit. The Motion for Intervention was granted on 16
at least prima facie evidence that it was legally issued in the absence of January 1995.
evidence to the contrary. However, this presumption may be rebutted.
Aside from petitioner’s own admissions, several corporate documents Issue: Whether the grant of compensation to Salas, et. al. is proscribed
disclose that the true party-in-interest is not petitioner but JAKA. It should under Section 30 of the Corporation Code.
be emphasized that JAKA executed, a deed of sale over 1,000 Mr. & Ms.
shares in favor of respondent Eugenio D. Apostol. On the same day, Held: Directors or trustees, as the case may be, are not entitled to salary or
respondent Apostol signed a declaration of trust stating that she was the other compensation when they perform nothing more than the usual and
registered owner of 1,000 Mr. & Ms. shares covered by a Certificate of ordinary duties of their office. This rule is founded upon a presumption
Stock. And, there is nothing in the records which shows that JAKA had that directors/trustees render service gratuitously, and that the return upon
revoked the trust it reposed on respondent Eugenia D. Apostol. Neither their shares adequately furnishes the motives for service, without
was there any evidence that the principal had requested her to assign and compensation. Under Section 30 of the Corporation Code, there are only
transfer the shares of stock to petitioner. In fine, the records are unclear on two (2) ways by which members of the board can be granted
how petitioner allegedly acquired the shares of stock of JAKA. compensation apart from reasonable per diems: (1) when there is a
Thus, for a valid transfer of stocks, the requirements are as follows: (a) provision in the by-laws fixing their compensation; and (2) when the
There must be delivery of the stock certificate; (b) The certificate must be stockholders representing a majority of the outstanding capital stock at a
endorsed by the owner or his attorney-in-fact or other persons legally regular or special stockholders' meeting agree to give it to them. Also, the
authorized to make the transfer; and, (c) to be valid against third parties, proscription, however, against granting compensation to director/trustees
the transfer must be recorded in the books of the corporation. At most, in of a corporation is not a sweeping rule. Worthy of note is the clear
the instant case, petitioner has satisfied only the third requirement. phraseology of Section 30 which state: "[T]he directors shall not receive
Compliance with the first two requisites has not been clearly and any compensation, as such directors." The phrase as such directors is not
sufficiently shown. without significance for it delimits the scope of the prohibition to
*The basis of a stockholder’s suit is always one in equity. However, it compensation given to them for services performed purely in their
cannot prosper without first complying with the legal requisites for its capacity as directors or trustees. The unambiguous implication is that
institution. The most important of these is the bona fide ownership by a members of the board may receive compensation, in addition to
stockholder of a stock in his own right at the time of the transaction reasonable per diems, when they render services to the corporation in a
complained of which invests him with standing to institute a derivative capacity other than as directors/trustees. Herein, resolution 48, s. 1986
action for the benefit of the corporation. granted monthly compensation to Salas, et. al. not in their capacity as
members of the board, but rather as officers of the corporation, more
particularly as Chairman, Vice-Chairman, Treasurer and Secretary of
Western Institute of Technology. Clearly, therefore, the prohibition with action in his complaint; and (3) the alleged wrong was in truth a wrong
respect to granting compensation to corporate directors/trustees as such against the stockholders of the corporation generally, and not against Cruz
under Section 30 is not violated in this particular case. Consequently, the or Minterbro, in particular. And while it is true that the complaining
last sentence of Section 30 which provides that "In no case shall the total stockholder must show to the satisfaction of the court that he has
yearly compensation of directors, as such directors, exceed ten (10%) exhausted all the means within his reach to attain within the corporation
percent of the net income before income tax of the corporation during the itself the redress for his grievances, or actions in conformity to his wishes,
preceding year" does not likewise find application in this case since the nonetheless, where the corporation is under the complete control of the
compensation is being given to Salas, et. al. in their capacity as officers of principal defendants or other trustees, as here, there is no necessity of
WIT and not as board members. making a demand upon the directors. The reason is obvious: a demand
upon the board to institute an action and prosecute the same effectively
FILIPINAS PORT SERVICES INC v. GO, ET AL. would have been useless and an exercise in futility.
FACTS:
The case involves a petition for review on certiorari. Bottom line, when it comes to cases involving two or more trustees, an
individual trustee can file a derivative suit duly following the requisites
We have here Eliodoro C. Cruz suing on behalf of the stockholders without the need to exhaust internal remedies where the trusteeship is
of Filipinas Port Services alleging that there has been numerous cases of under the complete control of the other trustees for it will be a waste of
mismanagement by the board of directors: time.
1. creation of an executive committee not provided for in the by-
laws of the corporation Tam Wing Tak vs Ramon Makasiar
2. disproportionate increase in the salary of officials February 22, 2013
3. re-creation of already existing positions Share this...
4. creation of additional positions with holders not doing any work 0 0000
to deserve any monthly remuneration. ADVERTISEMENTS
He prayed for the return of the salary received by all the unnecessarily Business Organization – Corporation Law – Ultra Vires Acts of Corporate
appointed members. Officers – Derivative Suit
The Trial Court sided with the respondent and ruled that the creation of Sometime before November 1992, Vic Ang Siong issued a check to
the executive committee and the additional position was legitimate given Concord-World Properties, Inc. The check amounted to P83.5 million.
that it was provided by the corporation’s by-law. However, the prayer for The check however bounced. In November 1992, Tam Wing Tak filed an
the return of salaries received was granted, even if the positions and the affidavit-complaint for violation of the Anti-Bouncing Checks Law
committee were valid, for the court ruled that Filipinas Port Services is not against Ang Siong. The fiscal did not file a criminal information against
a big corporation requiring multiple executive positions. Ang Siong because apparently Concord-World and Ang Siong are settling
The respondents appealed the decision and they received a favourable out of court (in fact Ang Siong already paid P19 million); and that Tam
decision as the Court of Appeals granted the respondents’ appeal, reversed Wing Tak was not authorized by the Board of Directors of Concord-World
and set aside the appealed decision of the trial court and accordingly to sue Ang Siong. Tam Wing Tak then filed a petition for mandamus to
dismissed the so-called derivative suit filed by Cruz, et al., compel the fiscal to file the information. Judge Ramon Makasiar
Cruz did not take the decision sitting down, hence the petition. dismissed the petition.
To counter the appeal filed by Cruz, respondents also claim that what Cruz ISSUE: Whether or not the petition should be granted.
filed is not a derivative suit. HELD: No. The petition for mandamus shall not lie. There was no grave
The petition was denied and the challenged decision of the CA was abuse of discretion when the fiscal refused to file the information.
affirmed. Only, the Supreme Court clarified the issue involving the Concord-World is the named payee in the check that bounced. As payee,
legitimacy of the derivative suit. Concord-World is the injured party hence only Concord-World can file the
criminal case against Ang Siong but it did not do so because it chose to
ISSUE: amicably settle the issue with Ang Siong. Where a corporation is an
Was the case filed by Cruz, on behalf of Filipinas Port Services Inc., a injured party, its power to sue is lodged with its board of directors or
derivative suit? trustees. This can be delegated but Tam Wing Tak never proved that he
was authorized by the Board of Concord-World.
HELD: But may the suit be considered a derivative suit where the Board’s
YES. authorization may not be had?
No. For a derivative suit to prosper, it is required that the minority
Under the Corporation Code, where a corporation is an injured party, its stockholder suing for and on behalf of the corporation must allege in his
power to sue is lodged with its board of directors or trustees. But an complaint that he is suing on a derivative cause of action on behalf of the
individual stockholder or an individual trustee may be permitted to corporation and all other stockholders similarly situated who may wish to
institute a derivative suit in behalf of the corporation in order to protect or join him in the suit. In this case, this was not complied with. Hence, Tam
vindicate corporate rights whenever the officials of the corporation refuse Wing Tak cannot sue Ang Siong.
to sue, or when a demand upon them to file the necessary action would be
futile because they are the ones to be sued, or because they hold control of Reyes v. RTC of Makati [G.R. No. 165744. August 11, 2008]
the corporation. In such actions, the corporation is the real party-in-interest 03 Oct
while the suing stockholder, in behalf of the corporation, is only a nominal OSCAR C. REYES, petitioner,
part. vs.
Here, the action below is principally for damages resulting from HON. REGIONAL TRIAL COURT OF MAKATI, Branch 142,
alleged mismanagement of the affairs of Filport by its directors/officers, it ZENITH INSURANCE CORPORATION and RODRIGO C.
being alleged that the acts of mismanagement are detrimental to the REYES, respondents.
interests of Filport. Thus, the injury complained of primarily pertains to [G.R. No. 165744. August 11, 2008]
the corporation so that the suit for relief should be by the corporation. FACTS:
However, since the ones to be sued are the directors/officers of the Petitioner and private respondent were siblings together with two others,
corporation itself, a stockholder, like petitioner Cruz, may validly institute namely Pedro and Anastacia, in a family business established as Zenith
a “derivative suit” to vindicate the alleged corporate injury, in which case Insurance Corporation (Zenith), from which they owned shares of stocks.
Cruz is only a nominal party while Filport is the real party-in-interest. The Pedro and Anastacia subsequently died. The former had his estate
Besides, the requisites before a derivative suit can be filed by a judicially partitioned among his heirs, but the latter had not made the same
stockholder or individual trustee are present in this case, to wit: in her shareholding in Zenith. Zenith and Rodrigo filed a complaint with
the Securities and Exchange Commission (SEC) against petitioner (1) a
a) the party bringing suit should be a shareholder as of the time of the derivative suit to obtain accounting of funds and assets of Zenith, and (2)
act or transaction complained of, the number of his shares not being to determine the shares of stock of deceased Pedro and Anastacia that
material; were arbitrarily and fraudulently appropriated [by Oscar, and were
unaccounted for]. In his answer with counterclaim, petitioner denied the
b) he has tried to exhaust intra-corporate remedies, i.e., has made a illegality of the acquisition of shares of Anastacia and questioned the
demand on the board of directors for the appropriate relief but the latter jurisdiction of SEC to entertain the complaint because it pertains to
has failed or refused to heed his plea; and settlement of [Anastacia’s] estate. The case was transferred to. Petitioner
filed Motion to Declare Complaint as Nuisance or Harassment Suit and
c) the cause of action actually devolves on the corporation, the must be dismissed. RTC denied the motion. The motion was elevated to
wrongdoing or harm having been, or being caused to the corporation and the Court of Appeals by way of petition for certiorari, prohibition and
not to the particular stockholder bringing the suit. mandamus, but was again denied.
ISSUES:
Indisputably, petitioner Cruz (1) is a stockholder of Filport; (2) he sought Mercantile Law
without success to have its board of directors remedy what he perceived as (1) Whether or not Rodrigo may be considered a stockholder of Zenith
wrong when he wrote a letter requesting the board to do the necessary with respect to the shareholdings originally belonging to Anastacia.
(2) Whether or not there is an intra-corporate relationship between the Benedicto Hornilla and Federico Ricafort were members of the Philippine
parties that would characterize the case as an intra-corporate dispute? Public School Teachers Association (PPSTA). In 1997, they accused the
Remedial Law Board of Directors of PPSTA of unlawfully spending the funds of PPSTA.
(1) Whether or not the complaint is a mere nuisance or harassment suit However, since the PPSTA was not initiating a complaint against the
that should be dismissed under the Interim Rules of Procedure of Intra- Board of Directors, the two then filed a suit on behalf of PPSTA against
Corporate Controversies; the Board of PPSTA.
(2) Whether or not the complaint is a derivative suit within the In the said suit, the Board of Directors were represented by Atty. Ernesto
jurisdiction of the RTC acting as a special commercial court. Salunat. Hornilla et al were against the legal representation being made by
RULINGS: Salunat for and on behalf of the Board of Directors because of the fact that
Mercantile Law Salunat is part of the ASSA Law Offices. And the ASSA Law Offices
(1) No. Rodrigo must, hurdle two obstacles before he can be considered a happen to be the retained law firm of the PPSTA. In short, Hornilla et al
stockholder of Zenith with respect to the shareholdings originally alleged that there is conflict of interests.
belonging to Anastacia. First, he must prove that there are shareholdings ISSUE: Whether or not there is conflict of interest.
that will be left to him and his co-heirs, and this can be determined only in HELD: Yes. The suit filed by Hornilla et al against the Board of PPSTA is
a settlement of the decedent’s estate. No such proceeding has been a derivative suit. Where corporate directors have committed a breach of
commenced to date. Second, he must register the transfer of the shares trust either by their frauds, ultra vires acts, or negligence, and the
allotted to him to make it binding against the corporation. He cannot corporation is unable or unwilling to institute suit to remedy the wrong, a
demand that this be done unless and until he has established his specific stockholder (in this case a member because PPSTA is non-stock) may sue
allotment (and prima facie ownership) of the shares. Without the on behalf of himself and other stockholders and for the benefit of the
settlement of Anastacia’s estate, there can be no definite partition and corporation, to bring about a redress of the wrong done directly to the
distribution of the estate to the heirs. Without the partition and corporation and indirectly to the stockholders. In such a case, even though
distribution, there can be no registration of the transfer. And without the it was the members who filed the case and not the corporation itself, the
registration, we cannot consider the transferee-heir a stockholder who may real party in interest is still the corporation (PPSTA) and the suing
invoke the existence of an intra-corporate relationship as premise for an members (Hornilla et al) are only the nominal party.
intra-corporate controversy within the jurisdiction of a special commercial Therefore, since it is the corporation suing, Salunat cannot represent the
court. The subject shares of stock (i.e., Anastacia’s shares) are concerned Board Members of PPSTA because he is a member of ASSA Law Office
– Rodrigo cannot be considered a stockholder of Zenith. which is the retained law firm of PPSTA. Surely, there is conflict of
(2) No. Court cannot declare that an intra-corporate relationship exists interest in him representing the Board while his law office represents the
that would serve as basis to bring this case within the special commercial corporation. Salunat was admonished by the Supreme Court.
court’s jurisdiction under Section 5(b) of PD 902-A, as amended because
Rodrigo’s complaint failed the relationship test above. Evangelista & Co. et.al. v. Estrella Abad Santos
Remedial Law FACTS:
(1) Yes. The rule is that a complaint must contain a plain, concise, and On October 9, 1954, a co-partnership with herein petitioners as capitalist
direct statement of the ultimate facts constituting the plaintiff’s cause of partners was formed under the name “Evangelista & Co.” The Articles of
action and must specify the relief sought. Section 5, Rule 8 of the Revised Co-partnership was, however, amended on June 7, 1955 so as to include
Rules of Court provides that in all averments of fraud or mistake, the herein respondent, Estrella Abad Santos, as an industrial partner.
circumstances constituting fraud or mistake must be stated with
particularity. These rules find specific application to Section 5(a) of P.D. Consequently, on December 17, 1963, Abad Santos filed suit against the
No. 902-A which speaks of corporate devices or schemes that amount to three (3) capitalist partners, alleging that the partnership, which was also
fraud or misrepresentation detrimental to the public and/or to the made a party-defendant, had been paying dividends to the partners except
stockholders. to her. It was further alleged that despite her requests that she be allowed
Allegations of deceit, machination, false pretenses, misrepresentation, and to examine partnership books, to give her information regarding the
threats are largely conclusions of law that, without supporting statements partnership affairs and to receive her share in the dividends declared by
of the facts to which the allegations of fraud refer, do not sufficiently state the partnership, the petitioners refused and continued to refuse. She
an effective cause of action. Fraud and mistake are required to be averred therefore prayed that the petitioners be ordered to render an accounting of
with particularity in order to enable the opposing party to controvert the the partnership business and to pay her the corresponding share in the
particular facts allegedly constituting such fraud or mistake. Tested against dividends.
these standards, charges of fraud against Oscar were not properly
supported by the required factual allegations. While the complaint ISSUE:
contained allegations of fraud purportedly committed by him, these Whether or not the Articles of Co-partnership shall be considered as a
allegations are not particular enough to bring the controversy within the conclusive evidence of respondent’s status as a limited partner?
special commercial court’s jurisdiction; they are not statements of ultimate
facts, but are mere conclusions of law: how and why the alleged HELD:
appropriation of shares can be characterized as “illegal and fraudulent” NO. The Court held that despite the genuineness of the Articles of
were not explained nor elaborated on. The case must be dismissed. Co-partnership the same did not express the true intent and agreement of
(2) No. The allegations of the present complaint do not amount to a the parties, however, as the subsequent events and testimonial evidences
derivative suit. First, as already discussed above, Rodrigo is not a indicate otherwise, the Court upheld that respondent is an industrial
shareholder with respect to the shareholdings originally belonging to partner of the company.
Anastacia; he only stands as a transferee-heir whose rights to the share are
inchoate and unrecorded. Second, in order that a stockholder may show a Article 1789 provides that ‘An industrial partner cannot engage in
right to sue on behalf of the corporation, he must allege with some business for himself, unless the partnership expressly permits him to do
particularity in his complaint that he has exhausted his remedies within the so; and if he should do so, the capitalist partners may either exclude him
corporation by making a sufficient demand upon the directors or other from the firm or avail themselves of the benefits which he may have
officers for appropriate relief with the expressed intent to sue if relief is obtained in violation of this provision, with a right to damages in either
denied. Lastly, Court found no injury, actual or threatened, alleged to have case.’ Since 1954 and until after the promulgation of the decision of the
been done to the corporation due to Oscar’s acts. If indeed he illegally appellate court, Abad Santos has served as a judge of the City Court of
and fraudulently transferred Anastacia’s shares in his own name, then the Manila and had been paid for services rendered allegedly contributed by
damage is not to the corporation but to his co-heirs; the wrongful transfer her to the partnership. Though being a judge of the City Court of Manila
did not affect the capital stock or the assets of Zenith. cannot be characterized a business and/or may be considered an
In summary, whether as an individual or as a derivative suit, the RTC – antagonistic business to the partnership, the petitioners, subsequent of
sitting as special commercial court – has no jurisdiction to hear Rodrigo’s petitioners’ answer to the complaint, petitioners reached the decision that
complaint since what is involved is the determination and distribution of respondent be excluded from and deprived of her alleged share in the
successional rights to the shareholdings of Anastacia Reyes. Rodrigo’s interest or participation as an alleged industrial partner in the net profits or
proper remedy, under the circumstances, is to institute a special income of the partnership.
proceeding for the settlement of the estate of the deceased Anastacia
Reyes, a move that is not foreclosed by the dismissal of his present Having always known the respondent is a City Judge even before she
complaint. joined the partnership, why did it take petitioners so many years before
excluding her from said company? Furthermore, the act of exclusion is
Benedicto Hornilla vs Ernesto Salunat premised on the ground that respondent has always been a partner, an
January 14, 2014 industrial partner. In addition, the Court further held that with the
Share this... consideration of Article 1767 that ‘By a contract of partnership two or
0 0000 more persons bind themselves, to contribute money, property, or industry
ADVERTISEMENTS to a common fund, with the intention of dividing profits among
Commercial Law – Corporation Law – Derivative Suit themselves’, the services rendered by respondent may legitimately be
Legal Ethics – Conflict of Interests considered the respondent’s contribution to the common fund.
LEGASPI TOWERS 300, INC v. AMELIA P. MUER (Jurisdiction;
Class Suit)
MissDennieIdea Uncategorized June 1, 2015 2 Minutes
LEGASPI TOWERS 300, INC., LILIA MARQUINEZ PALANCA,
ROSANNA D. IMAI, GLORIADOMINGO and RAY VINCENT,
Petitioners, vs. AMELIA P. MUER, SAMUEL M. TANCHOCO,
ROMEO TANKIANG, RUDEL PANGANIBAN,DOLORES
AGBAYANI, ARLENEDAL A. YASUMA, GODOFREDO M.
CAGUIOA and EDGARDO M.SALANDANAN, Respondents.
FACTS: Pursuant to the by-laws of Legaspi Towers 300, Inc., petitioners,
the incumbent Board of Directors, set the annual meeting of the members
of the condominium corporation and the election of the new Board of
Directors at the lobby of Legaspi Towers 300, Inc. The Committee on
Elections of Legaspi Towers 300, Inc., however, found most of the proxy
votes, at its face value, irregular, thus, questionable; and for lack of time to
authenticate the same, petitioners adjourned the meeting for lack of
quorum. However, the group of respondents challenged the adjournment
of the meeting. Despite petitioners’ insistence that no quorum was
obtained during the annual meeting held on April 2, 2004, respondents
pushed through with the scheduled election and were elected as the new
Board of Directors and officers of Legaspi Towers 300, Inc. and
subsequently submitted a General Information Sheet to the Securities and
Exchange Commission (SEC). On plaintiffs’ motion to admit amended
complaint to include Legaspi Towers 300, Inc. as plaintiff),the RTC ruled
denying the motion for being improper. Then, petitioners filed with the
Court of Appeals and held that Judge Antonio I. De Castro of the Regional
Trial Court (RTC) of Manila, did not commit grave abuse of discretion in
issuing the Orders denying petitioners’ Motion to Admit Second Amended
Complaint and that petitioners the justified the inclusion of Legaspi
Towers 300, Inc. as plaintiff by invoking the doctrine of derivative suit.
Petitioners’ motion for reconsideration was denied by the Court of
Appeals thereafter. Hence this petition.
ISSUE: Whether or not Derivative Suit proper in this case?
RULING: The Supreme Court DENIED the petition and AFFIRMED the
Decision of the Court of Appeals. Derivative Suit is not applicable. Since
it is the corporation that is the real party-in-interest in a derivative suit,
then the reliefs prayed for must be for the benefit or interest of the
corporation. When the reliefs prayed for do not pertain to the corporation,
then it is an improper derivative suit. The requisites for a derivative suit
are as follows:
1. a) the party bringing suit should be a shareholder as of the time
of the act or transaction complained of, the number of his shares not being
material;
2. b) he has tried to exhaust intra-corporate remedies, i.e., has
made a demand on the board of directors for the appropriate relief but the
latter has failed or refused to heed his plea; and
3. c) the cause of action actually devolves on the corporation, the
wrongdoing or harm having been, or being caused to the corporation and
not to the particular stockholder bringing the suit.
As stated by the Court of Appeals, petitioners’ complaint seek to nullify
the said election, and to protect and enforce their individual right to vote.
The cause of action devolves on petitioners, not the condominium
corporation, which did not have the right to vote. Hence, the complaint for
nullification of the election is a direct action by petitioners, who were the
members of the Board of Directors of the corporation before the election,
against respondents, who are the newly-elected Board of Directors. Under
the circumstances, the derivative suit filed by petitioners in behalf of the
condominium corporation in the Second Amended Complaint is improper.

Вам также может понравиться